You are on page 1of 137

Solutions

of

Electricity & Magnetism

Lesson 20th to 25th

By DC Pandey

20

Current Electricity
Introductory Exercise 20.1

2 pr
q
1. i = , here q = e, t =
t
v
ev
i=
\
2pr
1.6 10-19 2.2 106
=
2 3.14 5 10-11
= 1.12 10
= 1.12 mA

-3

2.0
8.43 1028 1.6 10-19 3.14
(0.5 10-3 )2

= 1.88 10-6 ms -1
3. Yes.
As current always flows in the direction of
electric field.

2. No. of atoms in 63.45 g of Cu = 6.023 1023


\No. of atoms in 1 cm 3 (8.89 g) of Cu
6.023 1023
=
8.89
63.54
= 8.43 1022
As one conduction electron is present per
atoms,
n = 8.43 1022 cm -3 or 8.43 1028 m -3
As i = neAvd
i
vd =

neA

4. False.
In the absence of potential difference,
electrons passes random motion.
5. Current due to both positive and negative
ions is from left to right, hence, there is a net
current from left to right.
dq
6. i = 10 + 4 t
= 10 + 4 t
dt
q

10

0 dq = 0

(10 + 4t ) dt

q = [10t + 2t 2 ]10
0 = 300 C

Introductory Exercise 20.2


rL
1. R =
A

= 1.72 10-8

35
2.05

10-3
3.14
2

= 0.57 W
2. (a)

J=

E
r

i = JA =

EA
r

0.49 3.14 (0.42 10-3 )2


2.75 10-8

= 9.87 A
(b) V = EL = 0.49 12 = 5.88 V
V 5.88
(c) R =
=
= 0.6 W
i 9.87
3. Let us consider the conductor to be made up
of a number of elementary discs. The
conductor is supposed to be extended to form
a complete cone and the vertex O of the cone

3
is taken as origin with the conductor placed
along x-axis with its two ends at x = r and
x = l + r. Let q be the semi-vertical angle of
the cone.
Consider an elementary disc of thickness dx
at a distance x from origin.
Resistance of this disc,
dx
dR = r
A
If y be the radius of this disc, then
A = py2
But y = x tan q
dx
dR = r 2
px tan 2 q
\Resistance of conductor
l +r
rdx
R = dR =
2
r
px tan 2 q
l +r
r
1
R=
p tan 2 q x r

1
r
1

2
p tan q r l + r

R=
But,

rl
pr ( l + r ) tan 2 q

r tan q = a
( r + l) tan q = b
rl
R=
p ab

4. True.

r=

1
s

r s=

1
s =1
s

5. RCu = RFe
4.1 (1 + a Cu DT ) = 3.9 (1 + a Fe DT )
4.1 [1 + 4.0 10-3( T - 20)]
= 3.9 [1 + 5.0 10-3( T - 20)]
4.1 + 16.4 10-3( T - 20)
= 3.9 + 19.5 10-3( T - 20)
3.1 10-3( T - 20) = 0.2
0.2
T - 20 =
3.1 10-3
= 64.5 C
T = 84.5 C

Introductory Exercise 20.3


1. Potential difference across both the resistors
is 10 V.
i1

VB = VA + 2 = 2 V
VC = VA + 5 = 5 V
VD = VC + 10 = 15 V
V - VB
i1 = C
=3 A
1
V - VA 15
i2 = D
=
= 7.5 A
2
2

i2

\
2W

10 V

4W

3. Current in the given loop is


E + 15
i=
8
E + 15
VAB = E - 2i = E - 2
=0
8

10
=5 A
2
10
i2 =
= 2.5 A
4

Hence,

i1 =

and

1W

i1

E =5 V

2. As A is grounded, VA = 0
C

and

4. Effective emf,
E = 8 1 - 2 1 = 6 V
Effective resistance of circuit

5V
10 V

R1

2V

i1

i2
D

i2

A
2W

10 V

i3
R2

R3

10 V

4
R = Rexternal + 10r = 2 + 10 1 = 12 W
E
6
i=
=
= 0.5 A
R 12

5. As R2 = R3 and V1 = V2
Potential difference across R1 is zero.
Hence, current through R1 i1 = 0
and current through R2
V

i2 = 1
R2
10
=
=1 A
10

E
6. i =
R+ r
Also, V = E - ir
i=
(a)

E-V
r
I

(b)

E
r

E
r

Introductory Exercise 20.4


6 =1 - E
E = -5V

1.
12 V

P
E

1W

1A
R

3W
T

2A

Applying KCL at junction R


i=1 + 2 =3 A
VST = VRU = VQP
Taking VST = VRU

And from
VST = VQP
6 = - ir + 12
12 - 6 6
r=
= =2W
i
3
2. Power delivered by the 12 V power supply,
P1 = Vi = 12 3 = 36 W
and power dissipated in 3 W resister,
P3 = i32 R3 = 22 3 = 12 W

Introductory Exercise 20.5


E1 E2 E3 10 4 6
+
+
+ +
r
r2
r3
2 2
1. E = 1
= 1
1
1
1
1 1 1
+
+
+ +
1 2 2
r1 r2 r3
10 + 2 + 3
=
2
= 7.5 V
1 1
1
1
and
= +
+
r r1 r2 r3
1 1 1
= + + =2
1 2 2
1

r=
2
= 0.5 W
E
2. i =
R+ r

Rate of dissipation of energy


E2 R
P = i2 R =
( R + r )2
For maximum or minimum power
dP
=0
dR
( R + r )2 - 2 R ( R + r )

E2
=0
( R + r )4

E2

( R + r )( r - R)
( R + r )4
E 2 ( r - R)
( R + r )3

=0
=0

R=r
3

(
+
)
(
1
)
3( r - R)( R + r )2
R
r
d 2P
= E2

2
( R + r )6
dR

- E2(4r - 2 R)
=
( R + r )4
d 2P
is negative at R = r.
dR2
Hence, P is maximum at R = r
E2r
E2
and Pmax =
=
2
4r
( r + r)
Clearly

3. When the batteries are connected in series


Eeff = 2E = 4V, reff = 2r = 2 W
For maximum power
R = reff = 2W
and Pmax =

2
Eeff
(4)2
=
=2 W
4reff 4 2

4. I g = 5 mA, G = 1 W, V = 5 V
V
5
R=
-G =
-1
Ig
5 10-3
= 999 W
A 999 W resistance must be connected in
series with the galvanometer.

5. G = 100 W, ig = 50 mA, i = 5 mA
igG
50 10-6 100
S=
=
\
i - ig 5 10-3 - 50 10-6
1
1
=
=
1 - 0.01 0.99
100
=
W
99
100
By connecting a shunt resistance of
W.
99
V
6. ig =
G
nV
and R =
- G = ( n - 1) G
ig
15
E
16
Potential gradient
V
15E
k = AB =
L
16 600
E
V/cm
=
640
E
E
(a) = kL L =
= 320 cm
2
2k
E
7E
(b) V = kl =
560 =
640
8
Also, V = E - ir
7E
E - ir =
\
8
E
i=
8r

7. VAB =

AIEEE Corner
Subjective Questions (Level 1)
q ne
1. i = =
t
t
Given,
i = 0.7 , t = 1 s, e = 1.6 10-19 C
0.7 1
it
n= =
\
e 1.6 10-19
= 4.375 108
2. q = it = 3.6 3 3600
= 38880 C
3. (a) q = it = 7.5 45 = 337.5 C
q
(b) q = ne n =
e

=
4. T =

337.5
= 2.11 1021
1.6 10-19

2 pr
1
v
f =
=
v
T 2 pr
=

2.2 106
2 3.14 5.3 10-11

= 6.6 1019 s -1
q
I=
= ef
T
= 1.6 10-19 6.6 1019
= 10.56 A

6
-8

= 1.7 10 W - m
l = 24.0 m
2
2
d
2.05

A = p = 3.14
10-3
2
2

5. (a) I = 55 - 0.65 t
dq
I=
dt

dq = Idt
q = I dt

= 3.29 10-6 m 2

R = 1.7 10-8

q = Idt = (55 - 0.65 t ) dt

t2
= 55 [ t ]80 - 0.65
2 0
= 440 - 20.8 = 419.2 C
(b) If current is constant
q
419.2
I=
=
= 52.4 A
t
8
6. i vd
\

vd 2

vd 2 =

vd1

= 6.00 10-4 ms -1
7.

vd =
=

= 0.12 W
L
10.
R=r
A
rL
A=
R
If D is density, then
D r L2
R
8.9 103 1.72 10-8 (3.5)2
=
0.125
-2
= 1.5 10 kg = 15 g

m = DV = DA L =

i2
i1
6 . 00
i2
vd1 =
1.20 10-4
i1
1 . 20

i
neA
1
8.5 1028 1.6 10-19 1 10-4

= 0.735 10-6 ms -1
= 0.735 mm/s
l
103
t=
=
vd 0.735 10-6
= 1.36 109 s = 43 yr
8. Distance covered by one electron in 1 s
= 1 0.05 = 0.05 cm
Number of electrons in 1 cm of wire
= 2 1021
\ Number of electrons crossing a given area
per second
= Number of electrons in 0.05 cm of wire
= 0.05 2 1021 = 1020
q ne
i= =
t
t
1020 1.6 10-19
=
= 1.6 10 = 16 A
1
L
9. R = r
A
Given,
r = 0.017 mW - m

24.0
3.29 10-6

11. At 20C,
R1 = 600 W, R2 = 300 W
At 50C,
R1 = R1(1 + a 1Dt )
= 600 (1 + 0.001 30) = 600 1.03
= 618 W
R2 = R2(1 + a 2Dt )
= 300 (1 + 0.004 30) = 336
R = R1 + R2 = 618 + 336
\
= 954 W
R - R 954 - 900
a=
=
R Dt
900 30
R = 600 + 300 = 900 W
= 0.002 C -1
12. As both the wires are connected in parallel,
VAl = VCu
iAl RAl = iCu RCu
L
L
iAl r Al Al2 = iCur Cu Cu
2
p dAl
p dCu

dCu = dAl

iCu r Cu LCu
iAl r Al LAl

= 1 10-3

2 0.017 6
3 0.028 7.5

= 0.569 10-3 m
= 0.569 mm.

and

17. The circuit can be redrawn as

Current density is maximum when L is


minimum, ie, L = d, potential difference
should be applied to faces with dimensions
2d 3 d.
V
.
J min. =
rd
V VA
(b) i =
=
R rL
Current is maximum when L is minimum
and A is maximum.
Hence, in this case also, V should be applied
to faces with dimensions 2d 3 d
V (2d 3 d ) 6Vd
and imax =
.
=
r ( d)
r
15. (a) R = r

0.104 3.14 (1.25 10-3 )2


14.0
= 3.64 10-7 W - m
V EL 1.28 14
(b) i =
=
=
= 172.3 A
R
R
0.104
(c) i = neAvd
i
vd =
neA
172.3
=
8.5 1028 1.6 10-19 3.14 (1.25 10-3 )2
= 2.58 10-3 ms -1

8 12
= 4.8 W
12 + 8
V
24
I=
=
=5A
Reff 4.8

18. Here, A and C are at same potential and B


and D are at same potential,
8W

24V

R1 = 10 R2
Also, R1 + R2 = 20

10 R2 + R2 = 20

B
6W

4W
D

12W

Hence, the circuit can be redrawn as


A,C

16. For zero thermal coefficient of resistance,


DR = 0
RC a CDT + RFe a Fe DT = 0
R1 - a Fe - 5.0 10-3
=
=
= 10
R2
aC
- 0.5 10-3

8W

Reff =

L
A

RA
r=
L
d
[r = = 1.25 mm = 1.25 10-3 m]
2

12W

24 V

24V

4W

12W

r = 2.84 10-8 W - m
E
V
14. (a) J = =
r rL

20
W = 1.82 W
11
R1 = 20 - R2 = 20 - 1.82 W
= 18.2 W
R2 =

8W

V
0.938
=
= 1.25 V/m
L 75 10-2
E
1.25
(b) J = r =
r
4.4 107

13. (a) E =

6W

B,D

1 1 1
1
1
= + +
+
R 4 8 12 6
6+3 +2+4
=
24
15 5
=
=
24 8
8
R= W
5
= 1.6 W
V 24
i=
=
R 1.6
= 15 A

19. Given circuit is similar to that in previous


question but 4 W resistor is removed. So the
effective circuit is given by

8
V
12
i=
=
R 36 / 13
13
=
A
3

12W

A,C

8W

24 V

6W

21. (a) i =

12 + 6
=3 A
1+2+3
i = 3A

B,D

1 1
1
1
= +
+
R 8 12 6
1 3+2+4 9 3
=
=
=
R
24
24 8
8
R = W = 2.67 W
3
V
24
i=
=
=9 A
R 2.67
20.

6W

4W
12W

12V

6W
3W
D

2W

A
6W

4W
4W

12

3W

12W

2W

A
1W

12 V

B
2W

C
3W

6V

VG = 0
VA = VG + 12 = 12 V
VA - VB = 3 V
VB = 12 - 3 = 9 V
VB - VC = 6 V
VC = 9 - 6 = 3 V
VG - VD = 6 V, VD = - 6 V
(b) If 6 V battery is reversed
12 - 6
i=
=1A
1+2+3
i = 1A

Wheatstone bridge is balanced, hence 4 W


resistance connected between B and C be
removed and the effective circuit becomes

12 V
B
G

A
6W
12V

4W

3W

6V

12W

2W
D

VG = 0,

VA - vG = 12 V, VA = 12 V
12V

9W

6W

12W

VA - VB = 1 V

12V

VB = 11 V
VB - VC = 2 V

36
W
13

VC = 9 V
VD - VG = 6 V

VD = 6 V

9
200
=5 A
5 + 10 + 25

22. i =

= 42.26 W
E
i=
= 0.102 W
Reff

Reading of voltmeter
V = E - ir
= 4.3 - 0.102 1
4.2 W
Reading of Ammeter,
V
4.2
i1 =
=
= 0.08 A
R + Ra 42

5W

200 V

2
10W
0
25W
3

(i) V3 - V0 = 5 25

V3 = 125 V
(ii) V0 - V2 = 5 10
V2 = - 50 V
(iii) V2 - V1 = 5 5
V1 = - 75 V
(iv) V3 - 2 = 5 35 = 175 V
(v) V1 - 2 = - 5 5 = - 25 V
(vi) V1 - 3 = - 200 V

24. Consider the directions of current as shown


in figure.
42V

5W

I1

4W

I2

6W

10V

8W
B

1W

I3

23. (a)

6W
i

i1

4V

Applying KVL in loop 1, 2 and 3, we


respectively get,
I1 + 6 ( I1 - I2 ) + 5I1 = 42

12I1 - 6I2 = 42
(i)

2I1 - I2 = 7
4I2 + 6 ( I2 - I1 ) + 8 ( I2 + I3 ) = 10
(ii)

9I2 - 3 I1 + 4I3 = 5
8 ( I2 + I3 ) + 16I3 = 4
(iii)
2 I2 + 6 I3 = 1
On solving, we get,
I1 = 4.7 A, I2 = 2.4 A, I3 = 0.5 A

i2

S
V

Reff = R|| Rv + Ra + r
50 200
=
+2+1
50 + 200
= 43 W
E
4.3
i=
=
= 0.1 A
Reff
43
\Reading of ammeter, i = 0.1 A
and reading of voltmeter = i ( R|| Rv )
= 0.1 40 = 4 V
E
r
(b)
i
i1

Resistor 5W 1 W
4W
6W
Current 4.7 A 4.7 A 2.4 A 2.3 A
R = 400W

25.

R
A

i2

100W 100W
i2

Reff = ( Ra + R)|| Rv + r
52 200
=
+1
52 + 200

200W

400W
200W

A 100W

100W D
B
200W
100W

i2

8W
16 W
2.9 A 0.5 A

100W
i1

i
i

10 V

i1
i

10 V

10
As Wheatstone bridge is balanced, 100 W
resistance between B and D can be removed,
ie,
100W

A
i2

200W

R2 =

D
10 V

10
1
=
A
300 30
Hence, reading of voltmeter
= Potential difference between B and C
20
V
= 200 i2 =
3
= 6.67 V
i1 = i2 =

R2 RV2
R2 + RV2

E
i

i1

i2
V1

i2
V2

V1 =

V2 =

R1
RV1 + RV2
RV2
RV1 + RV2

E=

= 120 V
2000
E=
200
5000

=
i2 =

= 80 V

R1

V2
S

R2

i1

1
A
20

R1
2000 1

i=
5000 12
R1 + RV1

1
A
30
\Current flowing through
1
1
S = i1 - i2 =
20 30
1
=
A
60
=

V1

E
R1 + R2
200
1
i=
=
A
2400 12
RV1
3000 1
i1 =

i=
5000 12
R1 + RV1

3000
200
5000

(ii) When S is closed,

R1

V2

i=

R2

6000
= 1200 W
5

(b) Current distribution is shown in figure

R1

As
R1 = R2
Hence,
reading of V1 = reading of V2
1200
=
200 = 100 V
1200 + 1200

26. (a) (i) When S is open.

V1

R2'

R1'

200W

100W
i1

R2

Now, R1 and V1 are in parallel and their


effective resistance
R1 RV1
6000
R1 =
=
= 1200 W
R1 + RV1
5
Similarly,
R2 and V2 are in parallel with their effective
resistance,

27. Effective emf of 2 V and 6 V batteries


connected in parallel
E r + E2r1 2 1 - 6 1
E = 1 2
=
r1 + r2
1+1
and

= -2V
rr
1
r = 1 2 = W
r1 + r2 2
= 0.5 W

11
2V

1W

6V
0.5W

4V

0.5W

2V

33. In case of charging


V = E + i r = 2 + 5 0.1 = 2.5 V
34. Clearly current through each branch is zero.

Net emf, E = 4 - 2 = 2 V
28. (a)

2W
B
4W 8W 8W 4W

2W

E1

E2

As E1 > E2
Current will flow from B to A.
(b) E1 is doing positive work
(c) As current flows from B to A through
resistor, B is at higher potential.

2V

35. i1 =

G
i1
E

On shunting
resistance S,

galvanometer
S
G

R + R +
As i1 = i2

GS
S+G

E
E
=
R + G R + R + GS
G+S
GS
R + R +
=R+G
G+S
G2
R =
G+S

31. (a) As voltmeter is ideal, it has infinite


resistance, therefore current is zero.
(b) V = E - ir E = 5.0 V
(c) Reading of voltmeter V = 5.0 V

i2 =

VP - VQ = 50 + 3.0 i
VQ = 100 - (50 + 60)
= - 10 V

R'

i2

3.0W

50 V
2.0W

the

P 0.5
(a) P = Vi V = =
= 5.0 V
i 1.0
(b) E = V - iR = 5 - 2 = 3 V
(c) It is clear from figure that positive
terminal of X is towards left.
150 - 50
30. i =
= 20 A
3+2

150 V

2V

E
R+G

R=2.0W E B

2V

4W

2V

4W

2V

2V

29. i2 R = 2 W < 5 W
Clearly X is doing negative work.

+
i

(i)
(ii)

32. V1 = E - i1r E - 1.5 r = 8.4


V2 = E + i2r E + 3.5 r = 9.4
On solving, we get
r = 0.2 W
E = 8.7 V

1W

4V 0.5W

36.
I

A I1
I2

I2 =

r
V
I=
R+ r
R

r
V

with

12

r
V
=
R + r IR
R IR - V 5 2500 - 100
=
=
r
V
100
100
r=
2500 = 20.16 W
12400

39. S =

ig
i - ig

(G + R)
S

R
G

37.
R=

60 V

300W

400W
V

Let R be the resistance of voltmeter


As reading of voltmeter is 30 V,
1
1
1
+
=
R = 1200 W
R 400 300
If voltmeter is connected across 300 W
resistor,

i - ig
ig

S -G

20 - 10-3

0.005 - 20
10-3
= 79.995 W
L1 - L2
0.52 - 0.4
40. r =
R=
5 = 1.5 W
L2
0.4
=

41. Let R be the resistance of voltmeter


V
A

100W

i
3W

60 V
3.4 V
300W
V

Effective resistance of 300 W resistor and


voltmeter
300 1200
R =
= 240 W
300 + 1200
60
i=
400 + 240
60
=
A
640
3
=
A
32
\Reading of voltmeter,
3
V = iR =
240
32
= 22.5 V
R
38. V2 =
V,
R1 + R2

R2 =

V2 =

rR2
120
=
r + R2
3

= 40 W
40
120
60 + 40

= 48 V

100 R
100 + R
100 R
=5 +
100 + R
3.4
i=
= 0.04
100 R
5+
100 + R
4R
0.2 +
= 3.4
100 + R

Re = 3 + 2 +

400W

R = 400 W
Reading of voltmeter,
100 400
100 R
V =i
= 0.04
100 + R
100 + 400
= 3.2 V
If the voltmeter had been ideal,
Reading of voltmeter
100
=
3.4 = 3.24 V
105
L
R
42. 1 = 1
L2 R2
L1
8
(L1 + L2 = 40 cm)

=
40 - L1 12

L1 = 16 cm

from A.

13
43. S =

ig
i - ig

Maximum power dissipated by the circuit


2
P max = Imax
Re
3
= 15 2.4 = 54 W
2

(G + R)
R=

i - ig
ig

S -G

20 - 0.0224
0.0250 - 9.36
0.0244
= 12.94 W
E
44. (a) i =
RV + r
=

Rv
V
i
r

47. Total power of the circuit, P = P1 + P2 + P3


= 40 + 60 + 75
= 175 W
V2
V2
As P =
R=
R
P
(120)2
=
= 82.3 W
175
48. Thermal power generated in the battery
R

V = iRV =
(b)

(c)

E
RV
RV + r

i
r

r
1
= 1% =
RV + r
100

P1 = i2r = i ( E - V )
= 0.6 W
Power development in the battery by electric
forces
P2 = IE = 2.6 W

RV = 99r = 99 0.45
= 44.55 W
RV
V
=
E RV + r

As RV decreases, V decreases, decreasing


accuracy of voltmeter.
45. (a) When ammeter is connected
E
IA =
RA + R + r
When ammeter is removed
R + R+ r
E
= A
I=
IA
R+ r
R+ r
I
(b) A = 99%
I
R+ r
99
=
RA + R + r 100
1
1
RA =
( R + r) =
(3.8 + 0.45)

99
99
RA = 0.043 W
R+ r
IA
(c) As
, as RA increases, I A
=
I
RA + R + r
decreases, decreasing the accuracy of
ammeter.
r max
36
46. Imax =
=
= 15 A
R
2.4
For the given circuit
1
3
Re = R + R = R
2
2

49. The given circuit can be considered as the


sum of the circuit as shown.
2W
35
A
16

2W

21/6A 2W

2/16A

14/6A

3W

7V

5/16A

3W
1V

2W
2A

3
A
16

2W
1A
3W

1V

7V

P1 = 7 2 = 14 W,
P2 = - 1 1 = 1 W
E1 - E2 12 - 6
50. (a) i =
=
= 0.5 A
R1 + R2 4 + 8
\

(b) Power dissipated in R1 = I 2 R1 = 1 W


and power dissipated in R = I 2 R2 = 2 W
(c) Power of battery E1 = E1I
= 12 0.5 = 6 W (supplied)
Power of battery E2 = E2I
= - 6 0.6 = - 3 W (absorbed)

14
E
12
51. I =
=
=2A
R+ r 5+1

1 1 1
1
1
54. (a)
= + +
+
R 4 6 14 4
4W

(a) P = EI = 12 2 = 24 W
(b) P1 = I 2 R = 22 5 = 20 W
(c) P2 = I 2r = 22 1 = 4 W

4W
4W
6W

8W

2W

4W

8W

6W

4W

52. (a)

2W

4W

1.60W
I1
I3

4W

I2 2.40W

6W

4.80W

14W

4W

1 31
=
R 42
42
R=
31
1
1
1
1
(b)
=
+
+
Re 2 R 2 R R

28.0V

1
1
1
1
=
+
+
R R1
R2
R3
1
1
1
1
=
+
+
R 1.60 2.40 4.80
R = 0.80 W
V 28.0
(b) I1 =
=
= 17.5 A
R1 1.60
V 28.0
=
= 11.67 A
I2 =
R2 2.40
V 28.0
=
= 5.83 A
I3 =
R3 4.80

R
A

B
R

2R

(c) I = I1 + I2 + I3 = 35.0 A

P2 =

V 2 (28)2
=
= 326.7 W
R2
2.4

V 2 (28)2
P3 =
=
= 163.3 W
R3
4.8
V2
R
Resistor with least resistance will dissipate
maximum power.
V2
53. (a) P =
V = PR
R
= 5 15 103 = 2.74 102
(f) As, P =

= 274 V
V
(120)2
(b) P =
=
= 1.6 W
R 9 103
2

2R

(d) As all the resistance connected in


parallel, voltage across each resistor is
28.0 V.
V 2 (28)2
(e) P1 =
=
= 490 W
R1
1.6

R
R

B A

Wheatstone bridge is balanced


R

Re =
2
i2+ i3 4W
2W

(c)
i1+ i2
A

i2

i3

i3
3W

i1

1W

2W

3W

i1 i + i
1 2
B
i1

1W

4W
2W

3W

1W

3W

1W

2W
B

15

2.4W
2W

2W

6W

2W

6.4

2W

i3 i4
(d)

i4

2W

10W

2W

2W

4W

5W

11W

i2

R2 R3
R1 + R2 + R3
R1 R3
RB =
R1 + R2 + R3

10W

10W 10W
5W
B

10W
10W

A
10W

10W

RC =

A
10W

(f)

25W
B

R2 R3
R1 + R2 + R3

10W

5W

10W

RA =

i 1 + i 2 + i3
B

O
1W

By Star-Delta Method
10W

5W 5W

5W

8W

i1 10W

10W

1
W
1W 2

i3

i4
5W

10W

10W
D

5.8W

5W

i2

10W

2W
4W

8W

B A

8.5R

1.52W

10W

5W

i3

i1 + i2 + i3
A

(e) A

2W

8W
2W

6W

A
A

2W
4W

A
B

2R

10W

5W
A

4.17W
A

2R

5W

R
A

2R

3R

2R

6
W
5

2R

As
circuit
is
symmetrical
about
perpendicular bisector of AB, lying on it are
at same potential.

16
F

2i1+i2
i1 1W
i1
2W

i1
B

2W

E
C

1W

R2 3 R
=
= 0.6
R1 5 R

R2 = 0.6 R1

2W

5W

5W

1W

56. F

A
8W

2W

15W
6W

40W
E

1W

15W

0.71W
A

2W

Clearly C and D, E and F are at same


potential.
B

15W

20W
30W

2W

1W

1W

2W

C,E

15W
8W

6W

40W

20W

30W D,F

30W
46W

2W

8W

55.
A

R2 = 3R

2W

2W

2W

2W

2W

1W

i1

(g)

Let R be the resistance of each conductor,


and R1 be the effective resistance between A
and F in first case then,
\
R1 = 5 R
If R2 be effective resistance between A and F
in second case then,

50W

Here, C and E , D and F are at same


potential.
Re = 23.3 W
r

57. (a)
a

r
b

2
r
3

2r
a

5
r
8

r
b a

5
r
3
B
r

17
b

(b)

2r

a
a

r/ 3

4
r
3

2r

b
r

(c)

r
b

a b

As Wheatstone bridge is balanced.


r
58. Re =
2
i1i3

(b)

i3

2i1+i2 i1
a
i
i1 2

2r
a b

i3

i1

i3

i3

i1

i2

i1i3
2r

As Wheatstone bridge is balanced


b

r
r
r
r

r
r

(d)

a r

r
4

r
r

r
r

a
r

(e)

r
a

8r/3

r
r

b
r

2r

2r

2r/ 3

8r/ 3

4/5r
b

Objective Questions (Level-1)


1. When ammeter is connected in series
Re = R + RA
Hence, net current decreases. So RA should
be very low.
2. Amount of charge entering per second from
one face is equal to the amount of charge
leaving per second at the other, hence I is
constant.
Again,
I
= not constant.
vd =
neA
eF
As
t
vd =
m

E=

3. R =

mvd
= not constant
et

V
I

[R] =

[V] [ML2T -3I-1 ]


=
[I]
[I]
= [ ML2T -3I-2 ]

1
4. s =
r
As unit of resistivity is ohm-m and
unit of s is ohm -1- m -1.
5. Fact.

r
r

a
b

2r/3

r
r

2r
2r

18
6. E = I ( R + r )
Case I
E = 0.5 (3.75 + r )
Case II

I1 + I2 + I3 = 0
6 - V0 3 - V0 2 - V0
+
+
=0
6
3
2
6 - V0 + 2 (3 - V0 ) + 3(2 - V0 ) = 0

E = 0.4 (4.75 + r )
On solving
r = 0.25 W, E = 2 V
5
I
50
7.
=
I = Ig
2
I g 20
S=

Ig
I - Ig

GG =

I - Ig

Ig

I
I
13. vd =
=
neA ne pr 2
vd =

V0 = 3 V

v
v
2I
= d =
nep (2r )2 2 2

14. Voltmeter has higher resistance than


ammeter.
Again higher the range of voltmeter, higher
will be its resistance.
1
R2
15. I2 =
I
1
1
1
+
+
R1
R2
R3

3
12
2
= 18 W

=
1
I
50
Ig
G
S=
G=
I - Ig
49

8. I g = 2% I =

R1

V2
9. P =
R

I1
2

P + DP =

R2=20W

V
R + DR

R3=15W

As R l
DR = - 10% R
V2
V2 1

DP =
=
- 1 P
0.9 R
R 0.9

10
=
P 11% P
9
10. Potential difference between any two points
is zero.
l -l
11.
r= 1 2 R
l2
75 - 60
=
10
60
= 2.5 W
12. (b) By applying KCL at O
A

I1

I2

I3

1
I
1
1

=
-
+
R1 I2 R2 R2
R3
0.8
1
1
=
-
+

0.3 20 20 15
1
=
60
R1 = 60 W
E
16. (d) I1 =
, V1 = I1 RV
RA + RV

I1
B
I2

3W

6W
O V0
2W
I3
C

= E - I1 RA
If resistance is connected in parallel with
voltmeter,

19
R
A

Pe =

19. As bulb A is in series with entire circuit.

I2

20. I =

I2 =

and

V2
= n 2P
Re

E
> I1
RRV
RA +
R + RV

Vab = E2 - Ir2 = 0
18
31 = 0
R+3

V2 = E - I2 RA < V1

17. Before connectivity resistance is parallel


with ammeter
A

E1 + E2
18
=
R + r1 + r2 R + 3

R=3 W

R
21. I2 =
I
R + RV
I

I1

I1

I2
E

I1 =

= E - I1 RA
After connecting resistance in parallel to the
ammeter.

R + 2500 = 125 R
2500
R=
W 100 W
24

RA

22.

I2

E
,
RA
+ RV
2
1
Reading of ammeter = I2
2
E
1
=
> I1
RA + 2 RV 2
2E
V = I2 RV =
< 2V1
RA + 2 RV
I2 =

18. Re =

V
R
=
I
RV
R + RV
100
R
=
5
2500 R + 2500

E
, V1 = I1 RV
RA + RV

R/10

R/10

R/10

R/10

R/10

R/10

R/10

R/10

R/10

R/10

R/5
R/5
R/5

R
n2

R/ 25

R/5
R/5

R/n
R/n

23.
R/n
2

P=

V
R

R1 20 1
=
=
R2 80 4

(i)
R1 + 15 40 2
=
=
R2
60 3

20
R1 15 2
+
=
R2
R2 3
15 2 1 5
= - =
R2 3 4 12

30. (a) VAB = kL = 0.2 100 = 20 mV


RAB
VAB =
E
RAB + R
RAB
0.02 =
2

RAB + 490

R2 = 36 W,
R
R1 = 2 = 9W
4

24. (b) As V1 =

25. (d)

VAB = - I (25 + 15)


1
= - 40 - 4 V
9

E
RPB

V
, R1 = R2
2
RV 100
= 50
100 + RV

RAB + 490 = 100 RAB


490
RAB =
4.9 W
99

RV = 100 W
2
=
= 0.4 W
+ r 4+1

31. (c) When key is open,

VAB = IRAB = 1.6 W


VPB 1.6
K =
=
= 0.016 V/cm
L
100
E
1.2
L= 1 =
= 75 cm
K 0.016

2R

2R

3R

3R

I1

I1
A

26. (d) VAB = 3 2 + 3 + 1 4 - 2 + 6 1


= 17 V
1A

2E
3R

I1 =

A
E

3/2R

2A
3R

2W
3A

2W

2W
A

27. (c) Ee =

2W
1W

1W

28. (a) V =

B
3V

2V

When key is closed

E1r2 + E2r1
=2V
r1 + r2
rr
re = 1 2 = 0.5 W
r1 + r2

For maximum power R = re


and

I1

Pmax =

2R
2R/3

2R

I2

Ee2
(2)2
=
=2W
4re 4 0.5

I2

A
E

4R/3

R
E
R+ r

2.2

r = - 1 R =
- 15
V

1.8

10
=
W
9
E1 - E2
29. (d) I =
R1 + R2 + r1 + r2
10 - 5
1
=
= A
25 + 15 + 2.5 + 2.5 9

I2
A

3E
4R
I1 8
=
I2 9

I2 =
\

2R/3

21
1
I
32. (b) S =
G = 34 3663
33
I - Ig
I
34
= 111 W
Ig

a
b

r
r

33. (b)

r
r

A
r

r
r

B A
r

B
r

= 2.5W
36. (c) Let RAD = RBC = x

2r

1W

2r/3
B A

5/3r

5/11r

1W

2r/3

11
1.5
5
= 3.3 W

r
r

r
r

B A

r
r

r
r

r
r

r/2

2r

1W

1W

P
R
B

2r

R
R

R
Q

R
R1 =
=3 W
6
l2
R2 =
R2 = 15 W
l1 + l2

R/2

R/3

R
Q

As the circuit is symmetrical about


perpendicular bisector of AB, all points
lying on it are at same potential.
L1
R
L1 + L2

1W

2r

B A

1W

1W 1W

38.

2r

35. (c) R1 =

1W

2 R2 + RR0 + R2 + RR0 = 2 RR0 + R02

3 R2 = R02
R

R= 0
3

r
A

1W

Clearly x < 1 as 1 W resistor is in parallel


with some combination.
Now
RAB = x + 1 + x
= 2x + 1
As x < 1
1 < RAB < 3
R ( R + R0 )
37. (d) RAB = R +
= R0
2 R + R0

5
r
11

1W

r=

34. (b)

1W

1W

5/3r

Re =

Hence R1 and R2 are in parallel


R1 R2
Re =
R1 + R2

2r

60 R1

R2

5
6 R = 25W
Q

5
11 R

22
4
R1 =
pr = 2 = R2
2pr
4
4
R3 =
2 r =
2pr
p
1
1
1
1
=
+
+
Re R1 R2
R3
1 1 p 4+ p
= + + =
2 2 4
4
4
Re =
W
4+ p

39. Wheatstone bridge is balanced.


A

P
R

R
2R/3

2R/3

R
R

R
R

B
R

2R/3

2R/3

B
A
R

4R/3 4R/3

4R/3

7R/3

7R/3

4R/3

42. (d) Points C and D are shorted hence the


portion above line CD can be removed.

7
R
6
L1
1
40. (d) R1 =
R=
R
L1 + L2
12

Re =

=3 W

30

A
B

R2 =

L2
11
=
R = 33 W
L1 + L2 12

41. (a) Resistance per unit length of wire


4
=
2 pr

R
R

R
R/2

B
R

R/2
R/2

R/2
R2

R/2

R/2

R/2

R/2

3R/2

B
A
B

R
R

R3

R/2

43. (b) As AB is line of symmetry,


we can fold the network about AB.

= 2.75 W

R1

R1 and R2 are in parallel,


3 33
R1 R2
Re =
=
R1 + R2 3 + 33

R/2

R/2

23

JEE Corner
Assertion and Reason
1. (d) V = IR, If V = 0 either I = 0 or R = 0

P=

2. (b) As all the resistors are in parallel


potential difference is same, hence
V2
is maximum if R is minimum.
P=
R
I 2t r
3. (b) dH = I 2dRt =
dH
A
I is same everywhere, hence portion having
less area is more heated.
I
Again J =
A
J A > J B.
\
Reason is also correct but does not explain
assertion.
4. (b) Both assertion and reason are correct but
reason does not explain the cause of decrease
in voltmeter reading.
5. (b) As RA < RV , more current passes through
ammeter when positions of ammeter and
voltmeter are interchanged and potential
difference across voltmeter becomes less that
emf of cell.
6. (c) During charging current inside the
battery flows from positive terminal to
negative terminal. Reason is false while
assertion is true.
E
7. (d) I =
is maximum when R is zero
R+ r
hence reason is false.

E2 R
is maximum at R = r.
( R + r )2

V
V2
,P=
both I and P are inversly
R
R
proportional to R hence both decrease with
increase in R which increases with
temperature.
According to Ohms law V I not V = IR.
As R can be variable also.

8. (c) I =

9. (d) Drift velocity is average velocity of all the


electrons but velocity of all electrons is not
constant.
L
10. (a) R = r
A
m
r= 2
ne t
with increase in temperature, electron
collide more frequently, i.e., t decreases,
increasing r and hence R.
E r + E2r1
11. (d) E = 1 2
\ E1 < E < E2
r1 + r2
If E1 < E2
rr
r = 1 2 , r < r1, r < r2
r1 + r2
12. (d)

R1 L1
=
R2 L2

Hence there is no effect of one while


measuring using meter bridge.

Objective Questions (Level-2)


E - E1 1.5 - 1.3
1. (b) I = 2
=
r1 + r2
r1 + r2
0.2
=
r1 + r2
V = E1 + Ir1

1.45 = 1.3 +

0.2
r1
r1 + r2

r1
0.15
=
0.2r1 = 0.15r1 + 0.15r2
r1 + r2
0.2
0.05r1 = 0.15 r2 r1 = 3 r2

(i)

2. (c) Let R = Resistance of voltmeter,


ER
= 198 V
V1 =
R1 + R
V2 =

ER
ER
=
= 180 V
R2 + R 2 R1 + R
2 R1 + R 198 11
=
=
R1 + R 180 10

20 R1 + 10 R = 11 R1 + 11 R
9 R1 = R

(i)
(ii)

24
From Eq. (i),
ER
= 198
R1 + R
10 R1
E = 198
= 220 V
9 R1
3. (b) P = I 2 R
As R is same for all bulbs and maximum
current passes through bulb A, it will glow
most brightly.
V
4. (c) R + RA =
=5W
I
R = 5 - RA < 5 W
L1 - L2
10
5. (a) r =
R=
132.40
L2
60
22.1 W
6. (b) Current through R when S is open.
E1 + E2
I1 =
R + r1 + r2
Current throughR when S is closed
E1
I2 =
R + r1
DI = I2 - I1
E1 + E2
E1
=
R + r1 R + r1 + r2
E1r2 - E2( R + r1 )
=
( R + r1 )( R + r1 + r2 )
DI = + ve if E1r2 > E2( R + r1 )
7. (a) VA = IR
2
I
3
I
x

35
=1
\Required ratio =
35
L - L2
x-y
9. (a) r = 1
R=
R
L2
y
R - 20
t - 10
10. (d)
=
40 - 20 30 - 20
R = t + 10
E
10
I=
=
R t + 10
dq
10
=
dt t + 10
30 10
30
q=
dt = 10[log e( t + 10)]
10
10 t + 10

= 10 log e 2
11. (b) Let l1 length is kept fixed and l2 is
stretched,
l
l
R1 = r 1 , R2 = r 2
A
A
Initial resistance,
(i)
R = R1 + R2
3
Now full is stretched times, ie,
2
3
l2 = ( l1 + l2 ) - l1
2
1
= ( l1 + 3 l2 )
2
A l
2 Al2
A2 = 2 2 =
l2
l1 + 3 l2
1
( l1 + 3 l2 )2
2
R2 = r
2 Al2

y
1
I
3

R2 = r

2
I 1.5 R = IR
3
1
VC = I 3 R = IR
3
VA = VB = VC

Now,

VB =

8. (d) Current through 15 W resistor


30
=
=2A
15
VBC = (2 + 5) 5 = 35 V
Voltage drop across R = 100 - (30 + 55)
= 35 V

12. (b)

( l1 + 3 l2 )2
4 Al2

R = 4 R
R1 + R2 = 4 ( R1 + R)
( l + 3 l2 )2
l1 + 1
= 4( l1 + l2 )
4l2
l2 1
=
l1 7
l2
1
=
l1 + L2 8

l1
X
=
l1 = 40 cm
R 100 - l1

If R = 8 W

25
l1
X
=
R 100 - l1
l1 = 60 cm
l1 - l1 = 20 cm

13. (c) I1 =

when, k1 is open and k2 is closed, net


resistance is
100 100
Rnet = r + R1 +
= ( r + R1 + 50)
100 + 100
I0
E
(ii)
\
=
2 r + R1 + 50

0.1
I
0.1 + 9.9

The above two equations are satisfied if r = 0


and R1 = 50 W.

10 mA

9W
A
I1

16. (b) 20 W, 100 W and 25 W resistors are in


parallel.

I2
0.1W

0.9W

4W
+

4W

20W

25W

But I1 = 10 mA
10
I=
10 mA = 1000 mA
0.1
= 1 kA

6W

6W

4A

R = 20 W
V = IR = 80 V

14. (d) Effective emf of two cells


E r + E2r1 2 6 + 4 2
E= 12
=
r1 + r2
2+6
20
=
= 2.5 V
8
RAB = 4 4 = 16 W
RAB
16
VAB =
E0 =
12
R + RAB
4 + 16

17. (a) Hence, points A and C, B and D are at


same potential.
V
R

C
A

V
k = AB = 2.4 V/m
L

The equivalent circuit is given by

Now, E = kl

E 2.5 25
L= =
=
k 2.4 24

R
R

A, C

15. When k1 and k2 both are closed, the


resistance R1 is short circuited. Therefore
net resistance is

B, D

k1
E, r

V
R/3

100 100
= r + 50
100 + 100
E
I0 =
r + 50

Rnet = r +
\

E, r

= 9.6 V

10W

100W

E, r

(i)

26
E

=1A
R
+r
3
R
V = I =3 V
3
Ig
r
18. (c) S =
G , G = r, S =
I - Ig
4
I=

1
1
(I - Ig ) Ig = I
4
5
= 0.006 A
10 5
19. (d) I1 =
= A
14 7

21. (a) Voltage sensitivity of voltmeter


1

Resistance of voltmeter
Vs1
R +G
= 2
\
Vs2
R1 + G
R2 + 50
30
=
20 2950 + 50

Ig =

I1

8W

R2 + G =

30 3000
= 4500
20
R2 = 4450 W

22. (b) For x = 0

6W

VAB = E
E
k1 =
L
EL1
E0 = k1L1 =
L

P
4W
I

I2

3W
A
10 V

10
A
I2 =
7
VP - VA = I 2 4
40
A
=
7
40
A
VP - VB = I1 8 =
7
VA - A B = 0
Another method
R
R
As, 1 = 3 , VB = VA
R2 R4
20. (b) For series connection
V1
R
= 1
V2 R2
R1 3
\
=
R2 2
L
rL
Now, R1 = r 1 = 12 ,
A1 pr1
L
rL
R2 = d 2 = 22
A2 pr2
R1 L1 r2
=

R2 L2 r1

\
r1
=
r2

r2 1
=
r1 2

For x = x (say)
RAB
E
RAB + x

VAB =
k2 =

RABE
( RAB + x ) L

E0 = k2L2 =

RABEL2
( RAB + x ) L

(ii)

From Eqs. (i) and (ii),


R + L2
L1 = AB
( RAB + x )
20 =

10 30
10 + x

x =5W

23. (d) To obtain null point similar terminal of


both the batteries should be connected.
24. (c) Wheatstone bridge is balanced.
20W
1.4A

4W
15W

I1 20W

4W

50W

10W

1.4A
I2

2 6
R2L1
=
=2
R1L2
3 1

(i)

50W

10W

R2
I1 =
I
R1 + R2
60
=
1.4
84
=1A

27
25. (b)

L1 - L2
R
L2
L2r
R=
L1 - L2
490 10
=
= 490 W
10

29. (c)

r=

E,B,H
R
A

B
E

A,C,F

10V
F

R
R

50W G 10W

More than One Correct Options

10V

I1

R
R

If connected in series
1
1
1
=
+
P P1 P2

I2

I3

t = t1 + t2
If connected in parallel
P = P1 + P2
tt
t= 12

t1 + t2

10V

10V

10
I1 =
= 2.5 A
2 2
26. (b) Effective resistance of voltmeter
3 kW resistor,
3 6
R1 =
= 2 kW
2+6
R1
2
V1 =
E = 10 = 5 V
R1 + R2
4
27. (d) P1 = P2 = P3, Clearly R2 = R3
R2
R1

and

31. E =

E1r2 + E2r1 6 3 + 5 2
=
r1 + r2
2+3

= 5.6 V
As there is no load.
V = E = 5.6 V
If
E1 = E2, I = 0
E - E2 6 - 5
I= 1
=
= 0.2 A
r1 + r2
2+3
32. Let V = Potential difference between T1 and
T2.

i
R3

H = P1t1 = P2t2
H
H
t1 = , t2 =
P1
P2

30.

i2 = i3 =

i
2

T1

B
T2

I2

i
1
P1 = i2 R1, P2 = R2 = i2 R2
2
4
1 2
P3 = i R3
4
R2 = 4 R1, R3 = 4 R1
R1 : R2 : R3 = 1 : 4 : 4
\
E
2
28. As E = kL1 k =
=
= 250
L1 500
1
V/cm
=
250
1
V = kL2 =
490 cm
250
= 1.96 V

I1

V
RA + RB
V
I2 =
RC

I1 =

Now,.

I A = I B = I1
IC = I2
Also, V = IC RC = I1( RA + RB )
= I A RA + ID RB
I B I1
RC
=
=
IC I2 RA + RB

28
33. As R1 R2

Ie =
If S2 is closed

V1 V2
V3 = V1 + V2

2W
a

R2

V1 = V2
L
R=r
A

39.
a

L2 = 2L1
R1 = R2
A2 = 2 A1

2r

b
r

a
r

2r

a
5r
3

5r
8

2r

r/ 2

r
2r
3

r
2r

d
r

2r

r
c

d
r

35. If E > 18 V current will flow from B to A and


vice-versa.

1
(For constant current)
Also, vd
A
1
vd 2 = vd1 vd1 = 2 vd 2
2
Again, vd E
\
E1 = 2 E2

If S1 is closed

>I

10 V

I
C

36. V = kl
If Jockey is shifted towards right, I and
hence k will decreases as k I.
Hence L will increase.
If E1 is increased, k will increase, hence L
will decrease.
If E2 is increased L will increase as V will
increase.
If r is closed V will decrease hence L will
decrease.
E
E
, Initially, I =
37. Ie =
Re + re
R+ r

r
2

I =6A
From b to a.
Vc - Va = 2 6 = 12 V

But
and
\

R+

>I

38. Vb - Va = - 10 + 2I = 2 V

34. As R1 = R2
R1

Ie =

V3

Also,

R
+r
2

V2

V1

29

Match the Columns


1. By applying KCL at e
i1 + i2 + i3 + i4 = 0

S R3

R1

R2

V1

V2

2W
a

1W
i1

e
i4

i3
1W

2W
d

2 - Ve 4 - Ve 6 - Ve 4 - Ve
+
+
+
=0
1
2
1
2
Ve = 4 V, I1 = - 2 A, i2 = 0 , i3 = 2 A, i4 = 0
2. Current is same at every point and A1 < A2
i
J = J1 > J 2
A
i
vd =
vd1 > vd 2
neA
R r
r=
= r1 > r2
L A
V
k = k1 > k2
L
3. When switch S is closed
V1 decreases, V2 increases,
\ Current through R1 decreases and through
R2 increases.

4. [R] =

[V] [ML2T -3A -1 ]


=
[I]
[A]

= [ML2T -3A -2 ]
[W ] [ML2T -2 ]
[V] =
=
[ q]
[AT]
= [ML2T -3A -1 ]
2 -3 -2
2
[ R][ A ] [ML T A ][L ]
[ s] =
=
[L]
[L]
= [ML3T -3A -2 ]
1
[ s ]=
= [M-1L-3T 3A 2 ]
[r ]
EA - EB
5. I =
=1A
R + rA + rA
4V 1W

1V

1W

1W

VA = EA - IrA = 3 V
VB = EB + IrB = 2 V
PA = IVA = 3 W
PB = IVB = 2 W

21

Electrostatics
Introductory Exercise 21.1

1. No, because charged body can attract an


uncharged by inducing charge on it.
2. Yes.
3. On clearing, a phonograph record becomes
charged by friction.

4. No. of electrons in 3 g mole of hydrogen atom


= 3 6.022 1023
\ q = ne = 3 6.022 1023 1.6 10-19
= 2.9 105 C

Introductory Exercise 21.2


1.

1 q1q2
1 e2
2 =

4pe0 r
4pe0 r 2
Gm1m2
Fg =
r2
Fe
e2
=
Fg 4pe0 Gm1m2
Fe =

2.

3. Let us find net force on charge at A.


A

9 109 (1.6 10-19 )2

6.67 10-11 9.11 10-31 1.67 10-27


FAB =

= 2.27 1039
1 q1q2
F=

4pe0 r 2
qq
e0 = 1 2 2
4p Fr
[ e0 ] =
=

Net force on charge at A


FA = FAB cos 30 + FAC cos 30
=

3 q2
4pe0 a 2
FAB cos 30

[IT ]2
[MLT ][L]

q
C

1 q2
1 q2
2 FAC =

4pe0 a
4pe0 a 2

[ qe ]2
[ F ][ r ]2
-2

FAC

FAC cos 30

= [ M-1 L-3 T 4 I 2 ]

FAB

60

SI units of e0 = C 2 N-1m -2.


FAC sin 30

FAB sin 30

31

4. F OA = F OC

and

F OB = - F OD
A
q

B
q

q
O
q
C

q
D

Hence, net force on charge at centre is zero.


5. No. In case of induction while charge comes
closer and like charge moves further from
the source.
+ + + + +

+
+
+
+

The cause of attraction is more attractive


force due to small distance. But if
electrostatic force becomes independent of
distance, attractive force will become equal
to repulsive force, hence net force becomes
zero.
6. When the charged glass rod is brought near
the metal sphere, negative induces on the
portion of sphere near the charge, hence it
get attracted. But when the sphere touches
the rod it becomes positively charged due to
conduction and gets repelled by the rod.
7. Yes as qmin = e
Fmin =

1 e2

4pe0 r 2

8. No. Electrostatic force is independent of


presence or absence of other charges.

9. F21 = - F12 = ( - 4 i + 3 j) N.

Introductory Exercise 21.3


1. False. E =

1
q

4pe0 r 2

2. VA > VB as electric lines of force move from


higher potential to lower potential.
3. False. Positively charged particle moves in
the direction of electric field while negatively
charged particle moves opposite to the
direction of electric field.
4. False. Direction of motion can be different
from direction of force.
s
5. E = s = Ee0 = 3.0 8.85 10-12
e0
= 2.655 10-11 C/m 2
6. q1 and q3 are positively charged as lines of
force are directed away from q1 and q3. q2 is
negatively charged because electric field
lines are towards q2.
7. If a charge q is placed at A also net field at
centre will be zero.

E
q

B
q

O
q
D

q
C

Hence net field at O is same as produced by


A done but in opposite direction,02 i.e.,
1 q
E=

4pe a 2
8. Net field at the centre (O) of wire is zero. If a
small length of the wire is cut-off, net field
will be equal to the field
due to cut-off portion, i.e.,
1 dq
dE =

O
4pe0 R2
q
R
dl
1 2p R
=

4pe0
R2
q dl
=
8p 2e0 R3

32
1
q
3 r
E=
4pe0 r

9.

=-

9 109 2 10-6
(3 2 + 42 )3/ 2

(3 i + 4 j) = - 144 (3 i + 4 j) N/C

Introductory Exercise 21.4


1. Gain in KE = loss of PE
1 1
1
1
mv2 =
q1q2 -

r
2
4pe0
1 r2
1
10-4v2
2
1
1
= - 1 10-6 2 10-6 9 109
1 0.5
2

v = 360
v = 6 10 ms -1
2. W = q ( VA - VB )
-6
1 - 1 10-6
1 - 1 10

= 2 10-6

4pe0

1
4pe0
2

-3
= - 9 10 J
= -9 m J

3. Whenever work is done by electric force,


potential energy is decreased.
W = - DU
U2 = U1 - W = - 8.6 10-8 J
qq
4. No. As U = 1 2
4pe0r
If there are three particles
1 q1q2 q2q3 q3q1
U=

+
+

4pe0 r12
r23
r31
Here U may be zero.
In case of more than two particles PE of
systems may same as if they were separated
by infinite distance but not in case of two
particles.

Introductory Exercise 21.5


1. Vba =

Wa b
q

= 12 102 = 1200 V

2. l = a x
(a) SI Units of l = C/m
l
a=
x
C/ m
Hence SI unit of a =
= C/m 2.
m
L

L x dx
a

4pe0 0 x + d
L dx
a L
=
dx - d

0x + d
4pe0 0

a
=
[[ x ]L0 - d [ln ( x + d )]L0 ]
4pe0
L + d
a
=
L - d ln

4pe0
d

3. Consider an elementary portion of length dx


at a distance x fro my centre O of the rod.

x
d

(b) Consider an elementary portion of rod at


a distance x from origin having length
dx. Electric potential at P due to this
element.
dV =

l dx
1

4pe0 x + d

Net electric potential at P


L 1
l dx
V =

0 4 pe
0 x + d

d
O
2l

dx

Electric potential at P due to this element,


l dx
1
dV =

4pe0 d 2 + x 2
V=

l
l

4pe0 l

dx
d2 + x 2

33
l -1 x l
sin
d - l
4pe0
q
x
=
2 sin -1
4pe0 2l
d
q
x
sin -1
V=
4pe0 l
d
=

4. Consider the cone to be made up of large


number of elementary rings.
O
q

Total potential at O
Q sin q L
QL sin q
V=
dl =
2pe0 RL 0
2pe0 RL
Q
[L sin q = R]
=
2pe0 L

dl

Charge on the elementary ring;


Q
dQ = s dA =
2px dl
p RL
2Ql sin q
or
dQ =
dl
RL
Potential at O due to this ring
1 dQ
dV =

4pe0 l
Q sin q
=
dl
2p e0 RL

U = qV
Qq
=
2pe0 L

Consider one such ring of radius x and


thickness dl. Let q be the semi-vertical angle
of cone and R be the radius of cone.

Introductory Exercise 21.6


1. (a) V = a ( x 2 - y2 )
v ^ v ^
^
^
i=
j = - 2ax i + 2 y j
E = -

V is constant hence E is zero.


For
x >4
V is decreasing at constant rate, hence E is
positive.
(50 - 100)
dV
3. E = == 10 V/m
dr
5 -0

(b) V = axy
v ^ v ^
^
^
i=
j = - a ( y i + x j)
E = -

True.

2. From x = - 2 to x = 0 & x = 2 to x = 4
V is increasing uniformly.

4. (a) VP - VD = E l = 0

(b) VP - VC = E l = 20 1 cos 0
= 20 V

Hence, E is uniform and negative


From x = 0 to x = 2

1m
C

1m

E = 20V/m

(c) VB - VD = - 20 1 = - 20 V
(d) VC - VD = - 20 1 = - 20 V

34

Introductory Exercise 21.7

B+q

x
Q
A

2l
Q
x

F29

1. F1 = qE towards right
F2 = qE towards left
Net torque about q,
t = qE (2l - x ) sin q + qEx sin q
= q (2l) E sin q = pE sin q

t = p E

2.

E1 =

1
q

4pe0 ( y2 + a 2 )2

E2 =

1
q

4pe0 ( y2 + a 2 )2

E3 =

1 2q

4pe0 y2

-q
4pe0

2
cos q
cos q ^
- 2
2 - 2
j
2
y +a
y + a 2
y
^
2q 1
y
=
j
4pe0 y2 ( y2 + a 2 )3/ 2
2
2 3/ 2
- y3 ^
2q ( y + a )
= 2 2
j
4pe0 y ( y + a 2 )3/ 2

F1
2l

E = - ( E3 - E1 cos q - E2 cos q) j

2 3/ 2

y3 1 + q
- y3
2

^
y
2q
=j

4pe0 y2 ( y2 + a 2 )3/ 2

As y >> a

3
3 q2
- y3
y 1 +
2
2y
2q
^

E=
j
4pe0
y5

2
3 qa ^
j
E=4pe0 y4

Net field at P

Introductory Exercise 21.8


1. (a) Charge q is completely the hemisphere
hence flux through hemisphere is zero.

(b) Charge inside the sphere is q hence flux


through hemisphere
f=

q
e0

(c) As charge q is at the surface, net flux


through hemisphere
f=

q
2 e0

2. When charge is at any of the vertex, net flux


through the cube,
q
f=
8e0
If charge q is at D,
flux through three faces containing D is zero
and the flux f is divided equal among other
three faces, hence

fEFGH =
and

1
q
=
f 2pe0

fAEHD = 0

3. True. As electric field is uniform, flux


entering the cube will be equal to flux
leaving it.
q
\
fnet = 0 fnet =
e0
q =0
4. (a) As net charge inside hemisphere is zero,
E
1

f1 + f2 = 0
But E is parallel to surface 2.

35
\
f2 = 0
Hence, f1 = 0
(b) Again, f1 + f2 = 0

POQ = 2q = 120
E

f2 = E pR2 = p R2E
\
f1 = - f2 = - p R2E
R/2 1
5. cos q =
= , q = 60
R
2

2p
R
3
Charge inside sphere,
q
q
2p
q= 0
R 0
2pR 3
3
\Flux through the sphere
q
q
f=
= 0
e0 3 e0
\Length of arc PQ =

6. Net charge inside the cube = 0.


\Net flux through the cube = 0.

Q
q

Introductory Exercise 21.9


Q

1 q + q + qB

=0
4pe0
2R
'+

3q

B
2q

(q

1. VB =

q+Q
) q' +
+Q
(q +Q)
q+Q

A +2q
q
q

C
B

2R

3R

2R

VB =
q B = - 2q
Total charge inside a conducting sphere
appears on its outer surface,
\Charge on outer surface of A = 2q
and charge on outer surface of B
= 2q - 2q = 0
2. Let q = charge on sphere B and charge f
flows from sphere C to A.

1 q + q + Q 2q - Q

+
=0
4pe0
2R
3R

(i)

3 q + q + f = 0
Again, VP = VC
2q - Q
1 q + Q
1 3 q + q
q

+
+

=
4pe0 R
2R
3 R 4pe0
3R
6( q + Q ) + 3 q + 2(2q - Q ) = 2(3 q + q )
4q + 4q + q = 0
On solving

36
5
24
Q=q, q = q
11
11
A

Charge
on 0
inner surface

- (q + Q )
6
=q
11

3.

2q + q'
C

(q')

q'

- (q + q + Q )
18
=
q
11

2q

B
A + 2q
q
A

Charge
on q + Q = 6 q + q + Q 3q + q = 9 q
outer surface
18
11
11
=q
11
A
Charge
on - q
inner surface

B
- 2q

Charge
on + 2q
outer surface

4q
3

C
4
+ q
3
2
+ q
3

AIEEE Corner
Subjective Questions (Level-1)
1. F =

1 q (Q - q)

4pe0
r2

4.

For maximum force


dF
1 Q - 2q
=

=0
dq 4pe0
r2
Q
q=
2
1 -2
d 2F
=

<0
dq2 4pe0 r 2
Q
Hence F is maximum at q = .
2
2. Minimum possible charge on a particle = e.
9
-19 2
1 e 2 9 10 (1.6 10 )
\Fmin =
2=
-2 2
4pe0 r
(1 10 )
3.

= 2.3 10-24 N
1 q1q2
Fe =

4pe0 r 2
Gm1m2
Fg =
r2
q1q2
Fe
=
Fg 4pe0 Gm1m2
=

(3.2 10-19 )2 9 109


6.67 10-11 (6.64 10-27 )2

= 3.1 1035

(i)
(ii)

F1 =

1 q1q2

4pe0 r 2

F2 =

1 q2

4pe0 r 2

(i)
(ii)

[As both the spheres are identical, find


charge on both the spheres will be equal]
q - q2
q= 1
2
q1 - q2 = 2q

From Eq. (ii),


q2 = 4pe0 r 2F2
=

(50 10-2 )2 0.036


9 109

= 10-12

q = 10-6 C = 1 mC
From Eq. (i),
(50 10-2 )2 0.108
q1q2 = 4pe0r 2F1 =
9 109
= 3 10-12
Also, q1 + q2 = 2q = 2 10-6
On solving
q1 = 3 mC
and
q2 = + 1 mC

37
5. (a) F1 =

q1 Q
1

4pe0 (3 a / 2)2
+a/2 +a
O Q q2
F2
F1

a
a1

1 4q1 Q

4pe0 9a 2
qQ
1
F2 =
2
4pe0 ( a / 2)2
1 4q2 Q
=

4pe0 a 2
F1 =

FAO
(i)

For net force on Q to be zero.


FAB = FAO
1 4q2
1 qq Q

4pe0 L2
4pe0 L2
9

q= Q
4
4
Q= q

9
-4
As Q is negative q =
q
9
(b) PE of the system
1 4q2 qQ
4qQ
U=

+
+

4pe0 L
x
L - x

(ii)

For net force on Q to be zero


F1 = F2
or
q1 = 9 q2
1 4q1 Q
(b) F1 =

4pe0 25a 2
a
q1

+Q
+

F2

F1 =

+a
2
Q
F1 q2

1 4q2 Q

4pe0 9a 2

For net force on Q to be zero.


F1 + F2 = 0
q1 25

=
q2
9

Hence, equilibrium is unstable.


N sin 60

O
N
R

60

Fe

N cos 60

B
+4q

Let the charge Q is planed at a distance x


from A (+ Q charge)
1 qQ
FOA =

4pe0 x 2
4q Q
1
FOB =

4pe0 ( x - x )2
For net force on Q to be zero.
FOA = FOB
4q Q
1 qQ
1
2 =

4pe0 x
4pe0 ( L - x )2
( L - x )2 = (2x )2
L
x=
3
Force on A,

1 4q2 4qQ 8qQ

=0
4pe0 L
3L
3 L

7. FBD of af placed at left can be given by

6. (a) In order to make net force on charge at A


and B zero, Q must have negative sign.
A
+q

1 4q2

4pe0 L2
1 qQ
=

4pe0 x 2
1 qq Q
=

4pe0 L2

FAB =

mg

DABD is equilateral
As beads are in equilibrium
mg = N sin 60
Fe = N cos 60
Fe
= cot 60
mg
q2 = 4pe0 R2 mg cot 60
4pe0 R2 mg
3
6pe0mg
= 2R
3

q=

38
Here, dE sin f components of field will cancel
each other.
Hence, Net field at O
1
l p/ 2
E = dE cos f =

cos f df
4pe0 R - p/ 2
1 2l
=

4pe0 R

8. As ball are in equilibrium


O
a
T

T cosa

13. Consider elementary portion of the rod of


length dl at a distance l from the centre O of
the rod.

a
Fe

Fe

T sina

dE sin f

r
mg

dl

Fe = T sin a
mg = T cos a
Fe = mg tan a
q2 = 4pe0r 2 tan a
r = 2 l sin a
2
q = 16pe0 l2 sin 2 a tan a
q = 3.3 10-8 C.

mg

Here,

dl

9 10 ( - 8.0 10 )
((1.2)2 + (1.6)2 )3/ 2
^

\
^

(1.2 i - 1.6 j )

12. Consider an elementary portion on the ring


of length dl subtending angle df at centre O
of the ring.
Charge on this portion,

dE sin f

dE cos f
dE cos f
dE
O

df

dE sin f
f
R

dl

dE cos f

dl

dq = l dl = l Rdf
1 dq
1 l df
2 =
dE =
4pe0 R
4pe0 R

dE sin f

Q
dl
L
1
dq
dE =

4pe0 ( a sec f)2


Q dl
1
=

4pe0 La 2 sec 2 f
dq = l dl =

= - 18 2 (1.2 i - 1.6 j ) N/C.

dE

dE cos f
P

Charge on this portion

10. See Q.7. Introductory Exercise 21.3.


q
1
11.
E=
1
r
4pe0 r 3
-9

f
Q

dE

9. Same as Q.7. Introductory Exercise 21.3.

dE

df

Now,

l = a tan f
dl = a sec 2 f df
1 Q df
dE =

4pe0 La

Net Electric field at P.


E = dE cos f
[ dE sin f components will cancel each other
as rod in symmetrical about P.]
1
Q q
=

cos f df
4pe0 La - q
1 2Q sin q
=

La
4pe0
L
L
But sin q =
=
2
2
4a + L2
L
2 a 2 +
2
2Q
1
E=

\
4pe0 a 4a 2 + L2

39
14. (a) As shown in figure, direction of electric
field at P will be along + ve y-axis.

Clearly resultant field is along angle bisector


of field towards 9 and 10.

6E1

6E1

6E1

E1

E2

E
6E1

6E1
6E1

Hence time shown by clock in the direction


of electric field is 9 : 30.
F - eE
16. (a) a =
=
m
m
- 1.6 10-19 1 103
=
9.1 10-31

(b) Positive x-axis.


y
E2

+Q

E1

+Q

(c) Positive y-axis.


E
E1

E2

+Q

1
q

4pe0 R2
11
E11

u
12
E12

10
E10

1
E1
E2

E3 3
E4

E8
7

E7 E
6
6

9 E9
8

(b) v = u + at
5 106
t=
= 2.8 10-8 = 28 ns.
1.76 1014
(c) Dk = work done by electric field.
= F x = - eEx
= - 1.6 10-19 1 103 8 10-3
= - 1.28 10-18 J
Loss of KE = 1.28 10-18 J
25
ms -1
17. Here, ux = u cos 45 =
2

15. Let E1 =

= - 1.76 1014 ms -2
u = 5.00 108 cm/s = 5 106 ms -1
v=0
v2 - u 2 = 2as
(5 106 )2
s=
= 1.4 10-2 = 1.4 cm
2 1.7 1014

E5
5

Resultant fields of two opposite charges can


be shown as given in figure.

u y = u sin 45 =

25
ms -1
2

a x = qE = 2 10-6 2 107
= 40 ms -1
a y = - 10 ms -1
1 2
y = u yt +
t
ay
y=

25
t - 5t 2
2

40
at the end of motion,
t = T and y = 0
5
s
T=
\
2

Case I.
In between two charges : let potential is zero
at a distance x from q1 towards q2.
x

Also at the end of motion,


x=R
1
x = ux t + a x t 2
\
2
2
25
5
5
R=

+ 20

2
2
2
= 312.5 m
m 2 sin 2q
18. (a)
R=
qE
qER
sin 2q =
mu 2
1.6 10-19 720 1.27 10-3
=
1.67 10-27 (9.55 103 )2
= 0.96
2q = 88 or 92
q = 44 or 46
2mh sin q
T=
2E
1
1.67 10-31
2
1.6 10-19 720

= 1.95 10-11 s

19. (a) a = -

V=
=

q2

q2
1 q1
1

=0
4pe0 x
4pe0 100 - x
1 2 10

x
4pe0

-6

-6
1 3 10

=0
4pe0 100 - x

200 - 2x = 3 x
x = 20 cm

Case II.
Consider the potential is zero at a distance x
from charge q, on its left.
x

100 cm

q1

\ V=
=

2 9.55 103

100x

q1

q2

q2
1 q1
1

=0
4pe0 x
4pe0 100 + x
1 2 10

x
4pe0

-6

-6
1 - 3 10

=0
4pe0 100 + x

200 + 2x = 3 x
x = 200 cm
21. Let us first find the potential at a point on
the perpendicular bisector of a line charge.
Consider a line of carrying a line charge
density l having length L.

-19

1.6 10
120 ^
eE
=j
m
9.1 10-31

dl

= - 2.1 1013 i m/s


-2
Dx 2 10
4
(b) t =
=
= 10-7 s
5
ux 1.5 10
3

vy = u y + a y t
4
= 3.0 106 2.1 1013 10-7
3
= 0.2 106 m/s

v = (1.5 105 ) i + (0.2 106 ) j

20. Absolute potential can be zero at two points


on the x-axis. One in between the charges
and other on the left of charge a1 (smaller in
magnitude).

O
q1

100cm
q2

Consider an elementary portion of length dl


on the rod.
Charge on this portion
dq = l dl
l dl
1
dV =

\
4pe0 r sec f
Now,

l = r tan f
dl = r sec 2 f df

41
\
\

l sec f df
4pe0
q
l
V = dV =
sec 2 f df
4pe0 q
l
=
[ln|sec q + tan q|]q- q
4pe0

dV =

l sec q + tan q

=
ln
4pe0 sec q - tan q
2l
ln|sec q + tan q|
=
4pe0
In the given condition
q = 60
Potential due to one side
2l
V1 = V2 = V3 =
ln|sec 60 + tan 60|
4pe0
2l
=
ln|2 + 3 |
4pe0

Hence, particle at B is faster than that at A.


24. Centre of circle is equidistant from every
point on its periphery,
1
q
Hence,
,
V0 =
4pe0 R
where q = Q1 + Q2 = - SQ
1 5Q
\

V0 = 4pe0 R
1
q
Similarly,
Vp =

2
4pe0
R + Z2
=-

4pe0

SQ
2

R + Z2

25. Initial PE
1 q1q2

4pe0 r1
1 q1q2
Uf =

4pe0 r2
Ui =

Work done by electric force


W = - DU = - (Uf - Ui )
O

Total potential at O
V = 3 V1 =
=

6l
ln|2 + 3 |
4pe0

Q
ln|2 + 3 |
2pe0 a

22. (a) V2 - V1 = - E d = - 250 20 10-2


= - 50 V
W = DV = q( V2 - V1 )
= 12 10-6 - 50 = - 0.6 mJ
(b) V2 - V1 = - 50 V
23. By work energy theorem
W =DK
1
1
q( V1 - V2 ) = mv22 - mv12
2
2
- 5 10-6 (20 - 800)
1
= 2 10-4( V22 - (5)2 )
2
v22 = 55
v2 = 55 = 7.42 ms -1
When a particle is released in electric field it
moves in such a way that, it decreases its PE
and increases KE

1 1
1
q1q2 -

r
4pe0
2 r1
W = - 9 109 2.4 10-6 ( - 4.3 10-6 )
1
1

0.25 2 0.15
=-

W = - 0.356 mJ
1 q1q2 q2q3 q3q1
26. (a) U =

+
+

4pe0 r12
r23
r31
4 10-9 ( - 3 10-9 )
= 9 109
0.2

( - 3 10-9 ) (2 10-9 )
+
0.1
+

4 10-9 2 10-9

0.1

U = 9 10-8 [ - 6 - 6 + 8] = - 360 nJ
(b) Let the distance of q3 from q1 is x cm. Then
qq
qq
1 q1q2
U=

+ 2 3 + 3 1 =0
4pe0 0.2
0.2 - x
x
4 10-9 ( - 3 10-9 )
9 109
20 100- 2

( - 3 10-9 ) 2 10-9
+
(20 - x ) 10-2

42
+

2 10

-9

-9

4 10
=0

6
6
8
+ =0
10 20 - x x

x = 6.43 cm

V = - (5 i - 3 j) - (5 k ) = 0
^

(b) r = 4 i + 3 k
^

V = - (5 i - 3 j) - (4 i + 3 j)
= - 20 kV

29. E = 400 j V/m

(a) r = 20 j cm = (0.2 j) m

V = - E r = - 80 V

(b) r = ( - 0.3 j) m

V = - E r = 120 V

V =0
^

30. E = 20 i N/C

(a) r = (4 i + 2 j ) m

= 4.07 105 V-m.


q
(b) f = q = e0f
e0

V = - E r = - 80 V

(b) r = ( 2 i + 3 j ) m

= 8.85 10-12 780 = 6.903 10-9


q = 6.903 nC

V = - E r = - 40 V
31. (a) [ A ] =

[ ML2 T -3 I-1 ]
[V ]
=
[ xy + yz + zx ]
[L2 ]

(c) No.
Net flux through a closed surface does
not depend on position of charge.

= [ ML0 T -3 I-1 ]
v ^ v ^ v ^

(b) E = - V = -
i+
j+
k
y
z
x
^

4
3
^
^
36. E = E0 i + E0 j
5
5

S = 0.2 j m 2 =

= - A [( y + z ) i + ( z + x ) j + ( x + y) k ]
(c) at (1m, 1m, 1m)

V - 0 = - (40 + 60)
V = - 100

q = e0f = 8.8 10-12 360


= 3.18 10-9 C
= 3.186 nC
3.60 10-6
q
36. (a) f =
=e0
8.85 10-12

(c) r = (0.15 k )

v
= - ( Ay - 2 Bx )
x
V
Ey = - ( Ax + C )
y
V
Ez = =0
Z
(b) For E = 0
Ex = 0 and Ey = 0
Hence,
Ey = 0
Ax + C = 0
C
x=A
Ex = 0
C
Ay - 2 B - = 0
A
2 BC
y=A2
2 BC
C
Hence, E is zero at - , .
A2
A
q
34. f =
e0

33. (a) Ex = -

(a) r = 5 k

= - 20 ( i + j + k )

28. V = - E r

32. VB - V0 = - E r

27. Let Q be the third charge


1 q2 qQ qQ
U=

+
+
=0
4pe0 d
d
d
q
Q=2

E = - 10 ( 2 i + 2 j + 2 k )

x 10-2

f = E S =

1^ 2
jm
5

4
Nm 2/C
25

43
4
2.0 103 N-m 2/C
25
= 320 N-m 2/C

1
p
a=
2
3
R = b tan a = 3 b

cos a =

37. E =

E0 x ^
i x1 = 0
l

E1 = 0
x2 = a

E a^
E2 = 0 i
l
Flux entering the surface
f1 = 0
Flux leaving the surface
E a3
f2 = E2a 2 = 0
l
5 103 (1 10-2 )3
=
2 10-2
= 0.25 N-m 2/C
q
Net flux, f =
e0
q
f2 - f1 =
e0

Hence proved

^
^
^
^
^
39. E = - B i + C j - D k, S1 = - L2 i, S2 = - L2 j,

^
^
^
S4 = - L2 k,
S4 = - L2 j,
S3 = - L2 i,

^
S6 = - L2 k
\

f1 = E S1 = BL2, f2 = E S2 = CL2,

f3 = E S3 = - BL2, f4 = E S4 = - CL2,

f5 = E S5 = - DL2, f6 = E S6 = DL2

(b) f = f1 + f2 + f3 + f4 + f5 + f6 = 0
40. W = 2p (1 - cos f)

q = e0 ( f2 - f1 )
= 8.85 10-12 0.25
= 2.21 10-12 C = 2.21 pC
38. Consider the charge is placed at vertex of the
cone of height b and radius R.

R
a
Q

Let a be the semi-vertical angle of the cone,


then solid angle subtended by the cone.
W = 2p (1 - cos a )
Flux passing through cone
W
f=
ftotal
4p
1
But
(Given)
f = ftotal
4
W=p
2p (1 - cos a ) = p
1
1 - cos a =
2

f1 = f2 =

2p (1 - cos a ) q
W

cot ftotal =
4p
4p
e0
1
=
(1 - cos a )
2e0

Total flux through the ring


f = f1 + f2
q
q
= (1 - cos a ) =
1e0
e0

2
2
R +l
l

41. From the given equation,


y

a
q
O

45

radius of hemisphere = a
and its centre is at ( a, 0, 0)

44
q A + q B + qC

b
c

s a2

=
- b + c

e0 b

1 q A + q B + qC
VC =

c
4pe0

1
VB =
4pe0

W = 2p (1 - cos a ) = 2p 1
2

2p 1
W
2 q
f=
ftotal =

4p
4p
e0
q
1
f=
1
2 e0
2

42. q1 = s (4pr ), q2 = s (4p R )


q2

s
e0

a 2 - b2 + c 2

44. (a) As charge Q is placed at the centre of the


sphere, charge - Q will appear on the
inner surface and Q on its outer surface.

q1

+Q

R
Q

Q
4 p ( r + R2 )
1 q1 q2
VA =
+

4pe0 r
R

But, q1 + q2 = Q s =

2
2
1 s (4pr ) s (4p R )

r
R
4pe0

1 Q ( r + R)

4pe0 r 2 + R2
1 q1 + q2
1 Q
VB =

=
4pe0 R 4pe0 R
=

Hence,

sin =

and

sout

-Q

4pa 2
Q
=
4pa 2

(b) Entire charge inside the sphere appears


on its outer surface, hence
sin = -

Q+q
Q
and sout =
4pa 2
4pa 2

(c) In case (a)


E=

43. q A = s (4pa ), q B = - s (4pb )

1 Q

4pe0 x 2

+Q

s
s

x
b

A
B
C

qC = s (4pc 2 )
1 q A q B qC
VA =
+
+

4pe0 a
b
c
s
= ( a + b + c)
e0

In case (b)
E = E1 + E2
E1 = Field due to charge Q.
E2 = Field due to charge on shell.
1 Q
E=

4pe0 x 2
for x < a
As field due to shell is zero for x < a.

45
and

E=

1 Q+q
, for x > a

4pe0 x 2

(d) Let Q A be the charge on inner sphere.


Q

45. Let Q be the charge on the shell B,


b
q
c
q

QA

b
q q
c
b
q
c

R
C

3R

B
a

1 q + Q - q
+

=0
c
4pe0 b
b - c
Q = q

VB =

Charge distribution on different surfaces is


shown in figure.
46. (a) Let E1 and E2 be the electric field at P due
to inner shell and outer shell respectively.

P r

47. (a) At r = R
1 Q - 2Q 3Q
V=
+
+

4pe0 R
2R
3 R
1 Q
=

4pe0 R
At r = 3 R
V=

+2Q
B

R
3R

1 QA
Q
VA =

=0
4pe0 R 3 R
Q
QA =
3

A
Q

1 Q - 2Q + 3Q

4pe0
3R

1 2Q
=

4pe9 3 R

(b) Let E1, E 2 and E 3 be the electric fields at


5
r = R due to shells A, B and C
2
respectively.
C

1 2Q
and E2 = 0
Now, E1 =

4pe0 r
1 2Q
\
E = E1 + E2 = E1 =

4pe0 r
1 2Q
Q
(b) VA =

4pe0 R 3 R
1 2Q - Q
VB =

4pe0 3 R
1 2Q 2Q
VA - VB =

4pe0 R 3 R
1 4Q
=

4pe0 3 R

(c) Whenever two concentric conducting


spheres are joined by a conducting wire
entire charge flows to the outer sphere.
\

Q A = 0, Q B = 0

3R
A
R
2R

2Q 3Q
Q

1
Q

4pe0 5 2
R
2
1
4Q
=

4pe0 25 R
1
2Q
E2 =

4pe0 5 2
R
2
E1 =

(outwards)

46
1
8Q
=

4pe0 25 R

(inward)

E3 = 0
5
Net field at r = R
2
E = E2 + E1 =

1
4Q

4pe0 25 R

(inward)

(c) Total electrostatic energy of system is the


sum of self-energy of three shell and the
energy of all possible pairs i.e.,
2
1 Q 2 ( - 2Q )
(3Q )2
U=

+
+
4pe0 2 R 2 2 R 2 3 R

Q ( - 2Q ) ( - 2Q ) 3Q Q 3Q
+
+

2R
3R
3R
1 Q
U=

4pe0 R
+

(d) Let q charge flows from innermost shell


to outermost shell on connecting them
with a conducting wire.
3Q +q
2Q
2R
Q2

3R

1 Q - q - 2Q 3Q + q

+
+

4pe0 R
2R
3R
1 3Q - 2q
=

4pe0
3R

VA =

1 Q - q - 2Q + 3Q + q
VB =

4pe0
3R

1 2Q
=

4pe0 3 R
But VA = VB
1 3Q - 2q
1 2Q

4pe0
3R
4pe0 3 R
Q

q=
2
\Charge on innermost shell = Q - q =

Q
2

and charge on outermost shell= 3Q + q =

7Q
2

1 3Q - 2q

4pe0
3R
1 2Q
=

4pe0 3 R

and

VA =

(c) In this case


E1 =

4pe0

Q
2

5
2 R R
2
2Q
1
(outward)
=

4pe0 25 R
1
2Q
E2 =

4pe0 5 2
R
2
1
8Q
(inward)
=

4pe0 25 R
5
Net electric field at r = R
2
1
6Q
(inward)
E = E2 - E1 =

4pe0 25 R

Objective Questions (Level-1)


1. f = E A
Units of f = N/C m 2 = N - m 2/C
or V/m m 2 = V - m
2. Net force
F = mg - qE
qE
g = g m
l
T = 2p
>T
g1

3. Electric lines of force terminate at negative


charge.
1 q2
4. F =

4pe0 l2
Initial PE
Ui =
Find PE

1 q2 q2 q2
= 3 Fl

+
+
4pe0 l
l
l

47
Uf =

1 q2 q2 q2 3
= Fl

+
+
4pe0 2l 2l 2l 2
3
W = Uf - Ui = - Fl
2

W = Uf - Ui = =

5. KE = qV

V1 : V2 : V3 =

q2
[4 - 2 2 ] J
4pe0a

8. Potential at point P

1
mv2 = qV
2
2qV
v=
m
q1V1
q2V2
q3V3
V1 : V2 : V3 =
:
:
m1
m2
m3

2R
P

e 1
e 2
2e 4
:
:
m
2m
4m

3R

1 q1 + q2

4pe0
3R

V=

V1 : V : V3 = 1 : 1 : 2
6. V =

2 q2
q2
+
[4 - 2 ]
4pe0 a 4pe0 a

1 q

4pe0 r

9 109 3 10-6
= 9000
3R
R =1m

9. As distance of every point of ring from axis is


same.
kq
, But x = 2 R
V=
R2 + x 2

4r

=
V =
1
7. Ui =
4pe0

1 2q V

=
4pe0 4r 2

10. For equilibrium,


w

q2

q2
4 +
2
2a
a

+q

r g
m

+q

A
q

+q

+q

- q2
[4 - 2 ]
4pe0 a
2

1 q
q
q
2 +
2 2
4pe0 a
a
2

2 q2
=4pe0 a

Uf =

kq
3R

30

q
sin

mg

Fe

mg cos q

mg sin 30 = Fe
1 q2
mg sin 30 =

4pe0 r 2
r=q

1
4pe0 mg sin 30

= 2.0 10-6

91 109
0.1 10

20 cm

1
2

48
11. Net force on C = 0

13. Data is not sufficient.

14. If the charges have opposite sign, electric


field is zero on the left of smaller charge.
15. Net field is only due to charge on C.
Aq

FCO

FCD

qC

FCA

FCB
E

(2 2 - 1) Q 2

FCB =

4pe0

FCD =

2 2
1 (2 2 - 1) Q

4pe0
a2
2

FCA =

1 (2 2 - 1) Q

4pe0
2a 2

FCO =

2
1 2 (2 2 - 1) Q

4pe0
a2

17. Charge distribution is shown in figure.


+8Q
2Q
+2Q
4Q
+4Q

(2 2 - 1)Q (2 2 - 1)Q (2 2 - 1)Q


+

2
a2
2

(2 2 - 1)Q 2q
+
+
=0
1
2

7Q
4
s
12. E =
e0

16. On touching two spheres, equal charge will


appear on both the spheres and for a given
total charge, force between two spheres is
maximum if charges on them are equal.

Net force on C
F = FCA + FCO + FCB cos 45 + FCO cos 45
=

1
q
q
E=

=
2
4pe0 (2a )
16pe0 a 2

a2

qB

q=-

F = eF =

18. V =
es
e0

Acceleration of proton
se
F
a=
=
m me0
1
s = ut + t 2
a
u =0
t=

25 me0
25
=
a
se
2 0.1 1.67 10-27 8.8 10-12

= 2 2 ms

2.21 10-9 1.6 10-19

1 q

4pe0 r

If drops coalesce, total volume remains


conserved,
4
4
pR3 = 1000 pr 3
3
3
R = 10r
1 1000q
V =

= 10V
4pe0 10q
19. VA =

1 q Q

+
4pe0 r
R

q
A
R

49
1 q + Q

4pe0 R
q 1 1
VA - VB =

4pe0 r R
VB =

O
R

VA - VB q
\
If q is doubled, VA - VB will become double.
20. Charge distribution is shown in figure.

1
q
1 1 3q

4pe0 R + r 2 4pe0 2 R
1
3
=
R + r 4R

2Q
3Q
3Q
Q
Q

4R = 3 R + 3r
R
r=
3
25. Net charge on any dipole is zero.
26. For net force to be zero.

^
^
^
21. f = E S = (5 i + 2 j) ( i ) = 5 V-m.
22. FDA = FDC =

1 Qq

4pe0 a 2

T cos q

q
qE

A Q

7 sin q

B
mg

mg
cos q
qE
or T sin q = qE T =
sin q
V1
1
q
27.
E1 =

=
4pe0 a
a
V2
1 q
E2 =
=
b
4pe0 b2
T cos q = mg T =

FDC

Q
C

FDB
FDA

FDB =

1
q2
2
4pe0 2a

Net force on charge at D

F0 = FDB + FDA cos 45 + FDB cos 45 = 0

1
q
4pe0 2
4
a

Q
Q
q
+
2 +
=0
2
2

q = -2 2Q

23. As VB = 0, Total charge inside B must be zero


and hence charge on its outer surface is zero
and on its inner surface is - q.
1
24. Vp =
V0

But

E1 = E2
V1 V2
=
a
b
V1 a
=
V2 b

28. Electric field on equatorial lines of dipole is


opposite to dipole moment.
29. Potential difference between two concentric
spheres is independent of charge on outer
sphere.
q
1
30. E =
2
4pe0 r

50
1 q
V=
= Er
4pe0 r
V
3000
r = =
=6m
E 500
6 ( - 3000)
q = 4pe0rV =
= - 2 mC
9 109
31. F1 = F2

qq
1 q1q2
1

=
1 2
4pe0 r12
4p Ke0 r22
r
50
r2 = 1 =
= 10 5 m
K
5

1
q VR
E=

= 2
4pe0 r 2
r
36. When outer sphere is earthed field between
the region of two spheres in non-zero and is
zero in all other regions.

37. W = F s = qEs cos q


W
4
E=
=
= 20 N/C
qs cos q 0.2 2 cos 60
38. V1 =

1 Q

4pe0 R

22.3 m

d 2 + R2
Q

32. Electric field at a distance r from infinite line


charge
l
E=
2pe0r

dV = - E dr
V2

l
1
ln
2pe0
2

W = q( V2 - V1 ) =

ql
1
ln
2pe0
2

34. r = (4 - 1)2 + (2 - 2)2 + (0 - 4)2 = 5 m


V=

1 q 9 10 2 10
=
4pe0 r
5

1 Q
Q

+
2
2
R

.
d
+
R
4pe0

1 2Q
2Q

V1 - V2 =
2
2
R
.
d
+
R
4pe0

Q 1
1

V1 - V2 =
+
2
R
.
d + R2
4pe
V2 =

33. As negative charge is at less distance from


the line charge, it is attracted towards the
line charge.
9

V2 - V1 =

dV = - E dr

-8

= 36 V

(b) and (c) are wrong.


1
q
35. V =

4pe0 R

39. Electric field inside a hollow sphere is


always zero.

40. W = F r = q E r
^

= q ( E1 i + E2 j) - ( a i + b j)
= q ( aE1 + bE2 )

At a distance r from the centre,

JEE Corner
Assertion and Reason
1. Negative charge always moved towards
increasing potential.
On moving from A to B potential energy of
negative charge decreases hence its KE
increases.
1 q1q2
2. U =

4pe0 r

If q1 and q2 have opposite sign, U decreases


with decrease in r.
dU
F= work done by conservative force
dr
always decreases PE.
dV
3. E = = - (10) = 10 V/m along x-axis.
dr
1
q
4. V =

4pe0 R

51
Inside the solid sphere.
1
qr
E=

4pe0 R3
R
at r =
2
1
q
V
E=

=
4pe0 2 R2 2 R

VA = - (4 i + 4 j) (4 i ) = - 16 V
VB = - (4 i + 4 j) (4 i ) = - 16 V
VA = VB
Hence, Assertion is false.
7. In the line going A and B, the energy of third
charge is minimum at centre.

Assertion is correct.
Reason is false as electric field inside the
sphere is directly proportional to distance
from centre but not outside it.
5. Gauss theorem is valid only for closed
surface but electric flux can be obtained for
any surface.
6. Let V0 = Potential at origin,

8. Dipole has both negative and positive


charges hence work done is not positive.
9. Charge outside a closed surface can produce
electric field but cannot produce flux.
1
qx
a
is maximum at x =
10. E =
2
2 3/ 2
4pe0 ( x + a )
2
1
q
is maximum at x = 0.
But V =

4pe0 a 2 + x 2

Objective Questions (Level-2)


1. Electrostatic force always acts along the line
joining the two charges, hence net torque on
charge + 2q is always zero.
U

30
+Q

+2q

As net torque is zero angular momentum of


charge remains conserved.
Initial angular momentum
Li = m ( V sin 30 ) R
When the separation between the charges
become minimum, direction of motion of
charge + 2q become perpendicular to the line
joining the charges.
V'

+2q

^
^
^
2. v1 = v j, v 2 = 2v cos 30 i + 2v sin 30 j
^

As velocity along y-axis is unchanged,


electric field along x-axis is zero.
For motion along x-axis,
vx2 - ux2 = 2a x ( x - x0 )
( 3v)2 - 0 3v2
=
2a
2a
3 mv2
Fx = ma x =
2a
3 mv2 ^
F =
i
2a

F =-eE

ax =

Also,

3 mv2 ^
E =i
2ea
Rate of work done by electric field at B
3 mv ^
^
^
P = F v =
i ( 3v i + v j)
2
a

+1q

\find angular momentum


Lf = mv r =

mvr
2

By conservation of angular momentum


3
Li = Lf r =
R
2

= 3 i+vj

3 3 mv3
2a

3. Electric field is always possible, hence a


must be positive and b must be negative.
+q
a

Q
b

52
4. The system can be assumed as a
combination of three identical dipoles as
shown in figure.
Here, P1 = P2 = P3 = Q (2a )

Uf =

1
q
q

=
4pe0 2 3 a 24pe0 a

Heat produced = Ui = Uf =

P2

q2
8pe0a

kq2
2a

7. Let Q charge flows to C

P1

60
60

4a

Q
q

P3

Net dipole moment of the system


P = P1 + P2 cos 60 + P3 cos 60
= 2 p = 4 Qa
Electric field on equatorial lines of short
dipole is given by
P
1
E=

4pe0 x 3
Qa
1 4 Qa
=

=
4pe0 x 3
pe0x 3
5. Potential at centre will be same as potential
at the surface of inner shell i.e., 10 V.
6. Initial charge distribution is shown in figure,
Initial energy of system
+q

q
+q
a

2q

2
1 q2 ( - q)
q2

+
+
4pe0 2a 2 2a 2 3 a

q ( - q) q ( q) q ( - q)
+
+

2a
3a
3a

5a 2
=
48pe0 a
When switch S is closed, entire charge flows
to the outer surface of outer shell,

1 q + Q ( - q)

+
=0
4pe0 3 a
4a
q
Q=4
- q
1 q
Q
\
VA =

+
+
4pe0 2a 3 a 4a
1 q
=

4pe0 s a
1
q

VA - VC =
4pe0 s a
kq
=
sa
VC =

1
q

4pe0 R

1 3q

4pe0 2 R
1
q
\ VC - VS =

4pe0 2 R
4
p dR 3 d R 2
1 3
=

=
4 pe0
2R
s E0
and

3a

2a

8. VS =

Ui =

3a

VC =

9. As particle comes to rest, force must be


repulsive, hence it is positively charged.
Again on moving down its KE first increases
than decreases, PE will first decrease than
increase.

53
10. (1) is correct as the points having zero
potential are close to Q2, |Q2| < |Q1|.
Again as potential near Q1 is positive, Q1 is
positive, hence (2) is correct.
At point A and B potential is zero not field,
hence they may or may not be equilibrium
point.
Hence (3) is wrong.
At point C potential is minimum, Q positive
charge placed at this point will have
unstable equilibrium but a negative charge
will be in stable equilibrium at this position.
Hence, (4) is wrong.
11. V1 is always negative and V2 is always
positive.
12. Electric field between the two points is
positive near q1 and negative near q2, hence
q1 is positive and q2 is negative.
Again neutral point is closer to q1, hence
q1 < q2.
13. Electric field due to a conductor does not
depend on position of charge inside it.

^
^
14. E = 400 cos 45 i + 4000 sin 45 j
^

= 200 2 ( i + j)

qE l
m

v=
At point B
T = qE +

17. Velocity of particle at any instant


V
O

(x, 0)

qE
t
m
\
L = mvr = qE x0t
Hence, angular momentum of the particle
increases with time.
V = at =

18. By work energy theorem


W = DK

1
mv2
2
1 Q
1 3Q
1
= - mv2
q

4pe R 4pe 2 R
2
0
0

Qq
u=
4pe0mR
q ( VS - VC ) = 0 -

19. Potential at the centre of negatively charged


ring



V A - V B = - E rAB

Q
^

mv2
= 2 qE
r

= - 200 2 ( i + j) (2 j - 3 i ) 10

-2

+Q
2R

= 2 2 V 2.8 V
15. Potential difference between two concentric
spherical shells does not depend on charge of
outer sphere. Hence,
V A - VB = VA - VB
But VB = 0
\
VA = VA - VB.
16. By work energy theorem,

l
q
l(1cosq)

Work done by electric field


= charge is KE
1
qE l (1 - cos q) = mv2 - 0
2

R
1

3R

1 -Q
Q
V1 =

4pe0 R
2R
Q
=8pe0 R
Potential at the centre of positively charged
ring
Q
1 Q
V2 =

4pe0 R 2 R
Q
=
8pe0 R
Kinetic energy required = Work done required
Q
= q ( V2 - V1 ) =
4pe0 R

54
20. Ex = Ey = -

Vx 2 - Vx1
x2 - x1
Vy 3 - Vy1
y3 - y1
^

=-

16 - 4
= 3 V/m
-2 -2

=-

12 - 4
= - 4 V/m
4 -2

24. T - mg cos q - qE sin q =

\E = Ex i + Ey j = (3 i - 4 j) V/m.
q

21. Consider a point P ( x, y)


where potential is zero.
Now, VP

2Q

Q
(+3a,0)

- 2Q
1
Q

+
2
2
2
2

4pe0 (3 a - x ) + y
(
)
3
x
+
a
+
y

=0
( x + 3 a )2 + y2 = 4 [(3 a - x )2 + y2 ]

3 x 2 + 3 y2 - 30ax + 27a 2 = 0

x 2 + y2 - 10ax + 9a 2 = 0
The equation represents a circle with radius
VP =

10a
2
=
- 9a = 4a
2
10
and centre at
a, 0 = (5a, 0)
2

Clearly points x = a and x = 9a lie on this


circle.
22. Work done = qEy = Charge in KE
1
Hence,
K f = mv2 + qEy
2
All other statements are correct.
23. Electrostatic force of attraction provides
necessary centripetal force.
mv2
lq
ie,
=
r
2pe0r

T=

V=

T
qE

mg

P(x,y)

(3a,0)

mv
l

lq
2pe0m

2pe0m
2pr
m
= 2pr
= 2pr
V
2 K lq
lq

Tension will be minimum when velocity is


minimum.
Minimum possible in the string is zero.
mv2
ie,
= - ( mg cos q + qE sin q)
l
Diff. both sides w.r.t. q
2mv dv
(i)
= mg sin q - qE sin q
l dq
For minima or maxima
dv
qE
= 0 q = tan -1
dq
mg
-1 qE
or
p + tan
mg
Differentiating Eq. (i) again,
2
2mv d 2v 2m dv
+
= mg sin q + qE sin q
2
l dq
l dq
d 2v
qE
\ 2 = + ve for q = tan -1
mg
dq
-1 qE
and ve for q = p + tan
mg
25. q A = s (4pa 2 ), q B = - s (4pb2 )
and qC = - s (4pc 2 )
1 q A + q B qC
VB =

b
c
4pe0

s a2
2 - b + c
e0 b

2
2
2

-q
1 q
q
q2
q2
26. Ui =

+
2
4pe0 a
a
a
a
2a

- 2 q2
=
4pe0a
=

Uf =

1 q2

4pe0 a

W = Uf - Ui =

1 q2
( 2 + 1)
4pe0 a

55
27. q A = s (4pa 2 ), q B = - s (4pb2 ), qC = s (4pc 2 )
Given, VA = VC
1 q A q B qC
1 q A + q B + qC

+
+

=
b
c 4pe0
c
4pe0 a

a 2 - b2

a-b+ c=
+c
c

a+ b=c
28. Potential at minimum at mid-point in the
region between two charges, and is always
positive.
1 q2
29. Ui =

=U
4pe0 r
Uf =

1 q2

3 = 3 U
4pe0 r

\W = Uf - Ui = 2 U
30. Loss of KE = Gain in PE
1
1 qQ
mv2 =

2
4pe0 r
1
r 2
v
31. When the spheres are in air

On dividing Eq. (ii) by Eq. (i),


Fe mg - FB
=
Fe
mg
FB
1
0.8 1
=1 =1 =
K
mg
1.6 2
K =2

1 q
q
32. VP =

2 2

4pe0 a
a 2 + b2

2
2

2q a + b - a
=

4pe0 a a 2 + b2

2 1/ 2

a 1 + b
- a

a 2
2q
2q b2
=
=
3

4pe0
a a 2 + b2
4pe0 a

[As b << a]
1 5q
33. In any case electric field at origin is

4pe0 r 2
1 5q
along x-axis and
along y-axis.

4pe0 r 2
2

34. < u > =

q
T
Tcosq
q

Tsinq

Fe

Fe
mg

mg

T cos q = mg
T sin q = Fe
(i)
Fe = mg tan q
\
When the spheres are immersed in liquid
O

1
1 1
q

e0 E2 = e0

2
2 4pe0 R2
9 109 1 10-9

1
9
= e0

2
12

e0
3
J/m
=
2

35. If Q is initial charge on B


1 Q
then, VA - VB =
=V
4pe0 b
Now, if A is earthed, let charge q moves on A
from ground, then
Q

T'
T'

T'cosq
FB

F'e
mg

F'e

Tsinq
mg

F1 = T sin q
g - FB = T cos q
Fe = ( mg - FB ) tan q

1 q Q
+ =0
4pe0 a
b
a
q=- Q
b

VA =
(ii)

56
1 q+Q
VB =

4pe0
b
1 Q
a
a
=
1 - = V 1 -
4pe0 b
b
b

v ^
36. E = -
i+
x
-2 ^
=-
i+
1
^

v ^ v ^
j+
k
y
z
-2 ^ -2 ^
j+
k
1
1
^

= 2 ( i + j + k ) N/C
If VP is potential at P, then

VP - V0 = - E r
^

VP - 10 = - 2( i + j + k ) ( i + j + k ) = - 6

42. Let charge q flows through the switch to the


ground, then
1 Q - q Q

=0
4pe0 r
2 r
1
q = Q
2
43. After n steps
-1
1
q = n Q and q = n - 1 Q
2
2
1 q
q
=0
+
\ VA =
4pe0 r 2 r
1 q + q

VB =
4pe0 2 r
=

VP = 4 V
37. On touching two spheres, charge is equally
divided among them, then due to induction a
q
charge - appears on the earthed sphere.
2

38. Negative charge will induce on the conductor


near P.

0
for r < rA

kQ P

39.
for rP < r < rB
r
k(Q A - Q B ) for r > r
B

As|Q B| > |Q A|
E is ve for r > rB.

v ^ v ^
40. E = -
i+
j
y
x
^

= k ( y i + x j)

|E| = k y2 + x 2 = kr
41. Let charge on outer shell becomes q.
B
2r

A
r

S2

S1

1
2

n+1

4pe0r

44. Consider a spherical Gaussian surface of


radius r ( < R) and concentric with the sphere,

1 Q + q
=0

4pe0 2 r
q = -Q

Charge on a small sphere of radius r


dq = d dV = 4pr 2d dr

r3
= r pd0 r 2 - dr

Total charge inside the Gaussian surface,


r
r3
q = 4p d0 r 2 - dr
0
R

r3
r4
= 4pd0

3 4 R
1
q d r
r2
2 = 0 \E =

4pe0 r
e0 3 4 R
45. Total charge inside the surface.
R3 R3 1
3
Q = 4pd0
= pd0 R
r 3
3
E=

VB =

d R3
1 Q
2 = 0 2
4pe0 r
12e0r

r
r2

3 4 R
For maximum intensity of electric field

46. E =

d0
e0

57
1 - r = 0
3 2R

r= R
3
d
d 2E
= - 0 = - ve,
2 Re0
dr 2
dE d0
=
dr e0

q1 = - Q
Now, if A is earthed
q1
q2

2
hence E is maximum at r = R.
3
2l 2 R 2


r
3 r0 R
47. Emax = 0 3 -
=
e0 3
4 R qe0

1 q2 q1

+ =0
4pe0 a
b
a
a
q2 = - q1 = Q
b
b

qA =

48. Potential difference between two concentric


spheres do not depend on the charge on
outer sphere.
49. When outer sphere B is earthed
q1
B

1 Q + q1

=0
4pe0 b

VB =

50. When connected by conducting wires, entire


charge from inner sphere flows to the outer
sphere, ie,
a
q3 = q1 + q2 - 1Q
b

a-b
=
Q
b

More than One Correct Options


1. Before earthing the surface B,
1 qA qB
VA =

+
=2V
4pe0 R 2 R
1 qA + qB 3
VB =

= V
4pe0 2 R 2
qA 1

=
qB 2
On earthing the sphere B,
qA = qA
1 qA + qB

=0
VB =
4pe0
2R

qB = - qA
qA
= -1
qB

As potential difference does not depend on


charge on outer sphere,
V
V A - VB = VA - VB =
2
1
VA = V
2

2. For the motion of particle


y

m
x

ux = 0, vx = v, a x =

qE
, a y = - g,
m

x0 = 0, y0 = 0
x = x 0 + ux t +
x=

1
ax t 2
2

qE 2
t
2m

(i)

y = y0 + u y t +
= ut -

1
a yt2
2

1 2
gt
2

At the end of motion


t = T , y = 0, x = R

(ii)

58
From Eq. (ii),

4. For all charges to be in equilibrium, force


experienced by either charge must be zero
ie., force due to other two charges must be
equal and opposite.

1
0 = u - gT T
2

2u 2 10
T=
=
=2s
g
10

q1

From Eq. (i),

q2

q3

qE 2
T
2m
1 10-3 104
=
4 = 10 m
2g

Hence all the charges must be collinear,


charges q1, and q3 must have same sign and
q2 must have opposite sign, q2 must have
maximum magnitude.
Such on equilibrium is always unstable.

Now,
v2y - u 2y = 2a y ( y - y0 )
At highest point (i.e., y = H), vy = 0
0 - (10)2 = - 2 10 ( H - 0)
H =5m

5. Flux through any closed surface depends


only on charge inside the surface but electric
field at any point on the surface depends on
charges inside as well as outside the surface.

R=

Q2

3. Let R be the radius of the sphere


1
q

V1 =
4pe0 R + r1
q 109 q

= 100
( R + S ) 10-2
1
q

V2 =
4pe0 R + r2

Q1

(i)

9 10 q

( R + 10) 10-2

= 75

(ii)

6. As net charge on an electric dipole is zero,


net flux through the sphere is zero.
But electric field at any point due to a dipole
cannot be zero.

On solving,
R = 10 cm,

7. Gausss law gives total electric field and flux


due to all charges.

5
50
10-9 C =
10-10 C
3
3
Electric field on surface,
5
9 109 10-9
1
q
3
E=

=
4pe0 R2
(10 10-2 )

8. If two concentric spheres carry equal and


opposite charges, Electric field is non-zero
only in the region between two sphere and
potential is is zero only outside both the
spheres.

q=

= 1500 V/m
Potential at surface,
1
q
V=

4pe0 R

5
10-9
3
10 10-2

9 109

= 150 V
Potential at Centre
3
VC = VS = 225 V
2

9. As force on the rod due to electric field is


towards right, force on the rod due to hinge
must be left.
The equilibrium is clearly neutral.
10. If moved along perpendicular bisector, for all
identical charges, electrostatic potential
energy is maximum at mid point and if
moved along the line joining the particles,
electrostatic potential energy is minimum at
the mid-point.

59

Match the Columns


1. (a s), (b q), (c r), (d p).
If charge at B is removed
B

at r = 2 R

E
F

Vout

11
V
8
1 q
=

4pe0 r

1
q
V

=
4pe0 2 R 2
1
qr
Ein =
3
4pe0 R

V2 =

R
2
1
q
V
V
E1 =

=
=
2
4pe0 2 R
2R 2
1
q
Eout =

4pe0 r 2

at r =
Enet = ED cos 30 + EE cos 30
= 3E
If charge at C is removed
Enet = ED cos 60 + Ef cos 60
=E
If charge at D is removed

E net = 0 and E B = - E E
and

EE = - EE

If charge at B and C both are removed,


Enet = EE + ED cos 60 + EF cos 60
= 2E
2. (a q), (b p), (c s), (d r).

V = - E r
If

^
r = 4 i, V = - 8 V,

If

^
r = -4i, V = 8V

If

^
r = 4 j, V = - 16 V,

If

^
r = - 4 j, V = 16 V

3. For a solid sphere


q
1
Vin =

(3 R2 - r 2 )
4pe0 2 R3
R
at
r=
2
q
R2
1
3 R2
V1 =

3
4pe0 2 R
4

(Q R = 1 m)

at r = 2 R
E2 =

1
q
V
V

=
=
2
4pe0 (2 R)
4R 4

\(a s), (b q), (c q), (d p).


4. (a r), (b q), (c q), (d s)
5. (a p), (b q), (c r), (d s)
For a spherical shell,
0 for r < R
E = Kq
r 2 for r R

Kq
for r R

V = R
Kq for r R
r

For a solid sphere,


Kqr
3 for r R
E = R
Kq for r R
r2

Kq
2
2
2 (2 R - r ) for r R

2
R
V =
Kq

for r R

r2

22

Capacitors
Introductory Exercise 22.1

1. C =

q
[ AT ]
= [C ] =
V
[ML2 T -3A -1 ]
-1

-2

7mC

3mC

7mC

3mC

= [M L T A ]
2. False.
Charge will flow if there is potential
difference between the conductors. It does
not depend on amount of charge present.
3. Consider the charge distribution shown in
figure.
10q

E4
E2

q
3

(q 4)
4

Hence, if q1 and q2 be charge on two plates


then.
q1

q2

q3

q4

E3
P

E1

Electric field at point P


EP = E1 + E3 - E2 - E4
10 - q
q -4
q
q
=
+
2e0 A 2e0 A 2e0 A 2e0 A
But P lies inside conductor
\ EP = 0

10 - q - q + q - q + 4 = 0

q = 7 mC
Hence, the charge distribution is shown in
figure.
Sort-cut Method
Entire charge resides on outer surface of
conductor and will be divided equally on two
outer surfaces.

q1 + q2
= 3 mC
2
q - q2
= 7 mC
q2 = 1
2
q - q1
q3 = 2
= - 7 mC
2

q1 = q4 =

q1

q2

q3

q4

61
4. Charge distribution is shown in figure.
q + q2
q
=q1 = q4 = 1
2
2
q - q2 5q
q2 = 1
=
2
2
q2 - q1
5q
q3 =
=2
2
\ Charge on capacitor = Charge on inner
side of positive plate.

5q
2
e A
C= 0
d
q 5q d
V= =
C 2e0 A
q=

and
\

Introductory Exercise 22.2


1. All the capacitors are in parallel

10V

1mF

q = CeV =

2mF

C1

C2

2
1200 = 800 mC
3

C1

C2

V1

V2

3mF
C3

q1 = C1V = 1 10 = 10 mC
q2 = C2V = 2 10 = 20 mC
q3 = C3V = 3 10 = 30 mC
2. Potential difference across the plates of
capacitor
V = 10 V
q = CV = 4 10 = 40 mC

800
q
(b) V1 =
=
= 800 V
1
C1
800
q
=
= 400 V
V2 =
2
C2
C1,V1

3. In the steady state capacitor behaves as


open circuit.
2W

4W
A

C2,V2

B
I

I
6W

30V

I=

30
=3 A
6+4

Potential difference across the capacitor,


VAB = 4 I = 4 3 = 12 V
\Charge on capacitor
q = CVAB = 2 12 = 24 mC
1
1
1
1 1
4. (a)
=
+
= +
Ce C1 C2 1 2
2
Ce = mC
3

Now, if they are connected in parallel,

C V + C2V2

Common potential, V = 1 1

C1 + C2

1 800 2 400 1600


V
=
=
1+2
3
1600
3200
q1 = C1V =
mC, q2 = C2V =
mC
3
3
5. Common potential
C V + C2V2
V= 1 2
C1 + C2
But V = 20, V2 = 0, V1 = 100 V, C1 = 100 mC
100 100 + C2 0
\
= 20
400 + C2

C2 = 400 mC

62

Introductory Exercise 22.3


1. Let q be the final charge on the capacitor,
work done by battery
C

q=

q0
(1 - e - t / t )
2

\At time t,
q0
(1 - e - t / t )
2
q
Charge on C2 = q0 - q = 0 (1 + e -t / t )
2
Charge on C1 = q =

4. Let q be the charge on capacitor at any


instant t

W = qV
Energy stored in the capacitor
1
U = qV
2
\Energy dissipated as heat
1
H = U - W = qV = U
2
2. We have
I = I0 e - t / t
I0
1
= I0 e - t / t e - t / t =
2
2
t = t ln 2 = 0.693 t
t = 0.693 time constant.
3. Let capacitor C1 is initially charged and C2 is
uncharged.
R

q0 q

+q

By Kirchhoffs voltage law


q
+ IR = E
C
dq CE - q
=
dt
RC
q
t dt
dq
q0 CE - q = 0 RC

q0 = CE (1 - e - t / t ) + q0e - t / t
where, t = RC
5. (a) When the switch is just closed,
Capacitors behave like short circuit.

C1(q0q)

I
q

C1

q
C2

At any instant, let charge on C2 be q, charge


on C1 at that instant = q0 - q
By Kirchhoffs voltage law,
( q0 - q)
q
- IR - = 0
C
C
dq q0 - 2q

=
dt
RC
q
t dt
dq

0 q0 - 2q = 0 RC

[ln ( q0 - 2q)]q0
1
=
[ t ]t0
-2
RC

R2
C2
R1

R3

\Initial current
Ii =

E
R1

(b) After a long time, i.e., in steady state, both


the capacitors behaves open circuit,
E
If =
R1 + R3
6. (a) Immediately after closing the switch,
capacitor behaves as short circuit,

63
I2

A
I1

I
E

R1

R2
C

I1 =

E
E
and I2 =
R1
R2

(c) Potential difference across the capacitors in


the steady state,
V =E
\Energy stored in the capacitor
1
U = CE2
2
(d) After the switch is open
R2

R1

(b) In the steady state, capacitor behaves as


open circuit,
E
I1 = , I2 = 0
\
R1

Re = R1 + R2
t = R3C = ( R1 + R2 ) C

AIEEE Corner
Subjective Questions (Level-1)
1. C =

-3
e0 A
Cd 1 1 10
=
A=
e0
d
8.85 10-12

= 1.13 108 m 2
e0 A1
e0 A2
2. C1 =
and C2 =
d
d
If connected in parallel
e A
e A
C = C1 + C2 = 0 1 + 2 2
d
d
e0( A1 + A2 ) e0 A
=
=
d
d
where, A = A1 + A2 = effective area.
Hence proved.
3. The arrangement can be considered as the
combination of three different capacitors as
shown in figure, where

A
2 = k3e0 A
C3 =
2d / 2
2d
k3e0

Therefore, the effective capacitance,


C2C3
C = C1 +
C2 + C3
=

e0 A k1
k1k3

2d 2
k2 + k3

4. (a) Let the spheres A and B carry charges q


and - q respectively,
q

b
d

A
C2
C1
C3

A
2 = k1e0 A
C1 =
2d
4d
A
k2e0
2 = k2e0 A
C2 =
2d / 2
2d
k1e0

1 q q

4pe0 a d
1 q q
- +
VB =
4pe0 b d
VA =

Potential difference between the spheres,


q 1 1 2
V = VA - VB =
+ 4pe0 a b d
4pe0
q
C=
=
V 1 + 1-2
a b d
Hence proved.

64
(b) If d
4pe0
4pe0 ab
=
1 1
a+b
+
a b
If two isolated spheres of radii a and b are
connected in series,
then,
C1C2
C =
C1 + C2
C=

Let effective capacitance between A and B


C AB = x
As the network is infinite,
C PQ = C AB = x
Equivalent circuit is shown in figure,
2C

RAB = C +

B
C

C
A

6. q = CV = 7.28 25 = 182 mC
-6
q 0.148 10
7. (a) V = =
= 604 V
-12
C 245 10
e A
Cd
(b) C = 0 A =
e0
d

C
C
2C

B
C
2C/3

5C/3
A

B
A

(b)
C
C

C
A

B
C

C
4C/3
A

B
C/3

(c)
2C
A
C
B

2C

8.85 10-12

2C

8. (a) E0 = 3.20 105 V/m


E = 2.50 105 V/m
3.20 105
E
k= 0 =
= 1.28
E 2.50 105
(b) Electric field between
capacitor is given by
s
E=
e0

C
Q

2C

245 10-12 0.328 10-3

= 9.08 10-3 m 2
= 90.8 cm 2
-6
q 0.148 10
(c) s =
=
= 16.3 mC/m 2
A 9.08 10-3

2Cx
=x
2C + x

2C 2 + Cx + 2Cx = 2Cx + x 2

x 2 - Cx - 2C 2 = 0
On solving, x = 2C or - C
But x cannot be negative,
Hence, x = 2C

5. (a)

C = C

\
Hence proved.

where, C1 = 4pe0a, C2 = 4pe0b


4pe0 ab
C =
\
a+b

the

plates

s = e0E = 8.8 10-12 3.20 105


= 2.832 10-6 C/m 2
= 2.832 mC/m 2

9. (a) q1 = C1V = 4 660 = 2640 mC


q2 = C2V = 6 660 = 3960 mC

of

65
As C1 and C2 are connected in parallel,
V1 = V2 = V = 660 V
C1 = 4.00mF

C2 = 6.00mF

660 V

(b) When unlike plates of capacitors are


connected to each other,
Common potential
C V - C1V1 6 660 - 4 660
V= 2 2
=
C1 + C2
6
= 220 V
q1 = C1V = 4 220 = 880 mC
q2 = C2V = 6 220 = 1320 mC
V
400
10. E =
=
= 8 104 V/m
d 5 10-3
Energy density,
1
1
u = e0E2 = 8.85 10-12 ( 8 10-4 )2
2
2
= 2.03 10-2 J/m 3
= 20.3 mJ/m 3
11. Dielectric strength = maximum possible
electric field
V
V
E= d=
d
E
5500
=
= 3.4 10-4 m
1.6 107
ke A
Cd
C= 0 A=
d
ke0
=

1.25 10-9 3.4 10-4


3.6 8.85 10-12

= 1.3 10-2 m 2
= 0.013 m 2
12. Let C P and CS be the effective capacitance of
parallel and series combination respectively.
For parallel combination,
U P = 0.19 J
1
U P = C PV 2
2
2 0.1
2U
C P = 2P =

= 0.05 F
V
(2)2
= 50 mF

For series combination,


US = 1.6 10-2 J = 0.016 J
1
US = C S V 2
2
2US 2 0.016
CS = 2 =

= 0.008 F
(2)2
V
= 8 mF
Now, C P = C1 + C2 = 5 mF
or
C2 = (5 - C1 ) mF
1
1
1
1
and
=
+
=
CS C1 C2 8
1
1
1
+
=

C1
S - C1 8
On solving,
C1 = 40 mF, C2 = 10 mF or vice-versa.
13. In the given circuit,
q
q
-E +
=5
VA - VB =
C1
C2
A

+q

C1
q

+q

C2

q
B

q
q
- 10 +
=5
10-6
2 10-6
q = 10 10-6 C = 10 mC
q
q
= 10 V, V2 =
=5V
V1 =
C1
C2

14. (a) In order to increase voltage range n


times, n-capacitors must be connected in
series.
Hence, to increase voltage range to 500V,
5 capacitors must be connected in series.
Now, effective capacitance of series
combination,
10
CS = Cn =
= 2 pF
5
Hence, no parallel grouping of such units is
required.
Hence, a series grouping of 5 such
capacitors will have effective capacitance
2 pF and can withstand 500 V.
(b) If n capacitors are connected in series and
m such units are connected in parallel,
Ve = nV
mC
Ce =
n
Here, V = 100 V
Ve = 300 V
V
n = e =3
\
V

66
C = 10 pF
Ce = 20 pF

nCe 3 20
=
=6
C
10
Hence, the required arrangement is shown
in figure.
m=

15. Case I.
V1 =

C2
V = 60 V
C1 + C2

V1

V2

100 V

V2 =

C1
V = 40 V
C1 + C2
C1 2
=
C2 3
3
C2 = C1
2

Case II.
V1 =

C2
= 10 V
C1 + C2 + 2

2mF

C1 + 2 90
=
=9
C2
10

C1 + 2 = 9C2

C1 + 2 = 9

3
C1
2

25
4
C1 = 2 C1 =
mF
2
25
= 0.16 mF
3
C2 = C1 = 0.24 mF
2
16. (a) q = CV = 10 12 = 120 mC
e A
(b) C = 0
d
If separation is doubled, capacitance will
become half. i.e.,
C
C =
2
C
q = E V = V = 60 mC
2
e0 A pe0r 2
(c) C =
=
d
d
If r is doubled, C will become four times, i.e.,
C = 4C
q = C V = 480 mC
17. Heat produced = Energy stored in the
capacitor
1
1
H = CV 2 = 450 10-6 (295)2
2
2
= 19.58 J
e A 8.85 10-12 2
18. (a) C = 0 =
d
5 10-3

= 3.54 10-6 F
= 3.54 mF
(b) q = CV = 3.54 10-9 10000
= 35.4 10-6 = 35.4 mC
V
10000
(c) E =
=
= 2 106 V/m
d 5 10-3
19. Given,
C1

A
V1

C2

C3

B
V2

100 V

V2 =

C1 + 2
= 90 V
C1 + C2 + C

C1 = 8.4 mF, C2 = 8.2 mF


C3 = 4.2 mF, V = 36 V

67
(a) Effective capacitance,
1
1
1
1
=
+
+
Ce C1 C2 C3
1
1
1
=
+
+
Ce = 2.09 mF
8.4 8.2 4.2
q = CeV = 2.09 36 = 75.2 mC
As combination is series, charge on each
capacitor is same, i.e., 75.2 mC.
1
1
(b) U = qV = 75.2 36 10-6
2
2
= 1.35 10-3 J = 1.35 mJ
(c) Common potential,
C V + C2V2 + C3V3
V= 1 1
= 10.85 V
C1 + C2 + C3
1
(d) U = (C1 + C2 + C3 )V 2
2
1
= ( 8.4 + 8.2 + 4.2) (10.85)2 10-6
2
= 1.22 10-3 J = 1.22 mJ
20. The Given circuit can be considered as the
sum of three circuits as shown
3mF
+12 12
+6
5V

6mF

+4

+2
2mF

4mF

3mF
+12 12
+

6mF

24 +12
2mF

+24

+24 12

24 4mF

10V
+
4
+

3mF
+4

5V

+8/3
8/3
8
6mF
2mF
+8

3mF
+20mC 20mC
6mF

4
+4

4mF

5V

10mC
5V

4
4

+50
mC
3

2mF 50
mC
+10mC
3
10V

+40
mC
3
4mF
40
mC
3

(Charge is shown in mC).


Hence, charge on 6 mF capacitor = 10 mC
40
and Charge on 4 mF capacitor =
mC
3
21. (a)

C1=8.4mF

C3=4.2mF

a
C2=4.2mF
b
C5=8.4mF

C4=4.2mF
8.4mF

a
4.2mF

2.1mF

b
8.4mF
8.4mF
a

2.52mF
a

6.3mF
b
8.4mF

(b) Charge supplied by the source of emf


q = CV = 2.52 10-6 220
= 554.4 mC
q1 = q5 = q = 554.4 mC
4.2
q2 =
q
4.2 + 2.1
4.2
=
554.4 mC = 369.6 mC
6.3
2.1
2.1
and q3 = q4 =
q=
554.4 mC
4.2 + 2.1
6.3
= 184.8 mC
q1 554.4
V1 =
=
= 66 V = V5
C1
8.4
q
369.6
V2 = 2 =
= 88 V
C2
4.2
q
184.8
V3 = V4 = 3 =
= 44 V
C3
4.2
22. Let C1 and C2 be the capacitances of A and B
respectively.
ke A
ke A
C1 = 1 0 1 , C2 = 1 0 2
\
d1
d2
C2
Now,
V1 =
V
C1 + C2
C2
130 13
(i)
=
=

C1 + C2 230 23

68

C1
V2 =
V
C1 + C2
C1
10
=
C1 + C2 23

C2=2mF
4mF

(ii)

From Eqs. (i) and (ii),


C1 10
=
C2 13
If dielectric slab of C1 is replaced by one for
which k = 5 then,
5e A
5
C1 = 0 1 = C1
d1
2
V2 C1 5C1 50
\
=
=
=
V1 C2 2C2 26
50
V2 +
V1 = 230
26
Also,
V1 + V2 = 230
50
V1 =
V1
26
V1 = 78.68 V
and
V2 = 151.32 V
23. In this case

2mF
20V

C3=4mF

+
3mF

C1=3mF

2mF
2mF

2mF
20V
3mF

3mF

6mF
20V

20V

3mF

6mF
C1,V1

C2

C3

Common potential,
C1V1
V=
C2C3
C1 +
C2 + C3
1 110
V=
1 + 1.2
110
=
2.2
= 50 V
Charge flown through connecting wires,
C2C3
=
V
C2 + C3
= 1.2 50
= 60 mC
24. (a) Hence, effective capacitance across the
battery is 3 mF.

(b) q = CV = 3 20 = 60 mC
(c) Potential difference across C1
6
V1 =
20 = 10 V
6+6
q1 = C1V1 = 3 10 = 30 mC
(d) Potential difference across C2
6
V2 =
20 = 10 V
6+6
q2 = C2V2 = 2 10 = 20 mC
(e) Potential difference across C3
4
V3 =
V2 = 5 V
4+4
q3 = C3V3 = 4 2 = 20 mC
25. (a) When switch S2 is open, C1 and C3 are in
series, C2 and C4 are in series their
effective capacitances are in parallel with
each other.
Hence,
C1C3
q1 = q3 =
V
C1 + C3

69
1 3
12 = 9 mC
1+3
C2C4
q2 = q4 =
C2 + C4
2 4
=
12 = 16 mC
2+4

C2C3Q
C1C2 + C2C3 + C3C1
C1C2C3V
=
C1C2 + C2C3 + C3C1

q=

(b) When S2 is closed, C1 is in parallel with


C2 and C3 is in parallel with C4.
C1

C3
S2

C2
V1

C4
V2

Therefore,
C3 + C4
V
C1 + C2 + C3 + C4
7
=
12 = 8.4 V
10
C1 + C2
V
V3 = V4 =
C1 + C2 + C3 + C4
3
=
12 = 3.6 V
10
q1 = C1V1 = 1 8.4 = 8.4 mC
q2 = C2q2 = 2 8.4 = 16.8 mC
q3 = C3V3 = 3 3.6 = 10.8 mC
q4 = C4V4 = 4 3.6 = 14.4 mC
V1 = V2 =

26. Initial charge on C1


Q = C1V0
Now, if switch S is thrown to right.
Let charge q flows from C1 to C2 and C3.
By Kirchhoffs voltage law,
q
C2
q

Qq
(Qq)

C1

q
C3
q

q
q Q-q
+
=0
C2 C3
C1
1
1
1 Q
=
q
+
+

C1 C2 C3 C1

C12 (C2 + C3 ) V
C1C2 + C2C3 + C3C1
C1C2C3V
q2 = q3 = q =
C1C2 + C2C3 + C3C1
q1 = Q - q =

e0 A
e AV
, q = CV = 0
d
d
e0 A
(a) C =
, q = q
2d
(As battery is disconnected)
q
V =
=2V
C
1 2 e0 AV 2
(b) Vi =
V =
2C
2d
1
1 e A
Uf = C V 2 = 0 (2V )2
2
2 2d
e AV 2
= 0
d
e0 AV 2
(c) W = Uf - Ui =
2d

27. C =

28. In the steady state, capacitor behaves as


open circuit,
R1

I1

P
I2

IC

E1

E2

R2

E1
= 1 mA and IC = 0
R1 + R2
E1 R2
VPD = I2 R2 =
R1 + R2

I1 = I2 =

When switch is shifted to B,


At this instant,
E1 R2
VPD =
R1 + R2
V
E1
I2 = PD =
= 1 mA
R2
R1 + R2

70
E1 R2
E2 +
E + VPD
R1 + R2
I1 = 2
=
R1
R1
( R1 + R2 ) E2 + E1 R2
=
R1
= 2 mA
IC = I1 + I2 = 1 + 2 = 3 mA

q2 = C2V = 3 18 = 54 mC
After closing the switch,
q1 = C1V1 = 6 12 = 72 mC
q2 = C2V2 = 3 6 = 18 mC
Dq1 = 18 mC, Dq2 = - 36 mC
V
18
30. (a) I =
=
=2A
R1 + R2
9

29. (a) When switch S is open, no current pass


through the circuit,

V=18.0V
I

V=18.0V

C1= 6mF

R1 = 6W

R1

C1

R2 = 3W

+ C2= 3mF

R2

C2

q=
Hence,

Now,

Vb - 0 = 0
Vb = 0
18 - Va = 0 Va = 18 V

Va - Vb = 18 V
(b) a is at higher potential.
(c) When switch S is closed,

and

C1C2
V = 2 18 = 36 mC
C1 + C2

Va - 0 = IR2 Va = 6 V
q 36
=
= 12 V
Vb - 0 =
3
C2

Va - Vb = - 6 V
(b) b is at higher potential.
(c) When switch S is closed, in steady state,
V =18V

V=18.0V
I

q1 +

R1

C1

R1

b
R2

C2

R2

C1

+
q2

C2

I=

V
=2A
R1 + R2

Vb - 0 = IR2 = 2 3
Vb = 6 V
(d) q1 = C1V = 6 18 = 108 mC

Va - vb = 6 V
q1 = C1V1 = 6 12 = 72 mC
q2 = C2V2 = 3 6 = 18 mC
Charge flown through S
= q1 - q2 = 72 - 18 = 54 mC

71
31.
E

II1

I1
q

E1 R2 + E2 R1 V
=
2
R1 + R2
R 3R
Re = R +
=
2
2
q = q0 (1 - e - t / t )
CE
q0 =
2
3 RC
t=
2
CE
q=
(1 - e -2t / 3 RC )
\
2
dq
E -2t / 3 RC
(b) I1 =
=
e
dt 3 R
q
In loop EDBA
+ I1 R - I2 R = 0
C
q
I2 =
+ I1
RC
E
E -2t / 3 RC
=
(1 - e -2t / 3 RC ) +
e
2R
3R
E
=
(3 - e -2t / 3 RC )
6R
Ee =

(a) Consider the circuit as combination of two


cells of emf E and OV.
C

F
I1

I2

B
I

I
E

3R/2

V/2

Objective Questions (Level 1)


2

Q
is independent of d.
2e0 A
q
2. C =
V

5. Incorrect diagram.

1. F =

On connecting the plates V becomes zero.


3. The system can assumed to a parallel
combination of two spherical conductors.
C = C1 + C2 = 4pe0a + 4pe0b
= 4pe0 ( a + b)
q
4. V =
C
On connecting in series
q = q = Charge on any capacitor
C
C =
n
nq
\
V =
= nV
C

6. Charge on capacitor of capacitance


2C
A

60V

A
60V

2C

C C
= V = 30 C
2 2
q
VMN = = 30 V
C
7. For equilibrium,
qE = mg
V 4
q = pr 3rg
d 3
r3
V
q

qE

mg

72
3

V2 r2
q
= 1
V1 r1
q2

R
R

V2 = 4 V

8. Electric field between the plates is uniform


but in all other regions it is zero.

9. Initially the capacitor offers zero resistance.


1W
E
12V

E
4E
=
3
R + R 7R
4
But potential difference across capacitor,
V = IR
4E
10 =
R

7R

E = 17.5 V
I=

6W

3W
1W
4A
2W

12V

i=

12
=4 A
1+2

10. q = CV = CE
11. In the steady state, capacitor behaves as
open circuit. the equivalent diagram is given
by
3
R
4

12. As all the capacitors are connected in series


potential difference across each capacitor is
E 10
V= =
= 2.5 V
4
4
VA - VN = 3V = 7.5 V
VA = 7.5 V
VN - VB = 2.5 V
VB = - 2.5 V
13. Heat produced = Loss of energy
C1C2
=
( V1 - V2 )2
2 (C1 + C2 )
=

2 10-6 2 10-6
2 (2 + 2) 10-6

(100 - 0)

= 5 10-3 J = 5 mJ

3R

q = q0e - t / h
I = I0 e - t / h
P = I 2 R = I02e -2t / h R = P0e -2t / h
h
h =

2
C V + C2V2 E
15. Common potential = 1 1
=
2
C1 + C2
14.

16. VA - VB = 6 + 3 2 -

9
+ 3 3 = 12 V
1

73
17. In the steady state, current through battery
12V

2W

4W

C/2

C
B

3C/2

3C/14

22.
1mF

2mF

2mF

19. For the motion of electron


mu 2 sin 2q
R=
=l
eE
mu 2 sin 2 q
and
H=
=d
2eE
Dividing Eq. (ii) by Eq. (i),
4d
tan q =
l
V 2Ve0
20. V = Ed d =
=
E
6
2 5 8.85 10-12
=
10-7
= 8.85 10-4 = 0.88 mm

(i)

1mF

(ii)
2mF

2
mF
3

y
2mF

23. C1 =

21.
R

1mF

1mF

18. C2 and C3 are in parallel


Hence, V2 = V3
Again Kirchhoffs junction rule
- q1 + q2 + q3 = 0

q1 = q2 + q3

C
B

6W

Potential difference across the capacitor,


3
VAB = 6 = 9 N
2
q = CVAB = 2 9 = 18 mC
\

12
3
I=
= A
6+2 2

2mF
A

C
A

P and Q are at same potential, hence


capacitor connected between them have no
effect on equivalent capacitance.

y
8
mF
3

k1e0 A k2e0 A
+
2d
2d

( k1 + k2 ) e0 A
(Parallel grouping)
2d
1
d
d
(Series grouping)
=
+
C2 2k1e0 A 2k2e0 A
2k1k2 e0 A
C2 =
k1 + k2 d
=

C1 ( k1 + k2 )2 (2 + 3 )2 25
=
=
=
4k1k2
4 2 3 24
C2

74
1

24.

A
P
C

Q
C

CC

B
A
C

d
4

2d

5
C

Capacitance of all other capacitance is same,


e A
i.e., C = 0
but that of formed by plates 4
d
C
as distance between these two
and 5 is
2
plates is 2d.
The equivalent circuit is shown in figure.

2C
B

25. Cases (a), (b) and


Wheatstone bridge.

(c)

are

balanced

26. The given arrangement can be considered as


the combination of three capacitors as shown
in figure.

5 24

C1

C3

6
C

3
C

k1e0 A
2d
A
k2e0
2 = k2e0 A
C2 =
d/2
d
A
k3e0
2 = k3e0 A
C3 =
d/2
d

C2

Hence,

B
A

C1 =

Effective capacitance,
C2C3
e A k
k2k3
C = C1 +
= 0 1 +

C1 + C2
d 2
k2 + k3
27. Here, plate 1 is connected to plate 5 and
plate 3 is connected to plate 6.

C
4C

7
A

B
C

4C

B
C

11C

\Ceq =

11e0 A 11
11
C=
=
7 mF = 11 mF
7
7d
7

75

JEE Corner
Assertion and Reason
1. Capacitance =

q
is constant for a given
V

capacitor.
2. Reason correctly explains the assertion.
1
3. U = qV , W = qV
2
4. For discharging of capacitor
q = q0e - t / t
q
dq
= - 0 e -t / t
t
dt
q
= - 0 e -t / t
RC
Hence, more is the resistance, less will be
the slope.
5. Charge on two capacitors will be same only if
both the capacitors are initially uncharged.

6. As potential difference across both the


capacitors is same, charge will not flow
through the switch.
7. C and R2 are shorted.
8. Time constant for the circuit,
t = RC
9. In series, charge remains same
q2
1
and
U=
U
2C
C
10. In series charge remains same
q
q
V1 = , V2 =
\
C1
C2
On inserting dielectric slab between the
plates of the capacitor, C2 increases and
hence, V2 decreases. So more charge flows to
C2.

Objective Questions (Level 2)


4Q ^
i
e0 A
2Q ^
1. E = i
e0 A
4Q ^
e Ai
0

for x < d

But, I01 = I02

for d < x < 2d


Also,
for 2d < x < 3 d

2. Let E0 = external electric field


and E = electric field due to sheet
E1 = E0 - E = 8
\
E2 = E0 + E = 12
s

E = 2 V/m
=2
2e0
s = 4e0
3. When the switch is just closed, capacitors
behave like short circuit, no current pass
through either 6 W or 5 W resistor.
4. For charging of capacitor
I = I0 e - t / t

t
t
V
t
ln I = ln R RC
ln I = log I0 -

V1 V2
=
R1 R2

1
1
>
R1C1 R2C2

R2C2 > R1C1


As only two parameters can be different,
C1 = C2
R2 > R1
and
V2 > V1
5. Charge on capacitor at the given instant.
q
CE
q= 0 =
2
2
Heat produced = Energy stored in capacitor
q2 CE2
=
=
2C
8
Heat liberated inside the battery,
r
=
Total heat produced
r + 2r
=

CE2
24

76
6. Capacitor is not inside any loop.
E - E0
7. I =
R + R0
q
VBA = - E
C
E

12. VA = - VB
ie, VA - VP = VP - VB
q
q

=
C123 Cn

+q C q

E
B

R0

E0

q
-E
C
( E - E0 ) RC
q = IRC +
R + R0

- E + IR =

8. C =

Cn = C123
1
1
1
1
=
+
+
Cn C1 C2 C3

13. When connected with reverse polarity


C1C2
H=
( V1 + V2 )2
2 (C1 + C2 )
C 2C
25
=
( V + 4V )2 =
CV 2
2 (C + 2C )
3
14.

H1 R2 R
=
=
H2 R1 S
2mF

C1C2
C1 + C2

5W
I1

e A
C1 = C2 = 0
d
e0 A
C=
2d
2e0 A
e A
, C2 = 0
C1 =
d
2d
C1 C2
2e0 A
C =
=
<C
C1 + C2
5d
Re =

9.

R
3

RC
t = ReC =
3
-t / t
q = q0(1 - e
)
= CV (1 - e -3t / RC )
C1C2
10. Energy loss =
( V1 - V2 )2
2 (C1 + C2 )
2 4
=
(100 - 50)2 10-6
2 (2 - 14)
= 1.7 10-3 J
11. q = q0e

[VP = 0]

- t / RC

I=-

q
dq
= 0 e - t / RC
dt RC

at t = 0
q0
= 10
RC
q
V0 = 0 = 10 R = 10 10 = 100 V
C
I=

R
I2

Also, H1 + H2 =

1
1
CV 2 = 2 10-6 (5)2
2
2

H1 + H2 = 25 mJ
H2 = 25 - 10 = 15 mJ
10
10 R
W
= R=
3
15 5
15. When current in the resistor is 1 A.
q
IR +
=E
C
q
1 5 + = 10
2
q = 10 mC

When the switch is shifted to position 2. In


steady state, charge on capacitor
q = 5 2 = 10 mC
but with opposite polarity.
\Total charge flown through 5 V battery,
= q + q = 20 mC
Work done by the battery = 20 5
= 100 mJ
Heat produced = W - DU
But,
DU = 0
\
H = W = 100 mJ

77
16. VA - VB = -

q q q
+ - =0
6 2 3
+

2mF

q2 =

R1

+
A

6mF

3mF

Hence, no charge will flow from A to B.


17. As potential difference across both the
capacitors is same, they are in parallel.
Hence, effective capacitance,
2e A
C= 0
d
e A
1
U = CV 2 = 0 V 2
2
d

2.5 0.5
30
2.5 + 0.5

= 12.5 mC
q
\ Vp - Va = 1 = 18 V
C1
q
Vp - Vb = 2
C3
12.5
=
=5 V
2.5
Vb - Va = 13 V
23. As all the capacitors are identical, potential
difference across each capacitor,
12V
+

18. Rate of charging decreases as it just charged.


19. Potential difference across capacitor = 6 V
1W

2W

I
5V

2V
D

4W
C

2mF

A
C1 = 60mF
C2 = 20mF

21. Common potential,


C V - C1V1 3 100 - 1 100
V= 2 2
=
C1 + C2
1+3
= 25 V

C1
1.0mF

q1
q2
2.5mF
C3

C2
1.5mF
q1
q2

30V

0.5mF
C4

V0

C3 = 30mF
C

q1 + q2 + q3 = 0
C1(C A - V0 ) + C2( VB - V0 ) + C3( VC - V0 ) = 0
C V + C2VB + C3VC
V0 = 1 A
C1 + C2 + C3

1 1.5
30 = 18 mC
1 + 1.5

E
=3 V
4
VN - VB = 3 V
VB = - 3 V
VA - VB = 12 V
VA = 9 V

24. By Kirchhoffs voltage law,

20. While charging


Re = R t = RC
While discharging
Re = 2 R t = 2 RC

22. q1 =

V=

6V

3W

q = CV = 2 6 = 12 mC
In loop ABCD,
I 1 - 2 - 5 = 0 I = 7 A

V0 =
Q

60 6 + 2 20 + 3 30
60 + 20 + 30
49
V
11

25. In the steady state, there will be no current


in the circuit.

78
3mF

B
3mF

3mF

1mF

3mF

3mF 1mF

2mF

1mF
10V

2W

1mF

20W

10W

10V

10W

C1=6mF
q1

C1=1mF
q3

20W
A

q2

10W

resistance.
30. Immediately after switch is closed, capacitor
behaves like short circuit.
V - t / RC
V
31. i1 =
, i2 = e - t / RC
e
R
2R
5t
i1 1 6 RC
= e
i2 2
Increases with time.
rd
d
32. R =
=
A sA
ke0 A
d
ke A
d
t = RC =
0
sA
d
e0 8.85 10-12
=
=
=6s
6
7.4 10-12
C=

10V

C2
3
10
V1 =
V=
10 =
V
C1 + C2
3+6
3

\
26. I1 =

C2=3mF

C1C2
29. H =
( V1 - V2 )2 is independent of
2 (C1 + C2 )

E1
18
=
=3 A
R1 + r1 5 + 1
E2
15
I2 =
=
= 2.5 A
R2 + r2 4 + 2

I1
18V
1W

3mF
A + B
q
I1
5W

4W
q

+
2mF

15V

In loop ABCD,
q
q
- I2 R2 + - I1 R1 = 0
3
2
5q

= 3 5 + 2.5 4 q = 30 mC
6
27. During discharging
q = q0 e - t / t
q0 = CE = 10 mC
at t = 12 s,
q = 10e -12/ 6 = 10e -2
= (0.37)210 mC
C1C2
28. q =
( E1 - E2 )
C1 + C2
C1
q
( E1 - E2 )
Vap =
=
C2 C1 + C2
E - E2
C
= 1
C + C 1
2
1

2W
I2

I2

33. i = i0e -t / t

- ln 4

i0
= i0e RC
2
ln 4
= ln 2
RC
ln 4 = ln 2 RC
RC = 2
2
2
R= =
=4W
C 0.5

34. Potential difference across each capacitor is


equal, hence they are in parallel,
charge on each capacitor
q = CeV = 2 10 = 20 mC
As plate C contributed to two capacitors,
charge on plate,
C = 2q = + 40 mC
35. Charge distribution on the plates of the
capacitor is shown in figure
Q/2

CV +

Q
2

Q/2

CV +

Q
V =
=
C

Q
2

CV +

C
Q
=V +
2C

Q
2

79
36. Let q be the charge on C2 (or charge flown
through the switches at any instant of time)
By Kirchhoffs law
S1

q=

C2q0
C1 + C2

t=

C1 + C2
C1C2 R

C
q0 1 - e

C1

C1C2
C=
C1 + C2

or
+
q

q0 q

q=

where,

+
C2

37. H =

q -q
q
- 0
=0
C2
C1
dq C2q0 - (C1 + C2 ) q
=
dt
C1C2 R
q
t
dq
dt
0 C2q0 - (C1 + C2 ) q = 0 C1C2 R
q
1
=
[ln|C2q0 - (C1 + C2 ) q]0
C1 + C2
1
=
t
C1C2 R
IR +

-
1 - e t

RC

C1C2
( V1 - V2 )2
2 (C1 + C2 )
=

q0
C1C2

2 (C1 + C2 ) C1

C q02
C2q02
=
2C1 (C1 + C2 ) 2C12

38. Electric field in the gap will remain same.


39. Electric field inside the dielectric slab
E V
.
E = =
k kd

More Than One Correct Options


1. Charge distribution is shown in figure
EB = E1 + E2 + E3 + E4
Q
Q
Q
Q
=
+
+
4e0 A 4e0 A 4e0 A 4e0 A
Q
=
2e0 A
+Q

+Q

2R

I2

2C

q2 = 2CE 1 - e RC

t
dq1 E - RC
= e
dt
R
t
dq2 2E - 2 RC
=
e
dt
R
q0
q1 1
1
= 1 =
q2 2
q02 2

+Q

|EA| = |EC| = |E1 + E2 - E3 + E4| but EA and


EC have opposite direction.

2. q1 = CE 1 - e RC

I1

t1 = t 2 = RC

3. V1 =

4 10-3
q
=
= 40V
C 100 10-6

80
U d
Q Qd
V= =
C e0 A

V2
200W

V1

900W

V d

6. When switch S is open

100W

A2

A1
C

I1 = 0
V1
40
2
=
=
A
I2 =
900 900 45
2
80
V2 = I2 200 =
200 =
V
45
9
80
E = V1 + V2 = 40 +
9
440
=
V
9
E
4. Initially I1 = 0,I2 = I =
R
C

I2

I1

I
+

C
+

C 2C 2
= C
C + 2C 3
2
q1 = CE
3
When switch S is closed
Ce = 2C
q2 = 2CE
Charge flown through the battery
4
Dq = q2 - q1 = CE = positive
3
Ce =

7. Let charge q flows to C1 at it falls to the free


end of the wire.
A

C1

C1 q

As the capacitor starts charging,


I2 decreases and I1 increases,
In the steady state
E
I1 = I = , I2 = 0
R
At any instant
P1 = I12 R, P2 = I22 R
Steady state potential difference across the
capacitor,
E
V=
2
1
CE2
U = CV 2 =
2
8
Q2
5. F =
independent of d.
2e0 A
Q
independent of d.
E=
e0 A
U=

Q 2 Q 2d
=
2C 2e0 A

q2
+

q3

2mF
C2

3mF
C3

q2q
+

q3q
+

C2

C3

By Kirchhoffs voltage law,


q2 - q q3 - q
q
+
=0
C2
C3
C1
q2
q
+ 3
C2 C3
q=
1
1
q
+
+
C1 C2 C3
V2 + V3
=
1
1
1
+
+
C1 C2 C3
150 + 120
q=
= 180 mC
1 1
1
+ +
2 3 1.5
q2 = q2 - q = 150 2 - 180
= 120 mC

81
8. C =

e0 A
d

I=
Q 2 Q 2d
=
U d
2C 2e0 A
Q Qd
V= =
V d
C e0 A
e A
1
C = 0 C
d
d

U=

E=

Q
E is independent of d.
e0 A

9. R = 1 + 2 = 3 W, C = 2 F
q0 = CV0 = 2 6 = 12C
At any instant
-t
q = q0 e RC

-t
q0 RC
dq
=
I=
e
dt RC
at t = 0
q
12
I= 0 =
=2 A
RC 3 2

q0
e
RC

- 6 ln 2
6

12 - 1

6
2

1
=1 A
2
Potential difference across 1 W resistor

1 1 = 1 V
Potential difference across 2 W resistor

1 2 = 2 V
\ By Kirchhoffs voltage law, potential
difference across capacitors = 1 + 2 = 3 V.
=2

10. q = C1V1 = 1 10 = 10 mF
6mF
1mF 4mF

3mF

1mF

4mF

9mF

V1

V2

V3

q 10
=
= 2.5 V
4
C2
q 10
=
V
V3 =
9
C3

V2 =

at t = 6 ln 2

Match the Columns


1. C1 = kC1 = 8 mF,

C
C2 = 2 = 2 mF
k
C C
q = 1 2 V = (1.6V ) mC
C1 + C2
q = (2V ) mC

\q < q
2

q2 (1.6 V )
U2 =
=
= 0.64V 2,
C2
2 2
q2
(2)2
U2 =
=
= (1 V ) mC
2C2 2 2
U2 < U2

q 1.6V
=
= 0.2V ,
C2
8
q (2V )
V2 =
=
= 0.5V
C2
4

1.6V
q
,
=
e0 A
e0 A
2V
q
V
=
=
E2 =
ke0 A 2 e0 A e0 A
E2 =

E2 > E2
( a q), (b q), (c q), (s p).
2. Before switch S is closed, charge distribution
is shown in figure (1).
4mF
+

40mC
+
3mF

20mC
+
2mF

V2 =

V2 < V

30V

Fig. 1
After switch S is closed, charge distribution
is shown in figure (2).

82
C

4mF
+

120mC
+
3mF

60mC
+
2mF

2C

Fig. 2
(a s), (b p), (c q ), (d s).
3. (a q), (b p, r), (c q), ( d p, p)
C1
C2
C3
(p)
a

V
V1 = V2 = V3 =
3
CV
q1 = q2 = q3 =
3
C1
C2
a

b
C3

V1 = V2 = V3 = V
q1 = q2 = q3 = CV
(r)

C2
a

b
C1
C3

As combination is series, q1 = q2
q1
=1

q2
U1 C2 8
=
=
U2 C1 9

2V
V
, V2 = V3 =
3
3
2CV
CV
q1 =
, q2 = q3 +
3
3
V1 =

(s)

C1V1 + C2V2 V
=
3
C1 + C2
1
1
U1 = C1V 2 =` CV 2
2
18
1
1
2
U2 = C2V = CV 2
2
9
C1C2
DU =
( V1 - V2 )2
2 (C1 + C2 )
C C
( V - 0 )2
=
2 (C1 + C2 )
1
= CV 2
6
(a r), (b p), (c q).
ke A e A
5.
C1 = 0 + 0
2d
2d
e0 A 3 e0 A
= ( k + 1)
=
2d
2d
1
d
d
=
=
+
C2 2ke0 A 2e0 A
d 1 + k
=

2e0 A k
2ke0 A
4e A

C2 =
= 0
d (1 + k)
3d
C1 9
=
C2 8
V =

30V

(q)

C1

(a s), (b s), (c s).

C2
a

6. Charge distribution is shown in figure.


(a p), (b p, q), (c s), (d p, q, r).
4Q

3
2V
V
, V1 = V2 =
V2 =
3
3
2CV
CV
, q1 = q3 =
q2 =
3
3

4. Common potential

7Q

3Q 3Q

2Q

2Q 2Q

7Q

7Q

23

Magnetics
Introductory Exercise 23.1

1. [ Fe ] = [ Fm ]
[ qE] = [ qvB]

|Fm| = qvB sin q

E = [v] = [L T -1 ]
B


2. F = q ( v B )


\
F ^ v and F ^ B
Because cross product of any two vectors is
always perpendicular to both the vectors.


3. No. As Fm = q ( v B )

If Fm = 0, either B = 0 or sin q = 0,
i.e., q = 0


4. F = q ( v B )
^

= - 4 10-6 10-6 10-2 [(2 i - 3 j + k )


^

(2 i + 5 j - 3 k )]
^

= - 4 10-2(4 i + 8 j + 16 k )
^

= - 16 ( i + 2 j + 4 k ) 10-2 N

Introductory Exercise 23.2


1. As magnetic field can exert force on charged
particle, it can be accelerated in magnetic
field but its speed cannot increases as
magnetic force is always perpendicular to
the direction of motion of charged particle.


2. Fm = - e ( v B )

By Flemings left hand rule, B must be along


positive z-axis.
3. As magnetic force provides necessary
centripetal force to the particle to describe a
circle.
mv2
qvB =
r
mv
r=

qB

(a)

r=

mv
qB

r m
Hence, electron will describe smaller circle.
2pr 2p m
(b)
T=
=
v
qB
1
qB
f =
=
T 2p m
1

f
m
\electron have greater frequency.
4. Electrons are refocused on x-axis at a
distance equal to pitch, i.e.,
n = p = v||T
2p mv cos q
=
eB

84
mv
5. (a) If L r =
,
qB

6. r =

O
q

p q
2 2








q/2

mv
2mk
=
eB
eB
2m eV 1 2mV
=
=
eB
B
e

For electron,
q

r=

= 1.67 10-4 m = 0.0167 cm


For proton
r=

(b) The particle will describe a semi-circle.


Hence, q = p
L
q
(c)
= cos
l
2
L
q
= cos

q
2
2 R sin
2
L
1
= sin q sin q =
R
2
p

q=
6

-31
100
1 2 9.1 10
0.2
1.6 10-19

7. r =

mv
=
qB

-27
100
1 2 1.67 10
-19
0.2
1.6 10

= 7 10-3 m = 0.7 cm
2m k
qB

rp : rd : ra =

m
q

1
2
4
:
:
1
1
2
= 1 : 2 :1

Introductory Exercise 23.3


1. Let at any instant

^
^
^
V = Vx i + Vy j + Vz k
Now, Vx2 + Vy2 = V02 = constant
qE
and
V2 = V0 f
m

V is minimum when V2 = 0
at
and

f =

2. After one revolution, y = 0,

2p mv sin q
qB

Hence, coordination of the particle,


2p mv sin q

= ( x, b) = 0,

qB


^
^
^
3. F = i ( l B ) = ilB [ i ( j + k )]

( F ) = 2 ilB

mv0
qE

Vmin = V0

x = p = pitch of heating

4. No. as i ( i + j + k ) = i j + i ( j k )
^

But i j = 0
^

\ i (i + j + k) = i ( j k)

85

Introductory Exercise 23.4


1. Consider the disc to be made up of large
number of elementary concentric rings.
Consider one such ring of radius x and
thickness dx.
Charge on this ring

w qx dx

px 2
p R2
wq
= 2 x 3dx
R
Magnetic moment of entire disc,
\
wq R
M = dM = 2 x 3dx
R 0
w q R4 1
2
= 2
= w qR
R 4 4
dM = di A =

2. M = i [(OA AB )]
dq =
dq =

q
q
dA +
2px dx
p R2
pR2
2qx dx
2

R
Current in this ring,
dq w dq w qx dx
di =
+
=
T
2p
pR2
\ Magnetic moment of this ring,

^
^
OA = OA cos q j + OA sin q k

^
AB = AB i

M = i OA AB [(cos q j + sin q k ) i ]

3 ^ 1 ^ ^
= 4 0.2 0.1
j + k i
2
2

= (0.04 j - 0.07 k ) A-m 2

Introductory Exercise 23.5


1. (a) B1 = B2 = B3 = B4
m
i
= 0
[sin 45 + sin 45 ]
4p l / 2
2
1

45
l
2

2pr = 4l r =
B=

r=

pm 0 i
= 24.7 mT (inward)
4l

2
m0 i
2. B =

4p x
(As P is lying near one end of conductor 1)
B2 = 0 (Magnetic field on the axis of a current
carrying conductor is zero)
2

m0 i

m 0 2 2i

4p
l
Net magnetic field at the centre of the
square,
m 8 2i
B = B1 + B2 + B3 + B4 = 0
4p
l
2 2 m 0i
=
= 28.3 mT (inward)
pl
=

(b) If the conductor is converted into a


circular loop, then

2l
p

B = B1
m i
= 0
4p x
By right hand thumb rule, direction of
magnetic field at P is inward.

86
3. Magnetic field due to straight conductor at O
i

m0 2i

4p R
Magnetic field at O due to circular loop
m i
B2 = 0
2R
By right hand thumb rule, both the filds are
acting inward.
Hence,
m i
B = B1 + B2 = 0 (1 + p )
2pR
4p 10-7 7
22
=
1 +

7
2p 10 10-2
B1 =

60 m 0i m 0i
(inward)
B2 =

=
360 2b 12b
60 m 0i m 0
(outward)
B3 =
=
=
360 2a 12a
As B3 > B2,
Net magnetic field at P,
B = B3 - B2
=

m 0i 1 1
12 a b

6. AB, AP and BP from Pythagorus triplet,


hence APB = 90
i1
A X
5.0cm
P

B1

13.0cm

12.0 cm

= 58 10-6 T = 58 mT (inward).
4. B1 = B2 = 0 (Magnetic field on the axis of
current carrying conductor is zero)

B X
i2

m 2i ^
B1 = 0 1 PB
4p r1

\
3

m 2i ^
B 2 = 0 2 AP
4p r2

R
2

B = B12 + B22

B3 =
=

1 m 0i m 0i

=
4 2R 8R
4p 10-7 5
8 3 10-2

= 2.62 10-5 T
= 26.2 mT (inward).
5. B1 = B2 = 0 (Magnetic field on the axis of
straight conductor is zero)
2

1
60
4

B2

i
i1
+ 2
r
r
2
1

m0
2p

2
2
4p 10-7
3
3

+

2p
0.05
0.12

= 1.3 10-5 T
= 13 mT
7. t = NIAB cos q
= 100 1.2 0.4 0.3 0.8 cos 30
= 9.98 N-m
Rotation will be clockwise as seen from
above.

87

Introductory Exercise 23.6


1. By right hand thumb rule, direction of
magnetic field due to conductor A, B, C and
D are as shown in figure.
C

A X

At point B
B1 =

m 0 I1
m I
, B2 = 0 2
4pe0 r2
4p r2

Net field at B
B = B2 - B1 =

B
D
BA

2m

BC
BB

m0 1
( I2 - I1 )
4p r2

10-7
(3 - 2) = 0.67 10-4 T
3 10-3

= 67 mT

B X

BA = BB = BC = BD =

3. Consider the cylinder to be made up of large


number of elementary hollow cylinders.

m 0 2I

4p r

Here, I = 5 A

R
r

r=

a
0.2
=
= 0.14
2
2

\Net magnetic field at P

Consider one such cylinder of radius r and


thickness dr.
Current passing through this hollow
cylinder,
di = jdA = j ( 2pr dr ) 2p br 2dr
(a) Total current inside the portion of radius r1,

B = ( BA + BD )2 + ( BB + BC )2
m0 4 2 I

4p
r
10-7 4 2 5
=
= 20 10-6 T
0.2 / 2

r1

I1 = di = 2pb r 2dr
0

= 20 mT
Clearly resultant
downward.

magnetic

field

is

2. At point A
I2

B1

I1

m
b
q

r1

r3
= 2 pb 1
3 0
2
= p br13
3
By amperes circuital law,
B dl = m 0i1
2 pr1

B2

B1 =

m 0 I1

4p r1

B2 = 0 (Magnetic field inside a current


carrying hollow cylinder is zero)
m I
\ Ba = B1 + B2 = 0 1
4p r1
=

10

-7

1 10-3

- 10-4 T

= 100 mT (upward)

2
B1 2 pr1 = m 0 p br13
3

m 0 br12
B1 =

3
(b) Total current inside the cylinder
R

i = 2 pb r 2dr
0

2
p bR3
3
m 2 i m 0 bR3
B2 = 0
=
4p r2
3 r2
=

88

AIEEE Corner
Subjective Questions (Level-1)

^
^
^
7. Let B = Bx i + By j + Bz k

1. Positive. By Flemings left hand rule.


2. Fm = evB sin q
Fe
v=
eB sin q
=


(a) F = q( v B )
^

1.6 10-19 3.5 10-3 sin 60

Bx = - 0.175 T, Bz = - 0.256 T
(b) Cannot be determined by this information.


(c) As F = q ( v B )

3. Fm = qvB sin q
= (2 1.6 10-19 ) 105 0.8 1
= 2.56 10-14 N


4. (a) Fm = e ( v B )
^


F ^B
^

= - 1.6 10-19 [(2.0 106 ) i + (3.0 106 ) j]


^

(0.03 i + 0.15 j)

Hence, B F = 0

^
8. B = B i

^
(a) v = v j

= - (6.24 10-4 N) k


^
F = q ( v B ) = - qvB k


^
(b) = Fm e ( v B ) = - (6.24 10-4 N) k

^
(b) v = v j


5. Fm = e ( v B )
^


^
F = q ( v B ) = qvB j

(6.4 10-19 ) k = - 1.6 10-19[(2 i + 4 j)


^

( Bx i + 3 Bx j)]
^

^
(c) v = - v i


F = q( v B ) = 0

6.4 10-19 k = - 1.6 10-19[2Bx k ]


6.4 10-19
- 3.2 10-19

= - 2.0 T

6. (a) As magnetic force always acts


perpendicular to magnetic field, magnetic
field must be along x-axis.
F1 = qv1B sin q1
5 2 10-3
F1
B=
=
qv1B sin q1 1 10-6 106 1
2

or

B = 10-3 T

^
B = (10-3 T ) i

(b) F2 = qv2 B sin q2


= 1 10-6 106 10-3 sin 90
= 10-3 N
F2 = 1 mN

= - 7.8 10-6 3.8 10 3( Bz i - Bx k )

= 9.46 106 m / s

Bx =

7.6 10-3 i - 5.2 10-3 k

4.6 10-15

^
^
(d) v = v cos 45 i - v cos 45 k


qvB ^
F = q( v B ) = j
2

^
^
(e) v = v cos 45 j - v cos 45 k


qvB ^ ^
F = q( v B ) =
(- j - k)
2
qvB ^ ^
=( j + k)
2
2m k
2 m eV
mv
9. r =
=
=
qB
eB
eB
B=

2mV
r
e

2 9.1 10-31 2 103


1.6 10-19

0.180

89
= 0.36 10-4 T
B = 3.6 10-4 T
mv
qBr
10. (a) r =
v=
qB
m
=

13. The component of velocity along the


magnetic field (i.e., vx ) will remain
unchanged and the proton will move in a
helical path.
z

1.6 10-19 2.5 6.96 10-3


3.34 10

vy cos wt

= 8.33 105 ms -1
T pm
(b) t =
=
2
qB
=

3.14 3.34 10

vy

vy sinwt

-27

1.6 10-19 2.5

= 2.62 10-8 s
1
(c) k = eV = mv2
2
mv2
V=
2e
3.34 10-27 ( 8.33 105 )2
=
2 1.6 10-19
= 7.26 103 V
= 7.26 kV
11. (a) - q. As initially particle is neutral, charge
on two particles must be equal and opposite.
(b) The will collide after completing half
rotation, i.e.,
T pm
t=
=
2
qB

+q q

10.0
12. Here, r =
= 5.0 cm,
2
mv
mv
(a) r =
B=
qB
qr
=

-27

9.1 10-31 1.41 106


1.6 10-19 5 10-2

= 1.6 10-4 T
By Flemings left hand rule, direction of
magnetic field must be inward.
T pm
(b) t =
=
2
qB
=

3.14 9.1 10-31


1.6 10-19 1.6 10-4

= 1.1 10-7 s

At any instant,
Components of velocity of particle along
Y-axis and Z-axis
vy = vy cos q = vy cos wt
and
vz = - vz sin q = vz sin wt
qB
where,
w=
m

^
^
^
\ v = vx i + vy cos w t j - vz sin w t k
14. For the electron to hit the target, distance
G S must be multiple of pitch, i.e.,
GS = np
For minimum distance, n = 1
2p mv cos q

GS = p =
qB

p=

B=

2p 2 mk cos 60
qB

(mv = 2 mk)

2p 2 mk cos 60
qp

2 3.14 2 9.1 10-31 2 1.6 10-16


=

1.6 10-19 0.1


B = 4.73 10-4 T

15. (a) From Question 5 (c)


Introductory Exercise 23.2
L
= sin q L = R sin q
R
R
R sin 60 =
2
mv0
mv
L=
=

2qB 2qB0
(b) Now, L = 2.1 L = 1.05 R
As L > R,

1
2

90
Particle will describe a semicircle and move
out of the magnetic field moving in opposite
direction, i.e.,
^

v = - v = - v0 i
T
pm
t=
=
2 qB0

and

^
^
16. v = (50 ms -1 ) i, B = (2.0 mT ) j
As particle move with uniform velocity,


F = q(E + v B ) = 0

^
E = B v = - ( 0.1 N/C)k

17. If v be the speed of particle at point (0, y, z )


then by work-energy theorem,

^
B = B j

E = EK.
v
vy
q
vx

mv
R
qB0 R
v=
m
qB0 R cos q
vx = v cos q =
m
qB0Z
(Q R cos q = Z)
=
m
2qE0Z q2B02Z 2
Now, vz = v2 - vx2 =
m
m2

^
^
18. Given, E = E j , B = B k,

^
^
v = v cos q j + v sin q k
As protons are moving undeflected,


F = 0 e (E + v B ) = 0
qvB0 =

e ( E j - vB cos q j) = 0

E
B cos q
Now, if electric field is switched off
2p mv sin q 2p mE tan q
p=
=
qB
qB2
or

Z
O

v=

(Component of velocity along magnetic field


= vz = v sin q)
1
mv2
2
But work done by magnetic force is zero,
hence, network done = work done by electric
force
= qEZ
1
qE0Z = mv2
\
2
2qE0Z

v=
m
W = DK =

As the magnetic field is along Y-axis,


particle will move in XZ-plane.
The path of particle will be a cycloid. In this
case, instantaneous centre of curvature of
the particle will move along X-axis.
As magnetic force provides centripetal force
to the particle,
z
vz

v
q

vx

R
X

19. F = I l B sin q
F
0.13
I=
=
lB sin q 0.2 0.067 sin 90
= 9.7 A
Fm
20. For no tension in springs
I I

Fm = mg



I lB = mg

mg
13.0 10-3 10
mg
I=
=
lB 62.0 10-2 0.440

= 0.48 A
By Fleming left hand rule, for magnetic force
to act in upward direction, current in the
wire must be towards right.
21. (a) FBD of metal bar is shown in figure, for
metal to be in equilibrium,
Fm + N = mg
Fm

Fm = mg - N
N
I lB = m - N

mg
V

lB = mg - N
R
R

V=
( mg - N )
lB

91

^
^
l 3 = cd = - (40 10-2 ) i + (40 10-2 m ) j

For largest voltage,


N =0
R mg 25 750 10-3 9.8
V=
=
lB
50.0 10-2 0.450


^
F3 = I ( l 3 B ) = - (0.04 N) k

^
^
l 4 = da = (40 10-2 m ) i - (40 10-2 m ) k

= 817.5 V


^
^
F4 = I ( l 4 B ) = (0.04 N) i + (0.04 N) k

(b) If I lB > mg
I lB - mg = ma
I lB - mg V lB
a=
=
-g
m
Rm
817.5 50 10-2 0.45
=
- 9.8
2 750 10-3

^
24. M = IA M
^

= (40 . 2 10-4 ) (0.60 i - 0.80 j)A-m 2

^
^
B = (0.25 T) i + (0.30 T) k

= 112.8 m/s 2
^


(a) t = M B

22. I = 3.50 A, l = - (1.00 cm ) i

= 0.20 p( 8.0 10-2 )2(0.60 i - 0.80 j)

l = - (1.00 10-2 m ) i

^
(a) B = - (0.65 T) j

= ( - 9.6 i - 7.2 j + 8.0 k ) 10-4 N-m.


^
Fm = I ( l B ) = - (0.023 N) k

(b) U = - M B = - (40.2 10-4 )(0.15) J

^
(b) B = + (0.56 T) k

- 6.0 10-4 J


^
Fm = I ( l B ) = (0.0196 N) j

^
(c) B = - (0.33 T) i


Fm = I ( l B ) = 0

^
(d) B = (0.33 T) i - (0.28 T ) k


^
Fm = I ( l B ) = - (0.0098 N) j

^
^
(e) B = + (0.74 T) j - (0.36 T ) k


^
^
Fm = I ( l B ) = - (0.0259 N) k + (0.0126 N) j
^

= (0.0126 N) j - (0.0259 N) K

^
23. B = (0.020 T) j

^
l1 = ab = - (40.0 cm) j
^

= - (40.0 10-2 m ) j

25. Consider the wire is bent in the form of a


loop of N turns,
L
Radius of loop,
r=
2pN
Magnetic dipole moment associated with the
loop
i L2
M = NiA = Ni pr 2 =
4pN 2
2
iL B
t = MB sin 90 =
4pN
Clearly t is maximum, when N = 1
and the maximum torque is given by
i L2B
tm =
4p
26. Consider the disc to be made up of large
number of elementary rings. Consider on
such ring of radius x and thickness dx.
Charge on this ring,


F1 = I ( l1 B ) = 0

x dx


^
l 2 = bc = (40.0 cm) k
^

= - (400 10-2 m ) k


^
F2 = I ( l 2 B ) = (0.04 N) i

= (40.2 10-4 )( - 0.24 i - 0.18 j + 0.2 k )

dq =

q
2q
2px dx = 2 x dx
p R2
R

92
Current associated with this ring,
dq w dq w q
di =
=
=
x dx
T
2p
pR2
Magnetic moment of this ring
wq
dM = px 2di = 2 x 3dx
R
Magnetic moment of entire disc,
wq R
1
M = dM = 2 x 3dx = w qR2
4
R 0

y
g

c
h

(i)

^
Mabcd = - i l2 k

Magnetic field at the centre of disc due to the


elementary ring under consideration
m di m 0w q2
dB = 0
=
dx
2x
2pR2
Net magnetic field at the centre of the disc,
m wq R
m wq
B = dB = 0 2 dx = 0
0
2pR
2pR
M pR3
\
=
B 2m 0
27. (a) By principle of conservation of energy,
Gain in KE = Loss in PE
KE = - PE cos q + ME
0.80 10-3
K
f cos q = 1 =1 ME
0.02 52 10-3
10
=
13
10
q = cos -1
= 76.7
13
10
(b) q = cos -1
= 76.7
13
Entire KE will again get converted into PE

^
Mefgh = i l2 k

^
Madeh = i l2 j
\Total magnetic moment of the closed path,

^
M = Mabcd + Mefgh + Madeh = i l2 j
31. Circuit is same as in Q.30

^
^
M = i l2 j = j

^
B =2 j

t = M B =0
32. B1 =

m0 I

4p r
Here, B1 and B2 are perpendicular to each
other, hence,
2
l

B = B12 + B22

= 1.5 10-16 s
-19
e 1.6 10
(b) i =
=
= 1.1 10-3 A
T 1.5 10-16

30. Suppose equal and opposite currents are


flowing in sides a d and e h, so that three
complete current carrying loops are formed,

m0 I

4p r
B2 =

28. DU = U2 - U1 = - MB - ( + MB)
= - 2 MB
= - 2 1.45 0.835 = - 2.42 J
-11
2pr 2 3.14 5.3 10
29. (a) T =
=
v
2.2 106

= 1.1 mA
(c) M = p r 2i
= 3.14 (5.3 10-11 )2 1.1 10-3
= 9.3 10-24 A-m 2

e
d

m 0 2I 10-7 2 5

=
4p r
35 10-2

= 2.0 10-6 T
= 2.0 mT
33. Clearly DBOC ~ DAOB
r2 AD
=
\
r6 BC

r2 = 2r
= 100 mm

93
q
a

C
r2

2a

p
q

I
1
a

r
4

and AD = 2BC = 200 mm


r
q = cos -1
= 45
BC
2
2I
m0 I
BBC =
[sin 45 + sin 45 ]
4p r
r
m0
(outwards)
=
4p
m I
BAD = 0 (sin 45 + sin 45 )
4p r2
=

a
q

m0 2 I

4p r2

(inwards)

Net magnetic field at O.


2 m 0I 1 1
B = BBC - BAD =
-
4p r1 r2

1
1
= 2 10-7 2
-3
-3
100 10
50 10
-6
(outwards)
= 2 10 T = 2 mT
34. Let us consider a point P ( x, y) where
magnetic field is zero. Clearly the point must
lie either in 1st quadrant or in 3rd quadrant.

=
B3 = B4 =

m0 I

4p a 2

m0 I
(sin 0 + sin 0)
4p 2a
m
I
= 0
4p 2a 2

Net magnetic field at P


B = B1 + B2 - ( B3 + B4 )
m
I
= 0
4p 2a
m
I m I
q
36. B = 2 0 - 0
=0
4p R 2 R 2p

q = 2 rad.

(inwards)

(outwards)

(inwards)

R
q

37. (a) Consider a point P in between the two


conductors at a distance x from conductor
carrying current I1 (= 25.0 A),

l2

P(xy)

x
l1

I2 = 75.0 A

I1 = 25.0 A
r

B=

35.

m 0 2I1 m 0 2I2

=0
4p y
4p x

I1x = I2 y

I
y= 1x
I
2

q = 45
B1 = B2 =

m0 I
(sin q + sin q)
4p a

Magnetic field at P
m I
m
I
B = 0 1 - 0 2 =0
4p x 4p r - x
I1
I

= 2
x r-x
r - x I2
=

x
I1
I1
25.0

x=
r=
40 = 10 cm
I1 + I2
100.0

94
(b) Consider a point Q lying on the left of the
conductor carrying current I1 at a distance x
from it.

x
I2 = 75.0 A
I1 = 25.0 A

m 0 I1 m 0 I2

=0
4p x 4p r + x
I1
I
= 2
x r+x
I1
25.0
x=
r=
40
I2 - I1
50.0

B=

2 0.0580 2.40 10

-2

4p 10-7 800

I = 2.77 A
(b) On the axis of coil,
2 NIA
m
B= 0 2
4 p ( r + x 2 )3 / 2
r2 + x 2
BC ( r 2 + x 2 )3/ 2

3
r2
B
r

x = 0.0184 m

3/ 2

=2

41. Let the current I2 ( = I ) upwards


I2

I1

I3
I4

= 20 cm
2 N pr 2I
m
38. B = 0 2
4 p ( r + x 2 )3 / 2
But, x = R
m NI
4 2 Br
B= 0
N=
m 0I
4 2r
=

4 2 6.39 10-4 6 10-2


4p 10-7 2.5

N = 69

39. For magnetic field at the centre of loop to be


zero, magnetic field due to straight
conductor at centre of loop must be outward,
hence I1 must be rightwards.
At the centre of the loop

B = - B1 + B2 - B3 + B4
m 2
= 0 [ - I1 + I2 - I3 + I4 ] = 0
4p r
I2 = I1 + I3 - I4
= 10 + 8 - 20
= -2 A
Negative sign indicates that current I is
directed downwards.

m I^
42. B KLM = - 0 i
4R

I2

D
I1

40. (a) B =

B = B1 - B2

m 0 2I1 m 0I2

=0
4p D
2R
pD
I1 =
I2
R

2 BR
m 0 NI
I=
m 0N
2R

m I^
B KNM = 0 j
4R

m I
^
^
B = B KLM + B KNM = 0 ( - i + j)
4R


m Iqv ^
(a) F = q ( v B ) = - 0
k
4R

^
(b) l1 = l 2 = - 2 R k


^
F1 = I ( l1 B ) = 2 IRB i


^
F2 = I ( l 2 B ) = 2 IRB i

^
F = F1 + F2 = 4 IRB i

95

43. (a) Length of each side

bB dl = - m 0I1 = - 5.0 10

-6

T-m

c B dl = m 0( I2 - I1 ) = 2.5 10

dB dl = m 0( I2 + I3 - I1 ) = 5.0 10

46.

I=

1
1
B dl =
3.83 10-7
m0
4p 10-7

= 0.3A
(b) If we integrate around the curve in the
opposite direction, the value of line integral
will become negative, i.e.,
- 3.83 10-7 T-m.

45. B dl = m 0I
As the path is taken counter-clockwise

direction, B dl will be positive if current is


outwards and will be negative if current is
inwards.

B dl = 0
a

-6

T-m

P1

a
qq

(a) By Amperes circuital law

B dl = m 0I

T-m

2 pr
l=
n
p
q=
n
l
pr
a = cot q =
cot q
2
n
m
i
B = n 0 (2 sin q)
4p a
m 2n 2 sin q
= 0
4p p r cot q
p
m 0 i n 2 sin 2
n
=
p
2p 2r cos
n
p
m 0i n 2 sin 2
n
(b) lim B = lim
p
n
n
2p 2r cos
n
m 0i

lim =
n 0
2r

44. B dl = 3.83 10-7 T-m

-6

P2

a
r

Current density
J=

I
2

2I
pa 2

a
pa 2 - 2p
2
Let us consider both the cavities are
carrying equal and opposite currents with
current density J.
Let B1, B2 and B3 be magnetic fields due to
complete cylinder, upper and lower cavity
respectively.
(a) At point P1

m 2I ^
m 2J pa 2 ^
B1 = - 0 1 i = - 0
i
4p r
4p
r
m I^
=- 0 i
pr
2
a
2
J

m
2 I2 ^ m 0
2 ^i
B2 = 0
i=

a
4p
4p r - a
r2
2
m 0I
^
=i
a

4p r -
2

m 0 2 I3 ^
m0
^
B3 =

i=
i
a
4p r + a

4p r +
2
2


B = B1 + B 2 + B 3

4
1
1 ^
+
+
i

a
a
r+
r r
2
2
2
2
m 0I 2r - a ^
B =

i
4pr 4r 2 - a 2
=

m 0I
4p

96
2

2r - a
, towards left.
2
2
4r - a
(b) At point P2

m I
\ (B ) = 0
4pr

y
x

q
q

B1

q
B 2 sin

B2

2 I3
a2
r2 +
4
m 0I

P2

B3
B2 cos q
B3 cos q

B dl = m 0l l
WXYZ

B dl + B dl + B dl + B dl = m 0l l
WX

X Y

Y Z

Z W

B l + 0 + B l + 0 = m 0l l
1
B = m0 l
2
In Fig. 2.

B2

B1

[sin q i - cos q j ]

2p 4r 2 + a 2

Q B2
Fig.2

[sin q i - cos q j ]

1
m0 l
2
B = B1 - B2 = 0,

B1 = B2 =

- 2 cos q ^j
2
2
r
4r + a

r
2r
but, cos q =
=
2
2
a
4r + a 2
r2 +
4
m 0I 2
^
4r
B =
j
\
- 2
2
2p r 4r + a
m I 2r 2 + a 2 ^
= 0 2
j
4pr 4r + a 2

m I 2r 2 + a 2
(B ) = 0 2
, upwards.
4pr 4r + a 2
m 0I
2p

At point Q,
1
m0 l
2
B = B1 + B2 = m 0 l


m I dl r m 0 q ( v r )
48. B = 0
=

4p
4p
r3
r3
B1 = B2 =

y
u
o

47. Let us first find magnetic field due a current


carrying infinite plate.
Consider a rectangular amperian
(WXYZ) as shown in Fig. 1.

B1

At point P,


B = B1 + B 2 + B 3
=

A
l
Fig.1

B3 sin q

m 2I ^ m I ^
B1 = 0 1 j = 0 j
4p r
pr
m0
2 I2
^
^
B2 =

[ - sin q i - cos q j ]
2
4p
a
r2 +
4
- m 0I
^
^
=
[sin q i + cos q j ]
2p 4r 2 + a 2
m
B3 = 0
4p

loop

^
v = ( 8.00 106 ms -1 ) j

^
(a) r = (0.500 m ) i

m 0 q( v r )
B =

4p
r3

97
^

10-7 6.00 10-6[( 8.00 106 j) (0.500) i ]

(0.500)

^
B = - (1.92 10-5 T ) k

10-7 4.00 10-6 [(2.00 105 ^i ) ( - 0.300 ^j)]


B1 =
(0.300)3

^
(b) r = - (0.500 m ) j

= - ( 8.89 10-7 T ) k

m q (v r )
B 2 = 0 2 23 2
4p
r2


m 0 q( v r )
B =

=0
4p
r3

^
(c) r = + (0.500 m ) k

m 0 q( v r )
^
B =

= (1.92 10-5 T ) i
4p
r3

^
^
(d) r = - (0.50 m) j + 0.500 m k

m 0 q( v r )
^
B =

= (1.92 10-5 T ) i
4p
r3
49. q = - 4.80 mC = - 4.80 10-6 C

^
v = (6.80 105 m / s ) i

10-7 ( - 1.5 10-6 )[( 8.00 105 ^i ) ( - 0.400 ^j)]


B2 =
(0.400)2
^

= - ( 7.5 10-7 T ) k

^
B = B1 + B 2 = - (16.4 10-6 T ) k
^

= - (1.64 10-6 T ) k
or

^
(a) r = (0.500 m ) i

m 0 q( v r )
B =

=0
4p
r3

^
(b) r = (0.500 m ) j

m 0 q( v r )
^
B =

= - (1.3 10-6 T ) k
4p
r3

^
^
(c) r = (0.500 m ) i + (0.500 m ) j

51. Magnetic force per unit length on the


conductor AB,
m 2I I
f = 0 1 2
4p
r
For equilibrium
m
f =
g=lg
l
m 2I I
(i)

lg = 0 1 2
4p
r
Suppose wire AB is depressed by x,
f


m 0 q( v r )
^
B =

= - (1.31 10-6 T ) k
4p
r3

^
(d) r = (0.500 m ) k

m 0 q( v r )
^
B =

= (1.31 10-6 T ) j
4p
r3

m q (v r )
50. B1 = 0 1 13 2
4p
r1
y
q

I1

lg

I2

Net force on unit length of wire AB


la = lg - f
m 2I I
m 2I I
= 0 1 2 - 0 1 2
4p
r
4p r - x
m 0 2 I1I2 x
=
4p r ( r - x )
If x << r

v
v2

q2

m 0I1I2
x
2r 2
m II
a = - 0 1 22 x
2l r

la = -

0.300m
0.400m

B = 1.64 10-6 T (inwards)

(ii)

98
General equation of SHM
(ii)
a = - w2x
Hence, motion of wire AB will be simple
harmonic.
From Eqs. (i) and (ii),
m 0I1I2
w=
2 lr 2
2 lr 2
2p
r
T=
= 2p
= 2p
w
m 0I1I2
g
0.01
= 2 3.14
9.8

D
I1

30 A

f CG

m 0 2I1I2

4p
r
f r
I2 =
m0
2I1
4p
4.00 10-5 2.50 10-2
=
10-7 2 0.600

= 8.33 A
(b) As the wires repel each other, current must
be in opposite directions.
m 2I I
53. f CD = 0 1 2
4p
r1

G
I3

r2

r1
10 A

20 A

m 2I I
= 0 2 3
4p
r2

f = f CD - f CG
I
I
mo
2 I2 1 - 3
4p
r1 r2
30
20
2 10

-2
3
10
5
10-2

f = f CD - f CG =
= 10-7

= 0.2 s
52. (a) f =

C
I
FCG 2 FCD
3 cm
5 cm

f = 12 10-4 N = 1.2 10-3 N/m


F = f l = 1.2 10-3 25 10-2
= 3 10-4 N
54. Force per unit length on wire MN
m 2I I
f MN = 0 1 2
4p a
m II L
F = f MN L = 0 1 2
2 pa
Torque acting on the loop is zero because
magnetic field is parallel to the area vector.

Objective Questions (Level 1)


1. Fact
2 pm
2. T =
is independent of speed.
qB
3. Outside the wire
m 2I
where, r is distance from the
B= 0
4p r
centre.
4. The path will be parabola if force acting on
the particle is constant in magnitude as well
as in direction.
m 2I
5. B = 0
4p r
4p rB
m0 =
2I
m Wb / m 2
Units of m 0 =
A
= Wbm -1A -1
6. Fact

7. M = i A , where A = Area vector.


8. Force acting on a closed current carrying
loop is always zero.
9. M = NIA

10. a ^ B a B = 0
^

( x i + j - k ) (2 i + 3 j + 4 k ) = 2x + 3 - 4 = 0
x = 0.5
12. A current carrying closed loop
experiences a force magnetic field.
mv
P
13. r =
,
=
qB qB

\
\

P = mv = momentum.
1
r
q
rp qa
=
rP : ra = 2 : 1
ra q p

never

99

3 cm

3 cm

5 cm
I

v=

2 eV
m

Magnetic force,
Fm = evB sin q
Fm v Fm v
Hence, if potential difference is doubled,
force will become 2 times.
19. Magnetic field at O due to P,

q q

6 cm

14. W = MB (cos q1 - cos q2 )


Here, q1 = p, q2 = p - q
W = MB (cos p - cos ( p - q))
= - MB (1 - cos q)
m0 I
15. BP =
(2 sin q)
4p r

Q
I

5 cm

R/2
2

r = 5 - 3 = 4 cm
= 4 10-2 m

3
sin q =
5
3
10 50 2
5
BP =
4 10-2

m 0 2I
m I

= 0
4p R / 2 p R

(inwards)

Magnetic field at O due to Q,


m
m I
2I
B2 = 0
= 0
4p R / 2 p R

(inwards)

B1 =

-7

= 1.5 10-4 T
= 1.5 gauss.
16. Magnetic field on the axis of current
carrying circular loop,
2M
m
(i)
B1 = 0 2
4 p ( r + x 2 )3 / 2
Magnetic field at the centre of current
carrying circular loop,
m 2M
(ii)
B2 = 0 3
4p r
From Eqs. (i) and (ii),
B2 ( r 2 + x 2 )3/ 2
=
B1
r3
=

(3 2 + 42 )3/ 2
33

125
27
125
B2 =
54 = 150 mT

27



17. F = I( l B ) = I ( ba B )
=



= - I ( ab B ) = I ( B ab)
18. Kinetic energy of electron,
1
K = mv2 = e V
2

R/2

Net magnetic field at O,


B = B1 + B2 =

2 m 0I
pR

20. As solved in Question 16,


B2 x 2 + R2

=
B1 R2

x 2 + R2

R2

x 2 + R2

3/ 2

3/ 2

=8

=4
R2
x= 3 R

21. Component of velocity of particle along


magnetic field, i.e.,
qE
vy =
t = aE t
m
is not constant, hence pitch is variable.
2 mK
mv
22. r =
=
qB
qB
Now, R =

2 mK
eB
R =

2m (2K )
e (3 R)

2
R
3

100
23. Same as question 1. Introductory exercise
23.6.

Note. Her diagram is wrong correct diagram


should be

A x

2
4

O
B x

24. r =

2 mK
2 mqV
mv
[K = qV ]
=
=
qB
qB
qB
r=

2 mV 1

q B

25. Magnetic field due to a conductor of finite


length.
m I
B = 0 (sin a + sin b )
4p r
Here, a = - q2, b = q1 and r = a
m I
B = 0 (sin q1 - sin q2 )
\
2a
26. In case C, magnetic field of conductor 1-2
and 2-3 at O is inward while those of 3-4 and
4-1 at O is outward, hence net magnetic field
at O in this case is zero.
2


27. dF = I ( dl B )

But B ||dl at every point,

hence, dF = 0.
28. B1 = B3 = 0 (Magnetic field on the axis of
current carrying straight conductor is zero)

a
b

- 1 m 0I ^
m I ^
B2 =

k = - 0 k,
4 2b
8b
1 m I ^ m I ^
B3 = 0 k = 0 k
4 2a
8a

B = B1 + B 2 + B 3 + B 4
=

m 0I 1 1 ^
k
8 a b

29. Current associated with electron,


q
I=
= ef
T
m I m ef
B= 0 = 0
2R
2R
30. Same as question 1(a). Introductory Exercise
23.5.
31. At point 1,
Magnetic field due to inner conductor is
non-zero, but due to outer conductor is zero.
Hence, B1 0
At point 2,
Magnetic field due to both the conductors is
equal and opposite.
Hence, B2 = 0
32. Apply Flemings left hand rule or right hand
thumb rule.
33. Magnetic field due to straight conductors at
O is zero because O lies on axis of both the
conductors.
f m 0I m 0If
Hence, B =

=
2p 2x
4px
34. Inside a solid cylinder having uniform
current density,

101
B=

m 0Ir
2 p R2

Here, r = R - x
\

B=

m 0I ( R - x )
2 p R2

35. Magnetic force is acting radially outward on


the loop.

JEE Corner
Assertion and Reason
1. For parabolic path, acceleration must be
constant and should not be parallel or
antiparallel to velocity.
2. By Flemings left hand rule.
3. Magnetic force on upper wire must be in
upward direction, hence current should be in
a direction opposite to that of wire 1.
Reason is also correct but does not explain
Assertion.

2 meV
mv
=
qB
eB

7. For equilibrium

Fe + Fm = 0


q E = - q( v B )


E =- v B = B v


8. Pm = Fm v

4. t = MB sin a
a = 90
\t = MB 0

As Fm is always perpendicular to v ,

Pm = 0

5. F2 = I lBO x1
y

r=

Again, Pe = Fe v , may or may not be zero.


9. Reason correctly explains Assertion.

F2

F4

3
x1

x2

F4 = I lB0x2
F4 > F2
\
Hence, net force is along X-axis.
6. Radii of both is different because mass of
both is different

10. Magnetic force cannot change speed of


particle as it is always perpendicular to the
speed of the particle.
v2
11. a =
R
but R also depends on v.
F
qvB
a= m =
\
m
m

a v

102

Objective Questions (Level 2)


Charge on this cylinder,

1. For net torque to be zero.

T
q

y
O
mg

IAB0 = mgR
mgR
mgR
I=
=
AB0 pR2B0
mg
=
p RB0
2. As it is clear from diagram,
l

I
( 4,0)

(2,0)

Effective length of wire,

^
l = (4 m ) i


F = I( l B)

F
I
a =
= ( l B)
m m
=

2
^
^
^
(4 i ( - 0.02 k )) = 1.6 j m /s 2
0.1

3. Impulse = Change in momentum


I lB dt = mv - 0
lB dq = mv
mv m 2 gh
dq =
=
lB
lB
4. Consider the sphere to be made up of large
number of hollow, coaxial cylinder of
different height and radius. Consider one
such cylinder of radius x, height y and
thickness.
Now, y = 2 R cos q, x = R sin q, dx = R cos q dq

q
(2p yx dx )
4
pR3
3
= 3 q cos 2 q sin q dq
Current associated with this cylinder,
dq w dq 3 w q
di =
=
=
cos 2 q sin q dq
T
2p
2p
Magnetic moment associated with this
cylinder,
3 qw
dM = di A =
cos 2 q sin q dq px 2
2p
3
dM = R2wqA cos 2 q sin 3 q dq
2
0
3
M = dM = R2q cos 2q sin 3 q dq
p2
2
0
3
= R2w q cos 2q (1 - cos 2 q)sin q dq
p
/2
2
dq =

cos 3 q cos 5 q
3 2
R wq

2
5 p/ 2
3

1 2
R wq
5

5. As solved in question 5(c). Introductory


Exercise 23.2.
L
= sin q
R
mV
Here, L = d, R =
qB
qB d
\
= sin q
mV
q V sin q
or
=
m
Bd
6. Force on portion AC will more compared to
that on portion CB.

103
7. Consider an elementary portion of the wire
carrying current I1 of length dx at a distance
x from end B.

mv
qB
qB ( b - a )
v
m

i.e.,

b-a

or

11. Consider an elementary portion of length dx


at a distance x from the pivoted end.
I1

I2

dx

dxe

2a
x

Charge on this portion

Force on this portion


dF = I1dx B
m 2I I
= 0 1 2 dx
4p a + x

q
dx
l
Current associated with this portion
dq qf
di =
=
dx
T
l
Magnetic moment of this portion
pqf 2
dM = px 2di =
x dx
l
pqf l 2
1
M=
x dx = pqfl2
l 0
3
dq =

Total force on wire AB


2 a dx
m
F = dF = 0 2I1I2
a a + x
4p
m II
= 0 1 2 ln 3
2p
8. Magnetic field line due to current carrying
conductor is shown in figure.
z

12. At x = 0, y = 2 m
Effective length of wire
^

l = (4 m ) j
y
x


^
^
\ Fm = I( l B ) = 3(4 j 5 k )
^

= 60 i N
13. Effective length of wire,

2 IA1
m
B1 = 0 2
4p ( x1 + r12 )3/ 2

9.

2I pr12
m
= 0 2
4p ( x1 + r12 )3/ 2
m
2I pr22
B2 = 0 2
4p ( x2 + r22 )3/ 2
B1 r12 ( x22 + r22 )3/ 2
=
B2 r22 ( x12 + r12 )3/ 2
But,
and
\

r1 = x1 tan q
r2 = x2 tan q
B1
=2
B2

10. b - a must be less than or equal to radius of


circular path,

I
T

S 60

3
4d
R

3
a cot 60
4
a
=
2
For equilibrium, I lB = Mg
2Mg

I=
lB
l = ST = 2

104
14. For particle not collide with the solenoid,
radius of path of particle half or radius of
solenoid.
mv r

qB 2
But B = m 0n i


19. As E = - v B
Net force on the particle must be zero.
20. Consider an elementary portion of length dy
at y - y on the wire.
Force on this portion,

rqB m 0qr n i
v>
=
2m
2m


dF = I ( dy B )

16. Magnetic force cannot do work on charged


particle, hence its energy will remain same,
so that q remains same.
Again, magnetic force is always along the
string, it will never produce a torque hence,
T will also remain same.

^
Here, dy = - dy j (Current is directed along

17. Let the x-coordinates of loops be as shown in


figure,

Total force on the wire,

negative y-axis).
^

dF = - I { dy j (0.3 y i + 0.4 y j)}


^

= - 2 10-3( - 0.3 y dy k )
1

F = dF = - 2 10-3 ( - 0.3 y dy k )
0

F = (3 10-4 k ) N


21. E = - v B

then,

b+a

rqB
|E | = vB =
B
m
(5 10-2 )(20 10-6 )(0.1)2
=
(20 10-9 )

F1 = Ia ( B0a ) - 0 = I a 2B0
F2 = Ia ( B0 ( b + a )) - Ia ( B0b)
= I a 2B0
F1 = F2 0

\ E = 0.5 V/m

(1 mg = 10-9 kg )

22. Condition is shown in figure.

B2
1

18. Consider an amperian loop of radius


x ( b < x < c ), threaded by current the
amperian loop,

B1

B
b

I = I =

x 2 - b2
c 2 - b2

c2 - x 2
c 2 - b2

I
\

I
2

I=

m 0I1
2 R1
m I
B2 = 0 2
2 R2
B1 =

m 0I m 0I ( c - x )
=
2px 2px ( c 2 - b2 )

B = B12 + B22
=

m0
2

I
I1
+ 2
R
R
2
1

105
2

m 2I ^
B1 = 0
j
4p a

m 2I ^
B2 = - 0 i
4p a

B3 = 0

5 2
4p 10
5

+
=

5 10-2
2
5 10-2

4p 10-7 2
=
= 4p 10-5 T
2 10-2
-7

23. Initially, net force on the particle is zero.


Hence,
E
V=
B
Now, if electric field is switched off.
mv
E
q = S
r=
=

qB SB2
m

24. For equilibrium,


mg
[f = magnetic force per unit length on
f =
l
the conductors]
m 0 2I1I2

=lg
4p
r
m 2I I
r= 0 1 2

4p l g
=

m 0i ^ ^
B = B1 + B 2 + B 3 =
( j - i)
2pa
26. Effective length, l = AC = 42 + 3 2
=5 m
C

F = I lB = 2 5 2 = 20 N
27. At point P,

10-7 2 100 50
0.01 10

= 0.01 m
Clearly, equilibrium of conductor B is
unstable.

25. If B1, B 2 and B 3 be magnetic fields at the

1
qx

4pe0 ( R2 + x 2 )3/ 2
2 iA
m
B= 0 2
4 p ( R + x 2 )3 / 2
q
qv
Hence, i =
=
T 2 pR
E=

and A = p R2
\

given point due to the wires along x, y and z


axis respectively, then

E
1 1 c2
=
=
B m 0e0 v v

More than One Correct Options


1. B1 =

B2 =

m 0 N1I1 4p 10-7 50 2
=
2 R1
2 5 10-2
m 0 N 2 I2
=
2 R2

= 4p 10-4 T
4p 10-7 100 2
2 10 10

If F = 0


Either, E = - v B ,

E 0, B = 0

-2

= 4p 10-4 T
If current is in same sense,
B = B1 + B2 = 8p 10-4 T
And if current is in opposite sense,
B = B1 - B2 = 0


2. F = Fe + Fm = q ( E + v B )

or


E = 0, or v B = 0


Again, If v B = 0

Either B = 0

or q = 0, i.e., v || B .

1
c =

m 0e0

106
3. The particle will describe a circle in x-y plane
with radius,
2
2
mv 1 8 + 6
r=
=
=5m
qB
1 2
2pm
T=
= p s = 3.14 s
qB

and
4.

t = MB sin q
U = - pE cos q
q = 80
Hence, t = 0, U = pE = maximum.
As PE (U) is maximum, equilibrium is
unstable.

5. Fact.
6. Upward and downward components of force
will cancel each other while leftward force is
more than rightward force, hence net force is
leftwards.


7. F = q E + q ( v B )
^

= q { E0 k + (v j) ( B0 i )}

= q ( E0 - vB0 ) k
E0
, particle will deflect towards
B0
positive z-axis.
E
If v > 0 , particle will deflect towards
B0
negative z-axis.
E
If v = 0 , particle will move undeflected and
B0
its KE will remain constant.

If v <

8. K = e V K V will become double


2mK
R=
R K will become 2 times.
qB
qB
is independent of kinetic energy.
w=
2pm
9. Use right hand thumb rule.
10. For cd to be in equilibrium, force on it must
be repulsive while for ab to be in equilibrium,
force on it must be attractive.
Equilibrium of cd will be stable while that of
ab will be unstable.

Match the Columns


1. ( a r), (b q), (c p), (d r)



Fm = q ( v B ) = - e ( v B )

and Fm = q E = - e E
2. (a r), (b s), (c q), (d p)


As Fm = q ( v B )
By Flemings left hand rule, positively
charged particles deflects towards left and
negatively charged particles deflects towards
right.
2 mK
mv
Again, r =
=
qB
qB
m

r
q
3. (a p, s), (b p, q), (c p, r), (d p, s)
Whenever a closed current carrying loop is
placed in uniform magnetic field, net force
experienced by it is zero.
Also
t = PE sin a

is maximum if a = 90, i.e., in case (b) only.


And
U = - PE cos a
U is positive if a is obtuse, i.e., in cases (a)
and (d).
and U is minimum if a = 0, i.e., in case (c).
4. (a q), (b r), (c s), (d s)
Use right hand thumb rule.
5. (a q), (b r), (c q), (d r)
1

F12

F14

F24

F13 F23

F21

F34

F31

F24

F32

F43

F41

Direction of different forces on different


wires is shown in figure.

107
I
I
^
F = ( l1 B ) + ( l 2 B ) = - B0I l k
2
2

6. (a q, s), (b p, r), (c p, r), (d q, s)

t = 0, because lines of action of force on the


two wires are equal and opposite.

^
If
B = B0 j

When the current is increased or the loop is


moved towards the wire, magnetic flux
linked with the loop increases. As a result of
this, induced current will produce in the loop
to decrease the magnetic field. Because
initial magnetic flux linked with the loop is
inward, induced magnetic flux will be
outward and induced current will be
anti-clockwise and vice-versa.

^
F = B0I l k
Again, lines of action of force on the two
wires are equal and opposite.
t =0

^
^
If
B = B0 ( i + j)

F =0

7. (a r, s), (b r, s), (c q, r), (d p, r)

t =0

y
I/2
I/2
I

If
I/2
x

I/2

Effective lengths of two conductors,


^

l1 = l2 = l i + l j
If

^
B = B0 i

^
B = B0 k

^
^
F = B0I l ( i - j)

|F | = 2B0I l
t =0

24

Electromagnetic
Induction
Introductory Exercise 24.1

1. Magnetic field inside the loop due to current


carrying conductor is inwards.
As the current in the conductor
increases, magnetic flux linked
with the loop increases as a
result of which, induced current
will produce in the loop to
produce an outward magnetic field, i.e.,
induced current will be anti-clockwise.

2. No.
Emf is induced if the field is time varying.
dfB
3.
= induced emf
dt
dfB
2 -3 -1
\
dt = [ V ] = [ML T I ]

Introductory Exercise 24.2


1. If the outward magnetic flux increases,
induced current will be in such a way that it
produces inwards magnetic flux, i.e., it will
be clockwise.
2. Magnetic flux linked with the coil will not
change, hence induced current will be zero.
3. If the current in coil 1 (clockwise) increases,
outward magnetic flux linked with the coil 2
increases and the coil 2 will produce induced
current in clockwise direction to oppose the
change in magnetic flux linked with it.

2 i'

i increasing
i

2 i'

i decreasing

Hence, if the current in coil 1 increases,


induced current will be in same sense and
vice-versa.

Introductory Exercise 24.3


1. fB = BS = B0 S e - at
df
e = - B = a B0 S e -at
dt
2. No.
As,

Fm = i lB = 0

Because, i = 0 as the circuit is not closed. As


net force acting on the bar is zero, no
external force is required to move the bar
with constant velocity.

109
f2 - f1
t
But, f1 = NB1 A cos q, f2 = NB2 A cos q
NA cos q ( B2 - B1 )
\
|e| =
t
|e|t
A=
N( B2 - B1 )cos q

3. |e| =

B=

Induced emf in this portion,


m
2vi
de = B dxv = 0
dx
4p d + x

80.0 10-3 0.4


50 (600 10-6 - 200 10-6 )

3
2

= 1.85 m 2
Side of square, a = A = 1.36 m
Total length of wire = 50 4a
= 50 4 1.36 = 272 m

5. (a) EMF induced in the bar ab,


l dx
m
e = de = 0 2 vi
0d + x
4p
m
= 0 2vi [ln ( d + x )]0l
4p
m 0vi d + l
=
ln
2p
d
m 0vi
l
=
ln 1 +
2p
d

4. (a) Consider an elementary portion of length


dx of the bar at a distance x from end a.
Magnetic field at this point,

(b) Magnetic field in the region ab is


inwards, hence by Flemings left hand
rule, positive charge will move up and a
will be at higher potential.

Or
d

a
dx

m0 2i

4p a + x

x
v l

Use Flemings right hand rule.


(c) No.
As flux linked with the square loop will
remain same.

Introductory Exercise 24.4


1. Potential difference across an inductor,
di
d
V =L
=L
(3 t sin t )
dt
dt
= 3 L [sin t + t cos t ]

Introductory Exercise 24.5


1. (a) Total number of turns on the solenoid,
40 10-2
l
N= =
d 0.10 10-2
= 400
L=
=

m 0N2A
l
4p 10-7 (400)2 0.90 10-4
40 10-2

(b)

= 4.5 10-5 H
di
e = -L
dt
= - 4.5 10-5
= 4.5 10-3 V
= 4.5 mV

0 - 10
0.10

110

Introductory Exercise 24.6


1. Consider a current i is flowing in the outer
loop.
i

Magnetic field at the centre of the loop.


m i
B= 0
2R

As R >> r, magnetic field inside smaller loop


may assumed to be constant.
Hence, magnetic flux linked with the
smaller loop,
m pr 2i
fm = B pr 2 = 0
2R
fm pm 0r 2
M=
=
2R
i

Introductory Exercise 24.7


1. (a) V0 = i0 R = 36 10-3 175 = 6.3 V
(b) i = i0 (1 - e - t / t )
L
where, t =
R
Now, at t = 58 ms
i = 4.9 mA
\ 4.9 = 36 (1 - e -58 / t )
31.1
e -58 / t =

36

t = 397 ms
L
= 397 ms
R
L = 175 397 10-6

= 69 mH
(c)
t = 397 ms
[ e]
[ V ][ t ]
2.
=
[L] =
[ i]
di
dt
[V ]
and
[ R] =
[ i]
L [L]
\
R = [ R] = [T ]
3. (a) Initially
E=L

(b)

di
dt

di E
=
dt L
12.0
=
= 4 A/s
3.00
E = VL + VR

di
+ iR
dt
di 1

= [ E - iR]
dt L
1
=
[12 - 1 7]
3.00
di 5

= = 1.67 A/s
dt 3
L 3
(c) t =
=
R 7
i = i0 (1 - e - t / t )
E
12
= (1 - e -t / t ) =
(1 - e -1.4/ 3 )
R
7

i = 0.639 A
E 12
(d) i0 =
=
= 1.71 A
R 7
E2
4. (a) P = Ei =
(1 - e - t / t )
R
(12)2
=
(1 - e -7t / 3 ) = 20.6 (1 - e -2.33t ) W
7
(b) Rate of dissipation of energy,
PR = i2 R = i02 R (1 - e -7t / R )2
= 20.6 (1 - e -2.33t )2 W
(c) Rate of increase of magnetic energy
di
PL = ei = L
i
dt
= 20.6 ( e -2.33t - e -4.67t ) W
(d) Clearly, P = PR + PL

E=L

5. No.
E = VL + VR and VR cannot be negative in
RL circuit.

111
R1 R2
= 2W
R1 + R2
E
5
i0 =
=
= 0.5 A
R + 8 10
L
1
t=
=
R + 8 10

6. Consider the system as a combination of two


batteries (E1 = 10 V and E2 = 0) as shown
A

R1 = 4W

8W

R=

i2

i1

R2 = 4W
E1 = 10V

1H

E2 = 0
D

8W

1H

E=

E1 R2 + E2 R1
=5 V
R1 + R2

i = i0 (1 - e - t / t )
i = 0.5 (1 - e -10t ) A
\Current through inductor
i = 2.5 (1 - e -10t ) A
In loop ABCDA
di
i1 R1 + 8i + L
- E1 = 0
dt
i1 4 + 8 0.5 (1 - e -10t ) + 1 (5e -10t ) - 10 = 0
i1 = (1.5 - 0.25 e -10t ) A

Introductory Exercise 24.8


1.

[ q] [ i][ T ]
]
[C ] =
=
[V ]
[V ]
[V ] [T ]
[ e]
=
[L] =
[ i]
di
dt
[ LC ] = [ L C ] = [T ]

2. In LC oscillations, magnetic energy is


equivalent to kinetic energy in spring block
system.
dq
dx
i=
v=
dt
dt
Also L is equivalent to inertia (m) in
electricity, hence
1
Magnetic energy = Li2 is equivalent to
2
1
kinetic energy = mv2.
2

|q| = 18 10-6 0.75 3.40


= 46.5 10-6 C
= 46.5 mC
di
1
(b) e = - L
= - L q
dt
LC
-4
q 4.8 10
= =
= 23.3 V
C 18 10-6

4. i0 = wq0

1
LC

where, w =

V0 =

q0
i
= 0
C wC

V0 = i0

20 10-3
L
= 0.1
C
0.5 10-6

= 20 V

3. In LC oscillations,
di
1
di
(a)
=q q = - LC
dt
LC
dt

Introductory Exercise 24.9


dfm
di
= - m 0 nNA
dt
dt
25
= - 4p 10-7
10 5.0 10-4 ( - 0.2)
0.01
e=-

1. (a) B = m 0 ni
fm = NBA = m 0n NAi

112
2

-6

= 3.14 10 V
= 3.14 mV
3.14 10-6
e
(b) E =
=
2pR 2 3.14 25 10-2 10
= 2 10-7 V/m
2. B = (2.00t 3 - 4.00t 2 + 0.8) T
dB
= (6.00t 2 - 8.00t ) T/s
dt
dB
is negative,
dt
hence B is decreasing in that interval.
dB
For t > 1.33 s,
is positive, hence B is
dt
increasing for t > 1.33 s.
(a) For point P2,
dfm2
dB
induced emf, V2 = = - pR2
dt
dt
Induced electric field at P2,
V
R2 dB
E= 2 =
2pr2
2r2 dt

From, t = 0 to t = 1.33 s,

=-

R
(6.00t 2 - 8.00t )
2r2

F = - eE =

R2
(6.00t 2 - 8.00t )
2r2

= 8.0 10-21 N
As magnetic field is increasing in this region,
induced electric field will be anti-clockwise
and hence, electron will experience force in
clockwise sense, i.e., downward at P2.
(b) For point P1,
dfm1
dB
Induced emf, V1 = = - pr12
dt
dt
Induced electric field at P1,
V
1 dB
E = - 1 = - r1
2pr1
2 dt
1
= - r1(6.00t 2 - 8.00t ) = 0.36 V/m
2
At, t = 2.00 s
magnetic field is increasing, hence, induced
electric field will be anti-clockwise, i.e.,
upward at P1 and perpendicular to r1.

AIEEE Corner
Subjective Questions (Level 1)
1. < e > = -

f2 - f1
B ( A2 - A1 )
=t
t
A1 = pr 2 = 3.14 (0.1)2
= 3.14 10-2 = 0.0314
2
2pr
A2 = a 2 =

4
2

2 3.14 0.1
=
= 0.025
4

100 (0.025 - 0.0314)


<e>=0.1
= 6.4 V

2. f1 = NBA = 500 0.2 4 10-4


= 0.04 Wb
f2 = - NBA = - 0.04 Wb
Average induced emf,
(f - f )
<e>=- 2 1
t
Average induced current,
(f - f )
<e>
<i>=
=- 2 1
R
Rt

Charge flowing through the coil


q=<i>t
( f2 - f1 )
( - 0.04 - 0.04)
=q=R
50
0.08
=
= 1.6 10-3 C
50
= 1.6 mC = 1600 mC
3. f1 = NBS, f2 = - NBS
Induced emf,
( f - f ) 2 NBS
<e>=- 2 1 =
t
t
Induced current
< e > 2 NBS
<i>=
=
R
Rt
Charge flowing through the coil,
2 NBS
q=<i>t=
R
4.5 10-6 40
qR

B=
=
2 NS 2 60 3 10-6

113
Hence, equal force in direction of motion of
coil is required to move the block with
uniform speed.

= 0.5 T

^
^
4. B = (4.0 i - 1.8 k ) 10-3 T,

^
S = (5.0 10-4 k ) m 2

f = B S = - 9.0 10-7 Wb
5. e = Blv = 1.1 0.8 5 = 4.4 V
By Flemings right hand rule, north end of
the wire will be positive.
6. A = pr 2 = 3.14 (12 10-2 )2 = 0.045 m 2
(a) For t = 0 to t = 2.0 s
0.5 - 0
dB
= slope =
= 0.25 T/s
dt
2.0 - 1
df
dB
e=- m =-A
dt
dt
= - 0.045 0.25 = - 0.011 V
|e| = 0.011 V
(b) For, t = 2.0 s to t = 4.0 s
dB
= slope = 0 e = 0
dt
(c) For, t = 4.0 s to t = 6.0 s
0 - 0.5
dB
= slope =
= - 0.25
dt
6.0 - 4.0
df
dB
e=- m =-A
= 0.11 V
dt
dt
7. (a) When magnetic flux linked with the coil
changes, induced current is produced in it, in
such a way that, it opposes the change.
Magnetic flux linked with the coil will
change only when coil is entering in (from
3L
L
L
to x = - ) or moving (from x =
x=2
2
2
3L
to x =
) of the magnetic field.
2
Because, of induced current, an opposing
force act on the coil, which is given by
BLv
B2L2v
F = ilB =
BL =
R
R
F
F0 =

B2L2v
R

(b) When the coil is entering into the


magnetic field, magnetic flux linked with
the coil increases and the induced
current will produce magnetic flux in
opposite
direction
and
will
be
counter-clockwise and vice-versa.
i
i0 =
i0
x
i0

8. Consider an elementary section of length dl


of the frame as shown in figure. Magnetic
flux linked with this section,
l
dl
i

dfm = BdB =

m 0 2i

adl
4p x + l

Total magnetic flux linked with the frame,


m ai a dl
fm = dfm = 0
2p 0 x + l
m ai
= 0 [ln ( x + a ) - ln x ]
2p
Induced emf
- dfm
m ai 1
1 dx
e=
= - 0
-
dt
2p x + a x dt
=

m 0a 2i
m 0a 2iv
v=
2px ( x + a )
2px ( x + a )

9. As solved in Qusetion 4. Introductory


Exercise 24.3.
i
d
v
l

F0
3L L O L
2
2
2

3L
2

BLv
R

114
m iv
l
e = 0 ln 1 +
d
2p

Effective emf
E r - E1r2
E= 21
r1 + r2
0.008 15.0 - 0.004 10.0
=
15.0 + 10.0

Here,

i = 10 A
v = 10 ms -1
l = 10.0 cm - 1.0 cm = 9.0 cm
d = 1.0 cm

= 0.0032 V
15 10
rr
r= 12 =
=6W
r1 + r2
25
E
0.0032
i=
=
= 0.003 A = 0.3 mA
R+ r
5+6

4p 10-7 10 10
9.0
e=
ln 1 +

2p
1.0

e = (2 10V ) ln (10) V
10. Induced current
e Blv
=
R
R
Force needed to move the rod with constant
speed = Magnetic force acting on the rod
Blv
ie.,
F = i lB =
lB
R
2
-2 2
B2l2v (0.15) (50 10 ) 2
=
=
R
3
F = 0.00375

i=

11. Suppose the magnetic field is acting into the


plane of paper.
Rods 1 and 2 can be treated as cells of emf
E1 ( = Blv1 ) and E2 ( = Blv2 ) respectively.
2

1
B

v2

v1

r2

12. (a) e = - L

(b) Current flowing from b to a is decreasing,


hence, a must be at higher potential.
13. (a) i = 5 + 16t, |e| = 10mV = 10 10-3 V
di
d
|e| = L
(5 + 16t )
10 10-3 = L
dt
dt
-3
10 10
L=
= 0.625 mH
16
(b) at t = 1 s
i = 5 + 16 (1) = 21 A
Energy stored in the inductor,
1
1
U = Li2 = 0.625 10-3 (21)2
2
2
= 0.138 J
dU
di
P=
= Li
= 0.625 10-3 21 16
dt
dt
= 0.21 W
14. From t = 0 to t = 2.0 ms
V -0
5.0 - 0
=
=0
t - 0 2.0 10-3

r1

R
r2

E1
r1

E
r

V = 2500 t
di
= 2500 t
dt
2500
di = L tdt
i
2500 t
0 di = L 0 tdt
1250 2
i=
t
L
t = 2.0 ms
1250
i=
(2.0 10-3 )2
150 10-3
L

E2

di
= - 0.54 ( - 0.030)
dt
= 1.62 10-2 V

R
i

Now, E1 = Blv1 = 0.010 10.0 10-2 4.00


= 0.004 V
E2 = Blv2 = 0.010 10 0 10-2 8.00
= 0.008 V

at

= 3.33 10-2 A

115
From t = 2.0 ms to t = 4.0 ms
V - 5.0
0 - 0.50
=
t - 2.0 10-3 (4.0 - 2.0) 10-3
V = - 2500 ( t - 2.0 10-3 ) + 5.0
= - 2500 t + 10.0
di
L
= - 2500 t + 10.0
dt
1
di = ( - 2500 t + 10.0) dt
L
1
i = [ - 1250 t 2 + 10.0 t ]
L
at t = 4 s
1
i=
[ - 1250 (4.0 10-3 )2
150 10-3
+ 10.0 (4.0 10-3 )]
-2

= 3.33 10 A
|e|
di
0.0160
15. (a)|e| = L L =
=
dt
di / dt 0.0640

18. (a)|e| = M

di
is constant, induced emf is
dt
constant.
(b) Coefficient of mutual induction remains
same whether current flows in first coil
or second.
di
Hence, | e| = M1
= 0.27 V
dt
As,

19. (a) Magnetic flux linked with the secondary


coil,
f2 = Mi1
f2 0.0320 400
M=
=
i1
6.52
= 1.96 H
(b) f1 = Mi2 = 1.96 2.54 = 4.9784 Wb
Flux per turn through primary coil
f
4.9784
= 1 =
N1
700

= 0.250 H
(b) Flux per turn
Li 0.250 0.720
f=
=
N
400
= 4.5 10-4 Wb
i -i
di
16. |e| = M
=M 2 1
dt
t
12 - 4
-3
50 10 = M
0.5
50 10-3 0 . 5
M=
= 3.125 10-3 H
8
= 3.125 mH
If current changes from 3 A to 9 A in 0.02 s.
i -i
di
|e| = M
=M 2 1
dt
t
9 -3
= 3.125 10-3
0.02
= 0.9375 V
17. (a) Magnetic flux linked with secondary coil,
fm2 = M i1
6.0 10-3 1000
f
M= 2 =
=2H
3
i1
dfm2
di
(b)
e==-M 1
dt
dt
0 -3
= -2
= 30 V
0.2
fm
600 5 10-3
(c) L = 1 =
=1H
3
i1

di
= 3.25 10-4 830
dt
= 0.27 V

= 7.112 10-3 Wb/turn.


20. Same as Question 2. Introductory Exercise
24.4
21.

i = i0(1 - e - t / t )
Rt
E
= (1 - e L )
R
Rt
di E - L
= e
dt L
Power supplied by battery,
Rt
E2
P = Ei =
(1 - e L )
R
Rate of storage of magnetic energy
Rt
Rt
di E2
P1 = Li
=
(1 - e L ) e L
dt
R
10 0.1

Rt

P1
=e L =e
P
L
2
22. (a) t =
=
= 0.2 s
R 10

= e -1 = 0.37

K
E

116
i
But i = 0
2

E 100
(b) i0 =
=
= 10 A
R 10
(c) i = i0(1 - e

t
t

i = 10 (1 - e

1
0.2 )

-5

= 10 (1 - e ) = 9.93 A
23. (a) Power delivered by the battery,
Rt
E2
P = Ei =
(1 - e L )
R
12.8 0.278

(3.24)2
(1 - e 3.56 )
12.8
= 0.82 (1 - e -1 ) = 0.518 W
= 518 mW
(b) Rate of dissipation of energy as heat
Rt
E2
P2 = i2 R =
(1 - e L )2
R
= 0.82 (1 - e -1 )2 = 0.328 W
= 328 mW
(c) Rate of storage of magnetic energy
P1 = P - P2 = 190 mW
di
24. E = VL + VR = L
+ iR
dt
=

VL

VR

Rt
L

i0
= i0 1 - e

Rt
L

1
2
1.25 10-3
L
t = ln 2 =
0.693
R
50.0
= 17.3 10-6 = 17.3 ms
1
1 1

(b) U = Li2 = L i02


2
2 2

1
i=
i0
2
Rt

i
i0 1 - e L = 0

Rt
2 -1
e L =

2
L
2

t = ln
R
2 -1
e

= 30.7 ms .
26. Steady state current through the inductor
L, r
i0

(a) Initially, i = 0
di E 6.00
= =
= 2.40 A/s
dt L 2.50
(b) When, i = 0.500 A
di E - iR 6.00 - 0.500 8.00
=
=
dt
L
2.50
= 0.80 A/s
Rt
E
(c) i =
1-e L

8.00 0.250
6.00

2.5
=
1-e

8.00

-0.8

= 0.750 (1 - e
) = 0.413 A
E 6.00
(d) i0 =
=
= 0.750 A
R 8.00
Rt

25. (a) i = i0 1 - e L

E
r
When the switch S is open
L
t=
R+ r
i0 =

(a) i = i0 e -t / t

(R + r)

E - L
e
r
(b) Amount of heat generated in the solenoid
i=

H = i2r dt = i02r e -2t / t dt


E 2 t -2 t / t
]0
- [ e
r 2

( R + r ) E2
=
2rL

117
When the switch S is open, current i2 flows
in the circuit in clockwise direction and is
given by
i2 = i0e -t / t
E
Here,
i2 =
R2
L
t=
R1 + R2

27. At any instant of time,


5 mH
i1
10 mH
i
20 V

i2 =

5W

L1

di1
di
= L2 2
dt
dt
L1i1 = L2i2
i1 = 2i2

(i)

In steady state,
inductors offer zero resistance, hence
20
i=
=4A
5
But
i1 + i2 = i
4
8
i2 = A, i1 = A
3
3
28. When the switch is closed,
i

i2
i1

L
R1
R2

E
(1 - e - R2 t / L )
R2
di2 E - R2 t / L
= e
dt L
Potential difference across L
di
V + L 2 = E e - R2 t / L = (12e -5t ) V
dt
i2 =

i2
i1
E

L
R1
R2

i2

R1 + R 2
t
L

E -
e
R2

12 -10t
e
= (6 e -10t ) A
2

29. For current through galvanometer to be


zero,
L1,R1

R3

i1

i1
L2,R2

R4
Q

i2

i2

VP = VQ
di1
di
L1
+ i1 R1 = L2 2 + i2 R2
dt
dt
Also,
i1 R3 = i2 R4
From Eqs.(i) and (ii),
di
di
L1 1 + i1 R1 L2 2 + i2 R2
dt
dt
=
i1 R3
i2 R4

(i)
(ii)

(iii)

In the steady state,


di1 di2
=
=0
dt
dt
R1 R2
R
R
\
=
1 = 3
R3 R4
R2 R4
Again as current through galvanometer is
always zero.
i1
= constant
i2
di1 / dt
or
= constant
di2 / dt
di1
dt = i1
or
(iv)
di2 i2
dt
From Eqs. (iii) and (iv),

118
L1 R3 R1
=
=
L2 R4 R2
30. (a) In LC circuit
Maximum electrical energy = Maximum
magnetic energy
1
1
CV02 = Li02

2
2
2

1.50
V

L = C 0 = 4 10-6
50 10-3
i
0

= 3.6 10-3 H
L = 3.6 mH

(b) f =

1
2p LC
1

2 3.14 3.6 10

-3

4 10

-6

= 1.33 103 Hz
= 1.33 kHz
(c) Time taken to rise from zero to maximum
value,
T
1
1
t=
=
=
4 4f 4 1.33 103
= 3 10-3 s = 3 ms.
31. (a) w = 2pf = 2 3.14 103
= 6.28 rad/s
1
1
-3
T = = 3 = 10 s = 1 ms
f 10
(b) As initially charge is maximum, (i.e.., it is
extreme position for charge).
q = q0 cos w t
q0 = CV0 = 1 10-6 100
= 10-4
-4
q = [10 cos (6.28 103 ) t ] C.
\
1
(c) w =
LC
1
1
L= 2 =
w C (6.28 103 )2 10-6

= 2.53 10-3
L = 2.53 mH

(d) In one quarter cycle, entire charge of the


capacitor flows out.
q 4CV
=
t
T
-6
4 10 100
=
= 0.4 A
10-3
<i>=

32. (a) V0 =

q0 5.00 10-6
=
C
4 10-4

= 1.25 10-2 V= 12.5 mV


(b) Maximum magnetic energy = Maximum
electric energy
1 2 q02
Li0 =
2
2C
q

i0 = 0
LC
5.00 10-6
= 8.33 10-4 A
i0 =
0.090 4 10-4
(c) Maximum energy stored in inductor,
1
= L i02
2
1
= 0.0900 ( 8.33 10-4 )2
2
= 3.125 10-8 J
(d) By conservation of energy,
q2 1 2 1 2
+ Li = Li0
2C 2
2
i0
But i =
2
q2 3 2
= Li0
2C 8
i
3
q = 0 3 LC =
q0
2
2
1.732
=
5.00 10-6
2
= 4.33 10-6 C
1
1 1

Um = Li2 = Li02
2
4 2

= 7.8 10-9 J
1
1
=
-3
LC
2.0 10 5.0 10-6

33. (a) w =

= 104 rad/s

di
= w2Q
dt
= (104 )2 100 10-6 = 104 A/s
(b) i = w

Q02

- Q2

= 104 (200 10-6 )2 - (200 10-6 )2 = 0


(c) i0 = wQ0 = 104 200 10-6 = 2 A
(d) i = w Q02 - Q 2

119

i0
= w Q02 - Q 2
2
w Q0
= w Q02 - Q 2
2
1.73 200 10-6
3
Q=
Q0 =
2
2
= 173 mC

34. As initially charge is maximum


q = q0 cos w t
and
|i| = i0 sin w t
1
1
where, w =
=
LC
3.3 840 10-6
19 rad./s
i0 = wq0 = 19 105 10-6
2.0 10-3 A = 2.0 mA
At t = 2.00 ms
q2
q2
(a) Ue =
= 0 (cos 2 wt )
2C 2C
(105 10-6 )2
[cos 2 (38 rad)]
=
2 840 10-6
Ue = 6.55 10-6 J = 6.55 mJ
1
1
(b) Um = Li2 = Li02 (sin w t )
2
2
1
= 3.3 (2 10-3 )2 sin 2(38 rad )
2
= 0.009 10-6 J = 0.009 mj
q2 1
(c) U = 0 = Li02
2C 2
= 6.56 10-6 J = 6.56 mJ
35. As the inward magnetic field is increasing,
induced electric field will be anticlockwise.



E




x





At a distance x from centre of the region,


Magnetic flux linked with the imaginary
loop of radius x
fm = p x 2B
- dfm
dB
e=
= - p x2
dt
dt
Induced electric field,
e
1 dB
E=
= x
2p x 2 dt
At a,
E=

1 dB
, towards left.
r
4 dt

At b ,
E=

1 dB
, upwards.
r
2 dt

At c,
E =0
36. Inside the solenoid,
B = m 0ni
dB
di
= m 0n
dt
dt
Inside the region of varying magnetic field
1 dB 1
di
E= r
= m 0nr
2 dt 2
dt
(a) r = 0.5 cm = 5.0 10-3 m
1
di
E = m 0rn
2
dt
1
= 4p 10-7 5.0 10-3 900 60
2
= 1.7 10-4 V/m
(b) r = 1.0 cm = 1.0 10-2 m
1
di
E=
rn
m0
dt
1
= 4p 10-3 5.0 10-3 900 60
2
= 3.4 10-4 V/m

120

AIEEE Corner
Objective Questions (Level 1)
di
dt
[ V ][ T ] [ ML2 T -3 A -1 ] [T ]
[L] =
=
[ i]
[A ]

1. V = L

8.

= [ ML2 T -2 A -2 ]
2. M n1n2
3. Both will tend to oppose the magnetic flux
changing with them by increasing current in
opposite direction.
4. Moving charged particle will produced
magnetic field parallel to ring, Hence
fm = 0
Velocity of particle increases continuously
due to gravity.
5. Induced electric field can exist at a point
where magnetic field is not present, i.e.,
outside the region occupying the magnetic
field.
6. At, t = 1 s
2W
a

4V

2H

2F
+
a

q = 4t 2 = 4 C
dq
i=
= 8t = 8 A
dt
di
= 8 A/s
dt
di d 2q
As,
=
= Positive
dt
dt
Charge in capacitor is increasing, current i
must be towards left.
di q
Vab = - 2I + 4 - L
dt C
4
= - 2 8 + 4 - 2 8 _ = - 30 V
2
di
d
7. |e| = M
=M
( i0 sin w t )
dt
dt
= w Mi0 cos w t
Maximum induced emf = w Mi0
= 100p 0.005 10
= 5p

1 2 1
L
2
Li0 = CV02 i0
=2
2
2
C
4 10-6

= 2 103 V
1
9. e = B l2w, is independent of t.
2
df Df
10. |e| =
=
dt
t
Df =|e|t = iRt
= 10 10-3 0.5 5
= 25 10-3 Wb
= 25 mWb.
11. As inward magnetic field is increasing,
induced electric field must be anti-clockwise.
Hence, direction of induced electric field at P
will be towards and electron will experience
force towards right (opposite to electric
field).
12. f = at ( t - t ) = att - at 2
df
|e| =
= at - 2at
dt
|e| at - 2at
i=
=
R
R
2
t 2
t ( at - 2 at )
H = i R dt =
dt
0
0
R
3 t
1 ( at - 2 at )
=

R 3 ( - 2a )
0
1
3 3
=
[ - a t - a 3t 3 ]
- 6 Ra
=

13. E = - L

a 2t 3
3R

di
dt

di
+ 15 - iR
dt
= - 5 10-3( - 103 ) + 15 - 5 1
= 15 V

14. VBA = - L

15.

di
= 10 A/s, at t = 0, i = 5A
dt
di
= 10 A/s
dt

121
di
- E =0
dt
= 5 3 + 1 10 - 10 = 15 V
d 2q
q

= w2q0 = 0
=

dt
LC
max

VA - VB = iR + L

di
16.
dt max
17. V = L

di
dt

E 12
=
= 40 A
R 0.3
1
1
U0 = Li02 = 50 10-3 (40)2
2
2
= 40 J
t
Rt

-
E
26. i = i0 1 - e t =
1-e L

25. i0 =

Rt

18. fm = BA cos q

19. A

df
dq
e = - m = BA sin q
dt
dt
dq
iR = BA sin q
dt
dq
dq
R = BA sin q
dt
dt
BA
dq =
sin q dq
R
BA 3p/ 2
q=
sin q dq = 0
R p/ 2
^
^
^
^
= ab k, B = 20t i + 10t 2 j + 50 k

fm = B A = 50 ab
df
e = - m =0
dt

20. E = Vb + iR
Vb = E - iR = 200 - 20 1 . 5 = 170 V
V
N
1
21. s = s Vs = 290 = 10 V
Vp N p
2
ip
N
= s
is
Np

is =

Np
Ns

ip = 2 4 = 8 A

22. Vr = 0, hence magnetic flux linked with the


coil remain same.
- df
\e=
=0
dt
1
23. s = at 2
2
Due to change in magnetic flux linked with
the ring, magnet experiences an upward
force, hence,
a<g
1 2
s < gt s < 5 m
2
di
24. VA - VB = L
=-at
dt

di E - L
= e
dt L
Rt
di
VL = L
= Ee L
dt
at t = 0
VL = E = 20 V
at t = 20 ms
-

R 20 10 -3

L
VL = Ee
5 = 20e 50L
R
= ln 4 R = (100 ln 4) W
50L
|e| 1 df 1
dB
27. |i| =
=
=
NA
R
R dt R
dt
10 10 10-4
=
108 10-4
20
=5A

28. In the steady state, inductor behaves as


short circuit, hence entire current flows
through it.
fm = AB cos q
But, q = 90
fm = 0
\
|e|
1 dfm
30. i =
=R
R dt
dq - nBA d
=
(cos q)

dt
R dt
nBA
dq
=
sin q
R
dt
nBA

dq =
sin q dq
R
2nBA
nBA p
sin q dq =
Q1 =
R 0
R
nBA 2p
Q2 =
sin q dq = 0
R 0
Q2
\
=0
Q1
29.

31. According to Lenzs law, induced current


always opposes the cause producing it.

122
t
e t

Rt
e L

= 1 R

5 2
15
=
1 - e 10 = 3 (1 - e -1 )

= 3 1 - A
e

32. i = i0 1

33. Velocity of AB is parallel to its length.


34. Velocity of rod is parallel to its length.
35. Vc - Va = Vc - Vb = BRV
and
Va - Vb = 0

df
37. E = dt
38. Magnetic flux linked with the
change, hence
e - 1 df
i=
=

=0
R
R dt
1
39. e = Blv cos q = Bl2w cos q
2

coil does not

Q v = w
2

As|cos q|varies from 0 to 1


1
e varies from 0 to B l2w.
2

36. Induced current always opposes the cause


producing it.

JEE Corner
Assertion and Reason
1. Magnetic flux linked with the coil is not
changing with time, hence induced current
is zero.

8. L = m rm 0n 2lA, for ferromagnetic substance,


mr > g
and L does not depends on i.

2. Both Assertion and Reason are correct but


Reason does not explain Assertion.

9. As soon as key is opened


a

3. Induced electric field is non-conservative but


can exert force on charged particles.
4. i = 2t - 8
di
=2
dt

R1
R2

di
= 2 2 = 4 V
Va - Vb = L
dt
di
5.
= ( imax ) w = 1 2 = 2 A/s
dt max
6. Va - Vb = Vc - Va
Vc > Va > Vb

i = i0 =

E
R1

10. Inductors oppose change in current while


resistor does not.

7. Fact.

Objective Questions (Level 2)


1. By conservation of energy
1
1
L i02 = mv02
2
2
m
i0 =
v0
k

iR

2. Wire AB behaves as a cell of emf, E = Blv

iC

123
E Blv
=
R
R
ic = 0
1
1
Uc = CE2 = CB2l2v2
2
2

\ fm = B tan q x 2
df
dx
e = - m = - 2B tan q x
dt
dt
= 2B tan q vx
R = r l = r (2x tan q)
where, r = resistance per unit length of the
conductor.
e Bv
\
i=
=
= constant.
R
r

iR =

3. Apply Flemings left hand rule.


4. For SHM,
v = w A cos wt
e = Blv = Blw A cos wt

e0 cos wt
e =
- e0 cos wt

for nT < t < (2n - 1)


for

T
2

(2n - 1)T
> t > nT
2

5. fm = BA
At any instant when wires have moved
through a distance x,
A = ( a + 2 x )2
fm = B( a + 2x )2
df
dx
|e| = m = 4B( a + 2x )
dt
dt
= 4B( a + 2x ) v0
|e| 4B ( a + 2x ) v0 Bv0
|i| =
=
=
R
l 4 ( a + 2x )
l
6. A = l2
dA
dl
= 2l
= - 2 la
dt
dt

at

dl

a = -
dt

fm = BA
df
dA
e=- m =-B
= 2Bla
dt
dt
l=a
e = 2a aB

7. At this instant, direction of motion of wire


PQ is perpendicular to its length.
e = Blv
\
8. q = CV = CBlv
= 20 10-6 0.5 0.1 0.2
= 0.2 mC
Plate A is positive while plate B is negative.
1
9. fm = BA = B lx
2
l
v
q

But l = 2x tan q

10. fm = BA cos q = BA cos wt


df
e = - m = w BA sin wt
dt
2
But
A=b
e = b2Bw sin wt
\
11. Induced emf
dB
= (1)2 2 10-3
dt
= 2 10-3 V
W = qe = 1 10-6 2 10-3
= 2 10-9 J
e = a2

12. In the steady


capacitor = 0.

state,

current

through

20
=4A
5
f1 = 0, f2 = iL L = 4 500 10-2
= 2 Wb
Df = f2 - f1 = 2 Wb.
1
1 1

13. Li2 = Li02


2
2 2

i0
i=
2
t

-
i
i0 1 - e t = 0

2 -1
e -t / t =
2

2
t = t ln
2 - 1

L
2
= ln

R 2 - 1
m i
14. B = 0
2pa
m iqv
F = qvB = 0
2pa
iL =

15. Consider an elementary section of loop of


width dx at a distance x from wire AB

124
B

18. Induced current opposes change in magnetic


flux.
P

19. VL = E - iR

20. The rod can be assumed as a cell of emf


E = Blv
The equivalent circuit is shown in figure,

C
a
S

dx

2W

m i
dfm = BdA = 0 C dx
2px
m iC b dx m 0iC
b
fm = 0
=
ln
2p a x
2p
a
m C
f
b
M = m = 0 ln
i
2p
a

R
C

x
E

dx

a
B
b

fm =

m 0iy
b
ln
2p
a

b
m 0i ln
dfm
dy
a
e=
=
dt
2p
dt
m 0iv b
e=
ln
a
2p
e m 0iv
b
i=
=
ln
R 2pR
a
Consider an elementary portion of length dx
of the rod at a distance, x from the wire PQ.
Force on this portion,
dF = i dxB
m 2i
= i 0 dx
4p x
b dx
m0
F=i
2i
4p a x
b
m iv b m i
= 0 ln 0 ln
2
p
R
a
2
p
a

=
17. E =

1 dB
r
E r
2 dt

3W

i=

E
Blv 0.50 0.25 4
=
=
2+3
5
5

21. Outside the region of magnetic field, induced


electric field,
r 2 dB Br 2
E=
=
2 R dt
2R
F = qE
1
t = qER = qBr 2
2

P
A

12W

4W

= 0.1 A

16. From previous question

i E

2W

b
A

1 m 0iv b
ln
vR 2p
a

22. VA - V0 = B (2 R)V

VA - V0 = 2 BRV
L
hL
23. L1 =
, L2 =
h+1
h+1
R1

L1

L2

R2
E

hR
hR
, R2 =
R1 =
h+1
h+1
1
1
1
=
+
Le L1 L2
h+1 h+1
=
+
hL
L
(h + 1) 1 1
=
+
L h 1
hL
Le =
( h + 1 )2
hR
L
L
Similarly, Re =
\ t= e =
2
R
R
( h + 1)
e

125
24. i = i0e -t / t
-T / t

Bi0 = i0e
T
t=
1
ln
B
L
25. Given, i02 R = P,
=t
R
when, choke coil is short circuited,
Total heat produced = Magnetic energy
stored in the choke coil
1 2 1
P 1
= Li0 = ( Rt ) = Pt
2
2
R 2
26. i = i0

Rt
e L

Rt
L

=1

Rt
= 0 = not possible.
L
27. To final time constant, short the battery and
find effective resistance in series with
inductor
R

R/2

R
Re =
2
L 2L
t=
=
Re
R
28. When switch is at position 1.
In steady state,
2

31. Initially, inductor offers infinite resistance,


hence,
di
i = 0 and
= maximum
dt
\
E = VL + VC + VR
But
VC = VR = 0
VL = E

32. Same as Q.12 objective Questions (Level 2).


33. Let V0 = Potential of metallic rod,
VB - V0 = B (2 R)V = 2 BR2w
V0 - VC = B (2 R)V = 2 BR2w
Adding Eqs. (i) and (ii), we get
VB - VC = 4 BwR2

34. e = Blvc
vc =

i2

\
R
E

L
L
R

i1

E
R
i2 = 0

i1 =

t = t ln 2
L
t = ln 2
2

30. At the moment when switch is thrown to


position 2,
current in capacitor = current in inductor
just before throwing the switch to position 2,
E
ic =

For current to be constant


i = i0
e

When switch is thrown to position 2.


E
E
i1 = , i2 = R
R
1 2 1 1
2
29. Li = L i0
2
4 2

i
i= 0
2
t

-
i
i0 1 - e t = 0

v1 + v2
2

1
Bl (v1 + v2 )
2
or
dA
e=B
dt
1
dA = l ( dx1 + dx2 )
2
1 dx1 dx2
e = Bl
+

dt
2 dt
1
= Bl (v1 + v2 )
2

(i)
(ii)

dx2

v2

e=

dx1

v1

126
35. Initially, capacitor offer zero resistance and
inductor offers infinite resistance.
Effective circuit is given by
R
R
R
E = 5V
R
R

|e| A dB
37. |i| =
=
R
R dt
B A B [(2b)2 - pa 2 ]
= 0 = 0
R
R
B0(4b2 - pa 2 )
=
R
As inward magnetic field is increasing, net
current must be anticlockwise. Hence
current in inner circle will be clockwise.
38. From Q. 48 Subjective Questions (Level 1).
m ai
a
fm = 0 ln 1 +
2p
x

R/3

5R/6=5 W

Case 1
x = b, a = a
m ai
a
fm1 = 0 ln 1 +
2p
b

m 0 ai b + a
=
ln

2p
b

E =5V

E =5V
R/2

E
=1A
R
t
R t
- 1
E
E - R2 C
36 . i1 =
1 - e L , i2 =
e

R1
R2

i=

4W

5W
5W

L = 10 mH
K
C = 0.1mF

i1

i2

R1= 10W

R2= 10W
E

L = 0.1mH

fm2

x=b-a
a=a
m 0 ai
a
=
ln 1 +

b
a
2p

m 0ai b

ln
b - a
2p

fm2 - fm1
<e>=t
fm - fm1
<e>
<e>=
=- 2
R
Rt
fm2 - fm1
q=<i>t=R
m 0 ai b + a
b
= - ln

ln
2pR b b - a
- m 0ai
b
ln 2
=
b - a2
2pR

m ai
b
|q| = 0 ln 2
b - a2
2pR

E = 20V

6W

Case 2

C = 0.1mF

i = i1 + i2
t
R t
- 1
E
E - R2 C
1-e L +
e
=
R2
R1

at t = 10-3 ln 2
10 10 -3 ln 2
10 3 ln 2

-3
-3
20
20
10 10
i=
e 10 0.1 10
1 - e
+
10
10

1
= 2 1 - + 2 = 2 A
2

39. Magnetic flux linked with the coil.


m n iA
fm = nBA = 0
2r
dfm
|e| =
dt
dfm
iR =
dt

127
1
dq
df
R = m dq = dfm
R
dt
dt
m 0n iA
m 0nA i
q=
di =
2rR 0
2rR
40. Induced electric field inside the region of
varying magnetic fields,
1 dB 1
E= r
= r(6t 2 + 2x ) = 3 r ( t 2 + x ) V/m
2 dt 2
R
At, t = 2.0 s and r =
= 1.25 cm
2
= 1.25 10-2 m
E = 3 1.25 10-2 (4 + x )
= 0.3 V/m
F = eE = 1.6 10-19 0.3
= 48 10-21 N
1 dB
41. E = r
E r
2 dt
42. As inward magnetic field is increasing,
induced electric field must be anticlockwise.
df
dB
43. e = m = pa 2
= pa 2B0
dt
dt
e
1
44. E =
= aB0
2pa 2
45. t = qEa = ia
qEa
a=
=
ma 2
qB0
=
2m

1
aB0a
2
ma 2

46. P = tw = t( at ) = ia 2 t
q2B02
= ma 2
t
m m2

dB
= 2 T/s, A = 0.2 0.4 = 0.08 m 2
dt
0.08
[Q R = r ( b + 2l)]
\ i=
2 = 16 A
1 1.0
As outward magnetic field is increasing,
induced current must be clockwise.
dA
dB
dB
48. e = B
+ A
= Blv + A
dt
dt
dt
At t = 2 s,
B = 4 T, A = 0.2 (0.4 - vt ) = 0.06 m 2
v = 5 cm/s = 0.05 m / s
\ e = - 4 0.2 0.05 + 0.06 2
= - 0.04 + 0.12 = 0.08 V
e
49. F = ilB = lB
R
0.08
=
0.2 4
1 0.8
= 0.008 N
50. When terminal velocity is attained,
power delivered by gravity = power dissipated
in two resistors
mgv = 0.76 + 1.2
1.96
v=
= 1 m/s
0.2 9.8
51. e = Blv = 0.6 1 1 = 0.6 V
e2
P1 =
R1

52. P2 =

At t = 1 s
q2B02a 2
P=
4m
47. i =

e
A dB
=
R R dt

R1 =

e 2 (0.6)2
=
= 0.47 W
P1
0.76

R2 =

e 2 (0.6)2
=
= 0.3 W
P2
1.2

e2
R2

128

More than One Correct Options


1
1
1. e = B v = BLv
2
2
By Flemings left hand rule, P must be
positive w.r.t. Q.
2. dfm = BdA = Ba dx
I
dx

m 0a i
dx
2px
m ai
fm = 0 ln 2
2pi
fm m 0a
M=
=
ln 2
i
2p
If the loop is brought close to the wire, upward
magnetic flux linked with the loop increases,
hence induced current will be clockwise.
=

3. f = Li = Henry-Ampere.
V dt Volt -second
V
L=
=
=
di / dt
di
Ampere
4. t =

e 80
=
= 20 A
R 4
q = it = 20 0.1 = 2 C
Current is not given as a function of time,
hence heat produced in the coil cannot be
determined.
i=

L
=1s
R
E
(1 - e - t / t )
R
= 4 (1 - e -t )

i = i0(1 - e - t / t ) =

At t = ln 2,
i = 2A
Power supplied by battery, P = EI = 16 J/s.
Rate of dissipation of heat in across resistor
= i2 R = 8 J/s
VR = iR = 4 V
Va - Vb = E - VR = 4 V
5. In both the cases, magnetic flux linked with
increases, so current i2 decreases in order to
oppose the change.
6. f1 = BA = 4 2 = 8 Wb, f2 = 0
f -f
8
e= 2 1 =
= 80 V
t
0.1

7. In LC oscillations,
1
w
1
,f =
w=
=
2p 2p LC
LC
1
T = = 2p LC
f
q
i0 = w q0 = 0
LC
q0
di
= wq0 =

LC
dt max
q
di
( VL )max = L
= 0
dt
max C
8. If magnetic field increases, induced electric
field will be anticlockwise and vice-versa.
9. q = 2t 2
i=

dq
= 4t
dt

di
= 4 A/s
dt
dq
As
= Positive
dt
Charge on the capacitor is increasing, hence
current flows from a to b.
1H

a
i

2F
c

4W

t = 1 s, q = 2 C, i = 4 A
di
= 4 A/s
dt
di
= 1 4 = 4 V
Va - Vb = L
dt
q 2
Vb - Vc = = = 1 V
c 2
Vc - Vd = iR = 4 4 = 16 V
Va - Vd = 4 + 1 + 16 = 21 V
1
10. Va - Vb = Bl2w
2
1
Vc - Vb = Bl2w
2
Va - Vc = 0
[Direction of velocity of rod a-c is parallel to
length a-c]

129

Match the Columns


[ B] =

1.

[ ML T -2 ]
[F]
=
[ i][ l]
[ A ][L ]

4. i1 =

E 9
= = 1.6 A
R1 6
i1

= [ ML0 T -2 A -1 ]
[ V ][ dt ] [ ML2 T -3 ][T ]
[L] =
=
[ di]
[A ]

i1

2. i = i0 (1 - e

L
=1s
R
E
=5A
i0 =
R
VR = iR = E (1 - e -t )
VL = E - VR = Et - t

At t = 0,
VL = E = 10 V, VR = 0
t =1s
1

VL = E (1 - e -1 ) = 1 - 10 V
e

10
V
VR =
e

at

3. In LC oscillations,
1
w=
=
LC

1
1

1
4

= 2 rad/s

q0 = 4 C
i0 = wq0 = 8 A

i2 =

When, q = 2 C
VL = VC =

E
(1 - e
R2

At

R2 t

q
= 8V
C

di 1 di
= 8 A/s.
=
dt 2 dt max
di
=1 8 = 8V
VC - VL = L
dt

) = 3 (1 - e -t / 3 )

t = (ln 2) s
q
21/ 3
= i2 R2 = q (1 - e - t / 3 )
1

= q 1 - 1/ 3
2

VL = E - i2 R2 = qe - t / 3 =
VR 2

VR1 = i1 R1 = 9 V
Vbc = VL + VR2 = 9 V
(a s), (b s), (c p), (d p).
5. Induced emf
f(Wb)

di
= w2q0 = 16 A/s.

dt max

When,

R2

)
t=

L
R1

= [ ML2 T -2 A -2 ]
[ LC ] = [T 2 ]
[ fm ] = [ B][ S ]
0
= [ ML T - 2 A -1 ] [L2 ] = [ ML 2 T - 2 A -1 ]
-t / t

i2

|e| = slope of f - t graph


4 -0
=
=2V
2 -0
|e| 2
|i| =
= =1A
R 2
|q| = |i|t = 1 2 = 2 C
As current i is constant
H = i2 Rt = (1)2 2 2 = 4J

25

Alternating Current
Introductory Exercise 25.1

VDC 100
=
= 10 W
I
10
V
150
Z = AC =
= 15 W
I
10
X L = Z 2 - R2 = (15)2 - (10)2
R=

1.

=5 5 W
XL
XL
5 5
L=
=
=
w 2 pf 2 3.14 50
\

7.7 H
As
\

0.036 H
VL = IX L = 50 5 V
= 111.8 V

2. For phase angle to be zero,

XL = XC
1
wL =
wC
1
1
L= 2 =
w C ( 2 pf )2 C
1
=
(360)2 10-6
XL = XC
Z =R
V 120
I=
=
=6A
Z
20

Introductory Exercise 25.2


1. Resonating frequency,

wr =

1
LC

-6

0.03 2 10
=

fr =

2. Resistance of arc lamp,

1
104
6

wr
104
=
2 p 2 3.14 6

1105 Hz
Phase angle at resonance is always 0.

VDC
I
40
=
=4W
10
Impedance of series combination,
V
200
Z = AC =
= 20 W
I
10
R 4 1
Power factor = cos f = =
=
Z 20 5
R=

131

AIEEE Corner
Subjective Questions (Level-1)
1. (a) X L = w L = 2 pfL

3. (a) Power factor at resonance is always 1,

= 2 3.14 50 2
= 628 W
X
(b) X L = w L L = L
w
XL
2
=
=
2 pf 2 3.14 50

as Z = R, Power factor = cos f =


(b) P =

I 0 E0 cos f E20
=
2
2R
(150)2
=
= 75 W
3 150

(c) Because resonance is still maintained,


average power consumed will remain
same, i.e., 75 W.

= 6.37 mH
1
1
(c) X C =
=
wC 2 pfC
1
=
2 3.14 50 2 10-6

4. (a) As voltage is lag behind current,

inductor should be added to the circuit


to raise the power factor.
R
(b) Power factor = cos f =
Z
R
60
250

Z=
=
=
W
cos f 0.720
3

= 1592 W = 1.59 kW
1
1
(d) X C =
C=
wC
w XC
1
=
= 1.59 mF
2 3.14 50 2
2

R
= 1.
Z

X C = Z 2 - R2

2. (a) Z = R + ( X L - X C )

250
2
=
- (60)
3

= R2 + w L wC

1
= (300)2 + 400 0.25 400

8
10-6

= 58 W
1
C=
w XC
1
=
2pf X C

= 367.6 W
V0
120
=
= 0.326 A
Z 367.6
X - XC
(b) f = tan -1 L
R
-1 212.5
= tan
- 35.3
300
As X C > X L voltage will lag behind
current by 35.3.
(c) VR = I 0 R = 0.326 300 = 97.8 V,
VL = I 0 X L = 32.6 V
VC = I 0 X C = 0.326 312.5
= 101.875 V 120 V
I0 =

1
2 3.14 50 58

= 54 mF
For resonance,
1
wr =
LC
1

L= 2
wr C
1
=
(2pf )2 C

L=

1
(2 3.14 50)2 54 10-6

= 0.185 H

132
(b) w = 1000 rad/s
60
\
I=
= 1.2 10-2 A
1000 5

5. V ( t) = 170 sin (6280 t + p / 3) volt

i ( t) = 8.5 sin (6280t + p / 2) amp.


V
170V
O

0.25

0.50

0.75 1.00

1.25

t (ms)

(c) w = 10000 rad/s


60
\
I=
= 1.2 10-3 A
10000 5
7. VR = (2.5 V ) cos [( 950 rad/s ) t ]

170V

(a) I =
i

VR
R

(2.5 V ) cos [(950 rad/s) t ]


300
= ( 8 . 33 mA) cos [(950rad / s) t ]
(b) X L = w L = 950 0.800
= 760 W
(c) VL = I 0 X L cos ( w t + p / 2)
VL = - I 0 X L sin w t
= - 6.33 sin [(950 rad / s) t ] V
8. Given, L = 0.120 H, R = 240 W, C = 7.30 mF,
I rms = 0.450 A , f = 400 Hz
X L = w L = 2pfL
= 2 3.14 400 0.120
= 301.44 W
1
1
XC =
=
wC 2pf C
1
=
2 3.14 400 7.3 10-6
=

(b) f =

12

12

13

12

t (ms)

w
6280
=
= 1000 Hz
2p 2 3.14
= 1 kHz

p p p
(c) f = - =
2 3 6
p
3
=
6
2
As phase of i is greater than V, current is
leading voltage.
(d) Clearly the circuit is capacitive in
nature, we have
R
cos f =
Z
3 R
2
= Z=

R
2
Z
3
170
V
Also,
Z= 0 =
= 20 W
8.5
i0

cos f = cos

3
Z = 10 3 W
2
Z = R2 + X 2C X C = Z 2 - R2
R=

Again,

= 400 - 300 = 10 W
1
1
1
XC =

=
wC
w X C 6280 10
= 1592
. mF
V
V
6. I =
=
XL wL
(a) w = 100 rad/s
\

I=

60
= 0.12 A
100 5

= 54.43 W
R
R
(a) cos f = =
2
Z
R + ( X L - X C )2
240
=
2
(240) + (301.44 - 54.43)2
= 0.697
f = cos -1(0697
.
) 458
.
(b) Z = R2 + ( X L - X C )2
= (240)2 + (301.44 - 54.43)2 = 344 W
(c) Vrms = I rms Z = 0.450 344
= 154.8 V 155 V
(d) Pav = Vrms I rms cos f
= 155 0.450 0.697 = 48.6 W
(e) PR = I 2rms R = (0.450)2 240 = 48.6 W
(f) and (g) Average power associated with
inductor and capacitor is always zero.

133

Objective Questions (Level-1)


1. In an AC circuit, cos f is called power

13. V = 10 cos 100p t

factor.

at t =

2. DC ammeter measures charge flowing in

the circuit per unit time, hence it


measures average value of current, but
average value of AC over a long time is
zero.
Z = R2 + ( X L - X C )2

3.

= R

V = 10 cos 100p
= 10 cos

+ w L w
C

Hence, for X L < X C , Z decreases with


increase in frequency and for X L > X C , Z
increases with increase in frequency.
p
4. As voltage leads current and f < , hence
2
either circuit contains inductance and
resistance
or
contains
inductance,
capacitance and resistance with X L > X C .
5. RMS value of sine wave AC is 0.707 I 0 ,

but can be different for different types of


ACs.

15. X C =

X
1
=
Z
3
-1 1
f = sin

3
3p
I E
17. f =
, P = 0 0 cos f = 0
2
2
VDC
18. R =
= 100 W
I DC
V
100
Z = AC =
= 200 W
I AC 0.5
16.

sin f =

X L = Z 2 - R2 = 100 3 W
L=

7. Z = R + ( X L - X C )

V0 I 0
[V0 and I 0 are peak voltage and
2
current through resistor only]
V
9. Vrms = 0 = 170 V
2
w
120
f =
=
19 Hz
2p 2 3.14

LC

19. I rms =

Vrms
= wCVrms
XC
= 100 1 10-6

200 2
2

I rms = 20 mA

10. Current is maximum at

X L X L 100 3
=
=
w 2pf 2p 50

3
H
=
p

8. P =

w = wr =

p
3
= 10
=5 3 V
6
2

1
1
1
X C or X C
wC
w
f

6. P = I v Ev cos f = 0
2

1
600

14. For purely resistive circuit f = 0.


2

1
s,
600

1
0.5 8 10-6

= 500 rad/s.
I 0 E0 cos f
11. P =
2
100 100
p
=
cos 10-3 = 2.5 W
2
3
1
12. X C =
= if w = 0, i.e., for DC
wC

20. V = VR2 + VL2 = (20)2 + (15)2

= 25 V, V0 = 25 2 V
I V cos f
21. P = 0 0
=0
2

cos f = 0

f = 90
22. R is independent of frequency.
23. L is very high so that circuit consumes

less power.

134
24. tan f =

XL
X
tan 45 = L
100
R
X L = 100 W
w L = 100 W
100
L=
16 mH
2 3.14 103

1
1
1
=
=
=
= 5 ms
4 f 4 50 200
26. f = 60

1
4 220
I 0 V0 cos f
2
P=
=
2
2
= 220 W

25. The minimum time taken by it in reaching

from zero to peak value =

T
4

JEE Corner
Assertion and Reasons
1. X C and X L can be greater than Z because
2

Z = R + ( X L - X C)

Hence, VC = IX C and VL = IX L can be


greater than V = IZ .
2. At

resonance X L = X C , with further


increase in frequency, X L increases but
X C decreases hence voltage will lead
current.
1
, if dielectric slab is inserted
3. fr =
2p LC
between the plates of the capacitor, its
capacitance will increase, hence, fr will
decrease.
4. q = Area under graph

1
1
4 (2 + 3) + 4 (2 + 4)
2
2
= 22 C
q 22
Average current = =
= 3.6 A
t
6
=

5. On inserting ferromagnetic rod inside the

inductor, X L and hence VL increase. Due


to this current will increase if it is lagging
and vice-versa.
6. VR = VL = VC R = X L = X C

Hence, f = 0 and I is maximum.


as Z = R2 + ( X L - X C )2 is minimum.
7. I = I L - I C = 0
8. P = I 2rms R = ( 2)2 10 = 20 W
9. Inductor coil resists varying current.
10. I 0 =

E0
2

2 2

, f = tan -1

R +wL

wL
R

11. At resonance, current and voltage are in

same phase and I 0 =

V0
. Hence, I 0
R

depends on R.

Objective Questions (Level-2)


Single Correct Options
1. For parallel circuit
-1 1 /

XL
-1 4
f = tan
= tan
1
/
R
3

= 53
2. Current will remain same in series circuit

given by

I = I 0 sin ( w t - f)
X

= I 0 sin w t - tan -1 L
R

3. R = R1 + RL = 10 W

X L = w L = 10 W,
1
XC =
= 10 W
wC

135
Reading of ammeter
V
10 2
I rms = rms =
R
10
= 2 A = 1.4 A
Reading of voltmeter,
V = I rms RL = 5.6 V
1
1
4. X C =
=
wC 2p 5 103 1 10-6
p
= 100 W
V 200
IR =
=
= 2 A,
R 100
V
200
IC =
=
=2 A
X C 100

8. For parallel RLC circuit,

I = I 2R + ( I C - I L )2
2

= V0
9.

7. As X C > X L voltage will lag with current.

Again V = VR2 + ( VL - VC )2 = 10 V
V < VC
R V
4
and
cos f = = R =
Z
V
5
Hence, a, b and c are wrong.

VL
2
= tan -1
VR
7

10. Clearly P is capacitor and Q is resistor,

as, VP = V Q , X C = R .
\When connected in series,
Z = X 2C + R2 = 2 R
p
and f = , leading.
4
1
p
A, leading in phase by .
\I =
4 2
4
11. I = I 2R + ( I C - I L )2

Here, I C < I or I L > I


12. I = I L - I C = 0.2 A
13. For a pure inductor voltage leads with

p
current by .
2
14. VR = IR = 220V
Hence it is condition of resonance, i.e.,
VL = VC = 200 V
H1 I 2DC R
I2
15.
= 2
=
=2
H2 I rms R ( I / 2)2
16. H = I 2rms R =

6. As voltage is leading with current, circuit

p
is inductive, and as f = ,X L = R
4
R
R
or
L= =
w 100

1
1

+ wC 2
w
L
R

V = VL2 + VR2 = 72.8 V


f = tan -1

5. Let i = i1 + i2

Another method
p

i = 5 1 + cos + 100 w t
2

2 p
= 5 2 cos + 50 w t
4

p
= 10 cos2 + 50 wt
4

p
1
Average value of cos2 + 50 w t =
4
2
10
\ average value of i =
= 5 A.
2

[Question is wrong. It should be choose


the correct statement].
where, i1 = 5 A, i2 = 5 sin 100 w t A
Average value of i1 = 5 A
Average value of i2 = 0
\Average value of i = 5 A

V
V
V
I = 0 + 0 - 0
R
XC XL

17.

V02 R
I 20 R
=
2
2
2( R + w2 L2 )

VL = IX L = Iw L
I
VC = IX C =
wC

If w is very small,
VL = 0,VC V0 .
V
18. Resistance of coil, R =
=4W
I
When connected to battery
V
12
I=
=
= 1.5 A
R+r 4+4

136
2

19. VR = V -

VC2

1
XC =
wC

=6V

f = tan -1

VC
4
= tan -1
VR
3

20. VC = V 2 - VR2 = 16 V
21.

22.

1
=8 W
50 2500 10-6

Z = R2 + ( X L - X C )2 = 5 W

p
I = I 0 sin t + p
2

p
3p
I = I 0 at + p =
2
2
t =1s
V
I0 = 0
2R

2
Vrms
R
Z2
(12)2 3
=
= 17.28 W
(5)2

Average power = I 2rms R =

25. Already X C > X L , with increase in w, X C

further decrease in w, X C increases and


X L decreases, hence, I will decrease.

3
= 3R
wC
V
I
I0 = 0 = 0
2R
2
VDC 12
23. R =
=
=3 W
I DC
4
X C =

26. For maximum current

w = wr =

1
LC

-6

1 10
=

4.9 10-3

105
rad/s.
7

27. In resonance,

24. X L = Z12 - R2 = (5)2 - (3)2

Z = R2P + X 2C 77 W

=4W

28. In resonance, cos f = 1.

More than One Correct Answers


1. VR2 + VL2 = 10000

(i)
(ii)
(iii)

VL - VC = 120
VR2 + ( VL - VC )2 = (130)2 = 16900
On solving
Vr = 50 V, VL = 86.6 V, VC = 206.6 V
V
50
5
and
cos f = R =
=
V 130 15
As VC > VL , circuit is capacitive in nature.
2.

i = 3 sin w t + 4 cos w t
= R sin ( w t + f)
4
R = 5 and f = tan -1
3
2i
2R 10
im = 0 =
=
p
p
p
If
V = Vm sin w t
current will lead with the voltage.
If
V = Vm cos w t
current will lag with voltage.

P
V
= 1 A, R =
= 60 W
V
I
For AC,
100
Z=
= 100 W
1
X C or X L = Z 2 - R2 = 80 W

3. I =

XL
80
4
=
=
H
w 2p 50 5p
1
1
125
or
C=
=
=
mF
w X C 2p 50 80
p
V
or
R + R =
I

R = 100 - 60 = 40 W
R 1 if R = Z
4. cos f =
=
Z 0 if R = 0
L=

5. As X L > X C , voltage will lead with the

current.
Z = R2 + ( X L - X C )2 = 10 2 W

137
XL - XC p
= = 45
R
4
R
1
cos f = =
Z
2

Vrms = VR2 + ( VL - VC )2 = 100 V ,

f = tan -1

6. As X L > X C , w > wr

with increase in w, X L and hence, Z will


increase while with decrease in w, Z will
first decrease and then increase.
V
7. X c = C = 50 W
I
VR = IR = 80 V
VL = IX L = 40 V

V0 = 100 2 V
V

8. I =

R2 + wL C
w

with change in L or C I may decrease or


1
.
increase depending on effect on wL wC

Match the Columns


1. (a) (p, r), (b) (q, r), (c s), (d) (p)

4. (a q),

Concept based insertion.

R=

2. (a) (p, s) current and voltage are in

same phase so either X C = 0, X L = 0


or
X C = X L 0.
(b) (q)
I = - I 0 cos w t
p
= I 0 sin w t -
2

f = 90 R = 0
(c) (r, s) current is leading with voltage
p
by , either X L = 0 or X C > X L
6
but X C and R are non-zero.
p
(d) (s) current lags with voltage by , R
6
and X L are both non-zero.
3. (a) (q, s), (b) (r, s), (c) (r, s),

(d) (r, s).


V
V2r
and P = 2
Z
Z
with increase in L, C or f , Z may increase
or decrease, hence power and current.
I=

VR 40
=
= 20 W
I
2

(b p)
VC = IX C = 2 30 = 60 V
(c r)
VL = IX C = 2 15 = 30 V
(d s )
V = VR2 + ( VL - VC )2
= 50 V
5. (a s) R is independent of f .

1
f
1
(c r ) X L
f

(b p) X C

(d q)
2

Z = R2 + w L w
C

i.e., first decreases then increases.

You might also like